*NURSING > HESI > HESI RN V1 - V7 Exit Exam (Latest Update) (All)

HESI RN V1 - V7 Exit Exam (Latest Update)

Document Content and Description Below

HESI RN V1 - V7 Exit Exam 1. Following discharge teaching, a male client with duodenal ulcer tells the nurse the he will drink plenty of dairy products, such as milk, to help coat and protect hi... s ulcer. What is the best follow-up action by the nurse? • 2. A male client with hypertension, who received new antihypertensive prescriptions at his last visit returns to the clinic two weeks later to evaluate his blood pressure (BP). His BP is 158/106 and he admits that he has not been taking the prescribed medication because the drugs make him “feel bad”. In explaining the need for hypertension control, the nurse should stress that an elevated BP places the client at risk for which pathophysiological condition? • 3. The nurse observes an unlicensed assistive personnel (UAP) positioning a newly admitted client who has a seizure disorder. The client is supine and the UAP is placing soft pillows along the side rails. What action should the nurse implement? • 4. An adolescent with major depressive disorder has been taking duloxetine (Cymbalta) for the past 12 days. Which assessment finding requires immediate follow-up? • 5. A 60-year-old female client with a positive family history of ovarian cancer has developed an abdominal mass and is being evaluated for possible ovarian cancer. Her Papanicolau (Pap) smear results are negative. What information should the nurse include in the client’s teaching plan? • 6. A client who recently underwear a tracheostomy is being prepared for discharge to home. Which instructions is most important for the nurse to include in the discharge plan? • 7. In assessing an adult client with a partial rebreather mask, the nurse notes that the oxygen reservoir bag does not deflate completely during inspiration and the client’s respiratory rate is 14 breaths / minute. What action should the nurse implement? • 8. During shift report, the central electrocardiogram (EKG) monitoring system alarms. Which client alarm should the nurse investigate firs? • 9. During a home visit, the nurse observed an elderly client with diabetes slip and fall. What action should the nurse take first? • 10. At 0600 while admitting a woman for a schedule repeat cesarean section (C-Section), the client tells the nurse that she drank a cup a coffee at 0400 because she wanted to avoid getting a headache. Which action should the nurse take first? • 11. After placing a stethoscope as seen in the picture, the nurse auscultates S1 and S2 heart sounds. To determine if an S3 heart sound is present, what action should the nurse take first? • 12. A 66-year-old woman is retiring and will no longer have a health insurance through her place of employment. Which agency should the client be referred to by the employee health nurse for health insurance needs? • 13. A client who is taking an oral dose of a tetracycline complains of gastrointestinal upset. What snack should the nurse instruct the client to take with the tetracycline? • 14. Following a lumbar puncture, a client voices several complaints. What complaint indicated to the nurse that the client is experiencing a complication? • “I have a headache that gets worse when I sit up” • “I am having pain in my lower back when I move my legs” • “My throat hurts when I swallow” • “I feel sick to my stomach and am going to throw up” 15. An elderly client seems confused and reports the onset of nausea, dysuria, and urgency with incontinence. Which action should the nurse implement? • 16. The nurse is assisting the mother of a child with phenylketonuria (PKU) to select foods that are in keeping with the child’s dietary restrictions. Which foods are contraindicated for this child? • 17. Before preparing a client for the first surgical case of the day, a part-time scrub nurse asks the circulating nurse if a 3 minute surgical hand scrub is adequate preparation for this client. Which response should the circulating nurse provide? • 18. Which breakfast selection indicates that the client understands the nurse’s instructions about the dietary management of osteoporosis? • 19. The charge nurse of a critical care unit is informed at the beginning of the shift that less than the optimal number of registered nurses will be working that shift. In planning assignments, which client should receive the most care hours by a registered nurse (RN)? • 20. A mother brings her 6-year-old child, who has just stepped on a rusty nail, to the pediatrician’s office. Upon inspection, the nurse notes that the nail went through the shoe and pierced the bottom of the child’s foot. Which action should the nurse implement first? • Cleanse the foot with soap and water and apply an antibiotic ointment • Provide teaching about the need for a tetanus booster within the next 72 hours. • have the mother check the child's temperature q4h for the next 24 hours • transfer the child to the emergency department to receive a gamma globulin injection 21. The mother of an adolescent tells the clinic nurse, “My son has athlete’s foot, I have been applying triple antibiotic ointment for two days, but there has been no improvement.” What instruction should the nurse provide? • 22. A 26-year-old female client is admitted to the hospital for treatment of a simple goiter, and levothyroxine sodium (Synthroid) is prescribed. Which symptoms indicate to the nurse that the prescribed dosage is too high for this client? The client experiences • Bradycardia and constipation • Lethargy and lack of appetite • Muscle cramping and dry, flushed skin • Palpitations and shortness of breath 23. A client with a history of heart failure presents to the clinic with a nausea, vomiting, yellow vision and palpitations. Which finding is most important for the nurse to assess to the client? • 24. The healthcare provider prescribes an IV solution of isoproterenol (Isuprel) 1 mg in 250 ml of D5W at 300 mcg/hour. The nurse should program the infusion pump to deliver how many ml/hour? (Enter numeric value only.) • 25. The pathophysiological mechanism are responsible for ascites related to liver failure? (Select all that apply) • Fluid shifts from intravascular to interstitial area due to decreased serum protein • Increased hydrostatic pressure in portal circulation increases fluid shifts into abdomen • Increased circulating aldosterone levels that increase sodium and water retention 26. The nurse is auscultating a client’s heart sounds. Which description should the nurse use to document this sound? (Please listen to the audio first to select the option that applies) • Murmur • Rationale: A murmur is auscultated as a swishing sound that is associated with the blood turbulence created by the heart or valvular defect. 27. The healthcare provider prescribes celtazidime (Fortax) 35 mg every 8 hours IM for an infant. The 500 mg vial is labeled with the instruction to add 5.3 ml diluent to provide a concentration of 100 mg/ml. How many ml should the nurse administered for each dose? (Enter numeric value only. If rounding is required, round to the nearest tenth) • 28. The nurse notes that a client has been receiving hydromorphone (Dilaudid) every six hours for four days. What assessment is most important for the nurse to complete? • Auscultate the client's bowel sounds • Observe for edema around the ankles • Measure the client’s capillary glucose level • Count the apical and radial pulses simultaneously • Rationale: hydromorphone is a potent opioid analgesic that slows peristalsis and frequently causes constipation, so it is most important to Auscultate the client's bowel sounds 29. A female client is admitted with end stage pulmonary disease is alert, oriented, and complaining of shortness of breath. The client tells the nurse that she wants “no heroic measures” taken if she stops breathing, and she asks the nurse to document this in her medical record. What action should the nurse implement? • 30. A client is receiving a full strength continuous enteral tube feeding at 50 ml/hour and has developed diarrhea. The client has a new prescription to change the feeding to half strength. What intervention should the nurse implement? • 31. A female client reports that her hair is becoming coarse and breaking off, that the outer part of her eyebrows have disappeared, and that her eyes are all puffy. Which follow-up question is best for the nurse to ask? • 32. After a third hospitalization 6 months ago, a client is admitted to the hospital with ascites and malnutrition. The client is drowsy but responding to verbal stimuli and reports recently spitting up blood. What assessment finding warrants immediate intervention by the nurse? • Capillary refill of 8 seconds • bruises on arms and legs • round and tight abdomen • pitting edema in lower legs 33. After the nurse witnesses a preoperative client sign the surgical consent form, the nurse signs the form as a witness. What are the legal implications of the nurse’s signature on the client’s surgical consent form? (Select all that apply) • The client voluntarily grants permission for the procedure to be done • The client is competent to sign the consent without impairment of judgment • The client understands the risks and benefits associated with the procedure 34. Following surgery, a male client with antisocial personality disorder frequently requests that a specific nurse be assigned to his care and is belligerent when another nurse is assigned. What action should the charge nurse implement? • 35. A client with cervical cancer is hospitalized for insertion of a sealed internal cervical radiation implant. While providing care, the nurse finds the radiation implant in the bed. What action should the nurse take? • 36. The client with which type of wound is most likely to need immediate intervention by the nurse? • Laceration • Abrasion • Contusion • Ulceration • Rationale: A laceration is a wound that is produced by the tearing of soft body tissue. This type of wound is often irregular and jagged. A laceration wound is often contaminated with bacteria and debris from whatever object caused the cut. 37. The nurse is planning care for a client admitted with a diagnosis of pheochromocytoma. Which intervention has the highest priority for inclusion in this client’s plan of care? • 38. When caring for a client who has acute respiratory distress syndrome (ARDS), the nurse elevates the head of the bed 30 degrees. What is the reason for this intervention? • To reduce abdominal pressure on the diaphragm • to promote retraction of the intercostal accessory muscle of respiration • to promote bronchodilation and effective airway clearance • to decrease pressure on the medullary center which stimulates breathing • Rationale: a semi-sitting position is the best position for matching ventilation and perfusion and for decreasing abdominal pressure on the diaphragm, so that the client can maximize breathing. 39. When assessing a mildly obese 35-year-old female client, the nurse is unable to locate the gallbladder when palpating below the liver margin at the lateral border of the rectus abdominal muscle. What is the most likely explanation for failure to locate the gallbladder by palpation? • The client is too obese • Palpating in the wrong abdominal quadrant • Deeper palpation technique is needed • The gallbladder is normal • Rationale: a normal healthy gallbladder is not palpable 40. A woman with an anxiety disorder calls her obstetrician’s office and tells the nurse of increased anxiety since the normal vaginal delivery of her son three weeks ago. Since she is breastfeeding, she stopped taking her antianxiety medications, but thinks she may need to start taking them again because of her increased anxiety. What response is best for the nurse to provide this woman? • • encourage her to use stress relieving alternatives, such as deep breathing exercises • • Explain that anxiety is a normal response for the mother of a 3-week-old. • Rationale: there are several antianxiety medications that are not contraindicated for breastfeeding mothers. 41. An older male client with a history of type 1 diabetes has not felt well the past few days and arrives at the clinic with abdominal cramping and vomiting. He is lethargic, moderately, confused, and cannot remember when he took his last dose of insulin or ate last. What action should the nurse implement first? • Start an intravenous (IV) infusion of normal saline • obtain a serum potassium level • administer the client's usual dose of insulin • assess pupillary response to light • Rationale: the nurse should first start an intravenous infusion of normal saline to replace the fluids and electrolytes because the client has been vomiting, and it is unclear when he last ate or took insulin. The symptoms of confusion, lethargy, vomiting, and abdominal cramping are all suggestive of hyperglycemia, which also contributes to diuresis and fluid electrolyte imbalance. 42. A client who received multiple antihypertensive medications experiences syncope due to a drop in blood pressure to 70/40. What is the rationale for the nurse’s decision to hold the client’s scheduled antihypertensive medication? • increased urinary clearance of the multiple medications has produced diuresis and lowered the blood pressure • the antagonistic interaction among the various blood pressure medications has reduced their effectiveness • The additive effect of multiple medications has caused the blood pressure to drop too low • the synergistic effect of the multiple medications has resulted in drug toxicity and resulting hypotension 43. Which client is at the greatest risk for developing delirium? • An adult client who cannot sleep due to constant pain. • an older client who attempted 1 month ago • a young adult who takes antipsychotic medications twice a day • a middle-aged woman who uses a tank for supplemental oxygen 44. Which intervention should the nurse include in a long-term plan of care for a client with Chronic Obstructive Pulmonary Disease (COPD)? • Reduce risks factors for infection • Administer high flow oxygen during sleep • Limit fluid intake to reduce secretions • Use diaphragmatic breathing to achieve better exhalation 45. Which location should the nurse choose as the best for beginning a screening program for hypothyroidism? • A business and professional women's group. • An African-American senior citizens center • A daycare center in a Hispanic neighborhood • An after-school center for Native-American teens 46. A female client has been taking a high dose of prednisone, a corticosteroid, for several months. After stopping the medication abruptly, the client reports feeling “very tired”. Which nursing intervention is most important for the nurse to implement? • Measure vital signs • Auscultate breath sounds • Palpate the abdomen • Observe the skin for bruising 47. A male client reports the onset of numbness and tingling in his fingers and around his mouth. Which lab is important for the nurse to review before contacting the health care provider? • capillary glucose • urine specific gravity • Serum calcium • white blood cell count 48. What explanation is best for the nurse to provide a client who asks the purpose of using the log-rolling technique for turning? • working together can decrease the risk for back injury • The technique is intended to maintain straight spinal alignment. • Using two or three people increases client safety. • turning instead of pulling reduces the likelihood of skin damage 49. A client receiving chemotherapy has severe neutropenia. Which snack is best for the nurse to recommend to the client? 50. Which action should the school nurse take first when conducting a screening for scoliosis? • 51. An unlicensed assistive personnel (UAP) assigned to obtain client vital signs reports to the charge nurse that a client has a weak pulse with a rate of 44 beat/ minutes. What action should the charge nurse implement? • 52. After a sudden loss of consciousness, a female client is taken to the ED and initial assessment indicate that her blood glucose level is critically low. Once her glucose level is stabilized, the client reports that was recently diagnosed with anorexia nervosa and is being treated at an outpatient clinic. Which intervention is more important to include in this client’s discharge plan? • 53. A client with a peripherally inserted central catheter (PICC) line has a fever. What client assessment is most important for the nurse to perform? • 54. The nurse administers an antibiotic to a client with respiratory tract infection. To evaluate the medication’s effectiveness, which laboratory values should the nurse monitor? Select all that apply • 55. A client is admitted to isolation with the diagnosis of active tuberculosis. Which infection control measures should the nurse implement? • Negative pressure environment • contact precautions • droplet precautions • protective environment 56. A school nurse is called to the soccer field because a child has a nose bleed (epistaxis). In what position should the nurse place the child? • 57. A young adult who is hit with a baseball bat on the temporal area of the left skull is conscious when admitted to the ED and is transferred to the Neurological Unit to be monitored for signs of closed head injury. Which assessment finding is indicative of a developing epidural hematoma? • 58. A female client with breast cancer who completed her first chemotherapy treatment today at an out-patient center is preparing for discharge. Which behavior indicates that the client understands her care needs • 59. Which instruction should the nurse provide a pregnant client who is complaining of heartburn? • 60. A client is admitted to the intensive care unit with diabetes insipidus due to a pituitary gland tumor. Which potential complication should the nurse monitor closely? • Hypokalemia • Ketonuria. • Peripheral edema • Elevated blood pressure • Rational: pituitary tumors that suppress antidiuretic hormone (ADH) result in diabetes insipidus, which causes massive polyuria and serum electrolyte imbalances, including hypokalemia, which can lead to lethal arrhythmias. 61. A female client reports she has not had a bowel movement for 3 days, but now is defecating frequent small amount of liquid stool. Which action should the nurse implement? • 62. After changing to a new brand of laundry detergent, an adult male reports that he has a fine itchy rash. Which assessment finding warrants immediate intervention by the nurse? • 63. The nurse should teach the parents of a 6 year-old recently diagnosed with asthma that the symptom of acute episode of asthma are due to which physiological response? • I 64. A 10 year old who has terminal brain cancer asks the nurse, "What will happen to my body when I die?" How should the nurse respond? • 65. The nurse is assessing a 3-month-old infant who had a pylorotomy yesterday. This child should be medicated for pain based on which findings? Select all that apply: • Restlessness • Clenched Fist • Increased pulse rate • Increased respiratory rate. • Increased temperature • Peripheral pallor of the skin 66. The nurse is preparing to administer an oral antibiotic to a client with unilateral weakness, ptosis, mouth drooping and, aspiration pneumonia. What is the priority nursing assessment that should be done before administering this medication? 67. nurse who is working on a surgical unit receives change of shift report on a group of clients for the upcoming shift. A client with which condition requires the most immediate attention by the nurse? • Gunshot wound three hours ago with dark drainage of 2 cm noted on the dressing. • Mastectomy 2 days ago with 50 ml bloody drainage noted in the Jackson-pratt drain. • Collapsed lung after a fall 8h ago with 100 ml blood in the chest tube collection container • Abdominal-perineal resection 2 days ago with no drainage on dressing who has fever and chills. • Rationale: the client with an abdominal- perineal resection is at risk for peritonitis and needs to be immediately assessed for other signs and symptoms for sepsis. 68. The nurse is caring for a client who had gastric bypass surgery yesterday. Which intervention is most important for the nurse to implement during the first 24 postoperative hours? • Measure hourly urinary output. • Rationale: a serious early complications of gastric bypass surgery is an anastomoses leak, often resulting in death. 69. When preparing to discharge a male client who has been hospitalized for an adrenal crisis, the client expresses concern about having another crisis. He tells the nurse that he wants to stay in the hospital a few more days. Which intervention should the nurse implement? • 70. An adult female client tells the nurse that though she is afraid her abusive boyfriend might one day kill her, she keeps hoping that he will change. What action should the nurse take first? • 71. In caring for a client with Cushing syndrome, which serum laboratory value is most important for the nurse to monitor? • Lactate • Glucose • Hemoglobin • Creatinine 72. Azithromycin is prescribed for an adolescent female who has lower lobe pneumonia and recurrent chlamydia. What information is most important for the nurse to provide to this client? • 73. A client in the emergency center demonstrates rapid speech, flight of ideas, and reports sleeping only three hours during the past 48h. Based on these finding, it is most important for the nurse to review the laboratory value for which medication? • 74. A male client who is admitted to the mental health unit for treatment of bipolar disorder has a slightly slurred speech pattern and an unsteady gait. Which assessment finding is most important for the nurse to report to the healthcare provider? • 75. A client was admitted to the cardiac observation unit 2 hours ago complaining of chest pain. On admission, the client’s EKG showed bradycardia, ST depression, but no ventricular ectopy. The client suddenly reports a sharp increase in pain, telling the nurse, “I feel like an elephant just stepped on my chest” The EKG now shows Q waves and ST segment elevations in the anterior leads. What intervention should the nurse perform? Administer prescribed morphine sulfate IV and provide oxygen at 2 L/min per nasal cannula . • 76. The nurse is developing a teaching program for the community. What population characteristic is most influential when choosing strategies for implementing a teaching plan? • 77. A client is being discharged with a prescription for warfarin (Coumadin). What instruction should the nurse provide this client regarding diet? • 78. A client who had a small bowel resection acquired methicillin resistant staphylococcus aureus (MRSA) while hospitalized. He treated and released, but is readmitted today because of diarrhea and dehydration. It is most important for the nurse to implement which intervention. • 79. A postoperative female client has a prescription for morphine sulfate 10 mg IV q3 hours for pain. One dose of morphine was administered when the client was admitted to the post anesthesia care unit (PACU) and 3 hours later, the client is again complaining of pain. Her current respiratory rate is 8 breaths/minute. What action should the nurse take? • 80. Which intervention is most important for the nurse to include in the plan of care for an older woman with osteoporosis? • 81. Based on the information provided in this client’s medical record during labor, which should the nurse implement? (Click on each chart tab for additional information. Please be sure to scroll to the bottom right corner of each tab to view all information contained in the client’s medical record.) • 82. An unlicensed assistive personnel UAP leaves the unit without notifying the staff. In what order should the unit manager implement this intervention to address the UAPs behavior? (Place the action in order from first on top to last on bottom.) 1. Note date and time of the behavior. 2. Discuss the issue privately with the UAP. 3. Plan for scheduled break times. 4. Evaluate the UAP for signs of improvement. 83. A client with intestinal obstructions has a nasogastric tube to low intermittent suction and is receiving an IV of lactated ringer’s at 100 ml/H. which finding is most important for the nurse to report to the healthcare provider? • 84. Which type of Leukocyte is involved with allergic responses and the destruction of parasitic worms? • Neutrophils • Lymphocytes • Eosinophils • Monocytes • Rationale: Eosinophils are involved in allergic responses and destruction of parasitic worms. 85. The healthcare provider prescribes the antibiotic cephradine 500mg PO every 6 hours for a client with a postoperative wound infection. Which foods should the nurse encourage this client to eat? • 86. Several months after a foot injury, and adult woman is diagnosed with neuropathic pain. The client describes the pain as severe and burning and is unable to put weight on her foot. She asks the nurse when the pain will “finally go away.” How should the nurse respond? • 87. One day following an open reduction and internal fixation of a compound fracture of the leg, a male client complains of “a tingly sensation” in his left foot. The nurse determines the client’s left pedal pulses are diminished. Based on these finding, what is the client’s greatest risk? • 88. The nurse is completing a head to be assessment for a client admitted for observation after falling out of a tree. Which finding warrants immediate intervention by the nurse? • 89. A client with multiple sclerosis (MS) has decreased motor function after taking a hot bath (Uhthoff’s sign). Which pathophysiological mechanism supports this response? • 90. While assessing a radial artery catheter, the client complains of numbness and pain distal to the insertion site. What interventions should the nurse implement? • 91. A client is admitted with an epidural hematoma that resulted from a skateboarding accident. To differentiate the vascular source of the intracranial bleeding, which finding should the nurse monitor? • 92. The nurse finds a client at 33 weeks gestation in cardiac arrest. What adaptation to cardiopulmonary resuscitation (CPR) should the nurse implement? • 93. When preparing a client for discharge from the hospital following a cystectomy and a urinary diversion to treat bladder cancer, which instruction is most important for the nurse to include in the client’s discharge teaching plan? • 94. For the past 24 hours, an antidiarrheal agent, diphenoxylate, has been administered to a bedridden, older client with infectious gastroenteritis. Which finding requires the nurse to take further action? • 95. After repositioning an immobile client, the nurse observes an area of hyperemia. To assess for blanching, what action should the nurse take? • 96. The nurse enters a client’s room and observes the client’s wrist restraint secured as seen in the picture. What action should the nurse take? • 97. A female client with acute respiratory distress syndrome (ARDS) is chemically paralyzed and sedated while she is on as assist-control ventilator using 50% FIO2. Which assessment finding warrants immediate intervention by the nurse? • 98. The development of atherosclerosis is a process of sequential events. Arrange the pathophysiological events in orders of occurrence. (Place the first event on top and the last on the bottom) 1. Arterial endothelium injury causes inflammation 2. Macrophages consume low density lipoprotein (LDL), creating foam cells 3. Foam cells release growth factors for smooth muscle cells 4. Smooth muscle grows over fatty streaks creating fibrous plaques 5. Vessel narrowing results in ischemia 99. Following a motor vehicle collision, an adult female with a ruptured spleen and a blood pressure of 70/44, had an emergency splenectomy. Twelve hours after the surgery, her urine output is 25 ml/hour for the last two hours. What pathophysiological reason supports the nurse’s decision to report this finding to the healthcare provider? • Oliguria signals tubular necrosis related to hypoperfusion 100. A nurse-manager is preparing the curricula for a class for charge nurses. A staffing formula based on what data ensures quality client care and is most cost-effective? • 101. When performing postural drainage on a client with Chronic Obstructive Pulmonary Disease (COPD), which approach should the nurse use? • 102. A client presents in the emergency room with right-sided facial asymmetry. The nurse asks the client to perform a series of movements that require use of the facial muscles. What symptoms suggest that the client has most likely experience a Bell’s palsy rather than a stroke? • 103. The nurse is teaching a client how to perform colostomy irrigations. When observing the client’s return demonstration, which action indicated that the client understood the teaching? • 104. The nurse should teach the client to observe which precaution while taking dronedarone? • 105. A client who sustained a head injury following an automobile collision is admitted to the hospital. The nurse include the client’s risk for developing increased intracranial pressure (ICP) in the plan of care. Which signs indicate to the nurse that ICP has increased? • Increased Glasgow coma scale score. • Nuchal rigidity and papilledema. • Confusion and papilledema • Periorbital ecchymosis. • Rationale: papilledema is always an indicator of increased ICP, and confusion is usually the first sign of increased ICP. Other options do not necessarily reflect increased ICP. 106. The nurse is caring for a client receiving continuous IV fluids through a single lumen central venous catheter (CVC). Based on the CVC care bundle, which action should be completed daily to reduce the risk for infection? • 107. During an annual physical examination, an older woman’s fasting blood sugar (FBS) is determined to be 140 mg/dl or 7.8 mmol/L (SI). Which additional finding obtained during a follow-up visit 2 weeks later is most indicative that the client has diabetes mellitus (DM)? • 108. A new mother tells the nurse that she is unsure if she will be able to transition into parenthood. What action should the nurse take? • 109. A client who was admitted yesterday with severe dehydration is complaining of pain a 24 gauge IV with normal saline is infusing at a rate of 150 ml/hour. Which intervention should the nurse implement first? • 110. An elderly female is admitted because of a change in her level of sensorium. During the evening shift, the client attempts to get out bed and falls, breaking her left hip. Buck’s skin traction is applied to the left leg while waiting for surgery. Which intervention is most important for the nurse to include in this client’s plan care? • 111. An Unna boot is applied to a client with a venous stasis ulcer. One week later, when the Unna boot is removed during a follow-up appointment, the nurse observes that the ulcer site contains bright red tissue. What action should the nurse take in response to this finding? • 112. At the end of a preoperative teaching session on pain management techniques, a client starts to cry and states, “I just know I can’t handle all the pain.” What is the priority nursing diagnosis for this client? • 113. The nurse note a visible prolapse of the umbilical cord after a client experiences spontaneous rupture of the membranes during labor. What intervention should the nurse implement immediately? • 114. A client who had a right hip replacement 3 day ago is pale has diminished breath sound over the left lower lung fields, a temperature of 100.2 F, and an oxygen saturation rate of 90%. The client is scheduled to be transferred to a skilled nursing facility (SNF) tomorrow for rehabilitative critical pathway. Based on the client’s symptoms, what recommendation should the nurse give the healthcare provider? • 115. A client who is newly diagnosed with type 2 diabetes mellitus (DM) receives a prescription for metformin (Glucophage) 500 mg PO twice daily. What information should the nurse include in this client’s teaching plan? (Select all that apply.) • Recognize signs and symptoms of hypoglycemia. • Report persist polyuria to the healthcare provider. • Take Glucophage with the morning and evening meal. 116. The nurse is developing an educational program for older clients who are being discharged with new antihypertensive medications. The nurse should ensure that the educational materials include which characteristics? Select all that apply • Written at a twelfth grade reading level • Contains a list with definitions of unfamiliar terms • Uses common words with few Syllables • Printed using a 12 point type font • Uses pictures to help illustrate complex ideas • Rationale: During the aging process older clients often experience sensory or cognitive changes, such as decreased visual or hearing acuity, slower thought or reasoning processes, and shorter attention span. Materials for this age group should include at least of terms, such as a medical terminology that incline may not know and use common words that expresses information clearly and simply. Simple, attractive pictures help hold the learner’s attention. The reading level of material should be at the 4th to 5th grade level. Materials should be printed using large font (18-point or higher), not the standard 12-point font. 117. During the admission assessment, the nurse auscultates heart sounds for a client with no history of cardiovascular disease. Where should the nurse listen when assessing the client’s point of maximal impulse (PMI) (Click the chosen location. To change, click on a new location) • 118. An older male adult resident of long-term care facility is hospitalized for a cardiac catheterization that occurred yesterday. Since the procedure was conducted, the client has become increasingly disoriented. The night shift nurse reports that he attempted to remove the sandbag from his femoral artery multiple times during the night. What actions should the nurse take? (Select all that apply.) • 119. An older male comes to the clinic with a family member. When the nurse attempts to take the client’s health history, he does not respond to questions in a clear manner. What action should the nurse implement first? • 120. The nurse caring for a client with acute renal fluid (ARF) has noted that the client has voided 800 ml of urine in 4 hours. Based on this assessment, what should the nurse anticipate that client will need? • 121. Which intervention should the nurse include in the plan of care for a child with tetanus? • 122. Suicide precautions are initiated for a child admitted to the mental health unit following an intentional narcotic overdose. After a visitor leaves, the nurse finds a package of cigarettes in the client’s room. Which intervention is most important for the nurse to implement? • 123. A family member of a frail elderly adult asks the nurse about eligibility requirements for hospice care. What information should the nurse provide? (Select all that apply.) • 124. A client with atrial fibrillation receives a new prescription for dabigatran. What instruction should the nurse include in this client’s teaching plan? • 125. A nurse with 10 years experience working in the emergency room is reassigned to the perinatal unit to work an 8 hour shift. Which client is best to assign to this nurse? • 126. An infant who is admitted for surgical repair of a ventricular septal defect (VSD) is irritable and diaphoretic with jugular vein distention. Which prescription should the nurse administer first? • 127. The nursing staff on a medical unit includes a registered nurse (RN), practical nurse (PN), and an unlicensed assistive personnel (UAP). Which task should the charge nurse assign to the RN? • 128. While teaching a young male adult to use an inhaler for his newly diagnosed asthma, the client stares into the distance and appears to be concentrating on something other than the lesson the nurse is presenting. What action should the nurse take? • 129. After several hours of non-productive coughing, a client presents to the emergency room complaining of chest tightness and shortness of breath. History includes end stage chronic obstructive pulmonary disease (COPD) and diabetes mellitus. While completing the pulmonary assessment, the nurse hears wheezing and poor air movement bilaterally. Which actions should the nurse implement? (Select all that apply.) • 130. The nurse caring for a 3-month-old boy one day after a pylorotomy notices that the infant is restless, is exhibiting facial grimaces, and is drawing his knees to his chest. What action should the nurse take? 131. A 4-year-old with acute lymphocytic leukemia (ALL) is receiving a chemotherapy (CT) protocol that includes methotrexate (Mexate, Trexal, MIX), an antimetabolite. Which information should the nurse provide the parents about caring for their child? • 132. Two days after admission a male client remembers that he is allergic to eggs, and informs the nurse of the allergy. Which actions should the nurse implement? (Select all that apply) • Notify the food services department of the allergy. • Enter the allergy information in the client’s record. • Add egg allergy to the client’s allergy arm band. 133. The rapid response team’s detects return of spontaneous circulation (ROSC) after 2 min of continuous chest compressions. The client has a weak, fast pulse and no respiratory effort, so the healthcare provider performs a successful oral, intubation. What action should the nurse implement? • 134. After administering an antipyretic medication. Which intervention should the nurse implement? • 135. A client with hyperthyroidism is being treated with radioactive iodine (I-131). Which explanation should be included in preparing this client for this treatment? • 136. After a colon resection for colon cancer, a male client is moaning while being transferred to the Postanesthesia Care Unit (PACU). Which intervention should the nurse implement first? • 137. The nurse is caring for a group of clients with the help of a licensed practical nurse (LPN) and an experienced unlicensed assistive personnel (UAP). Which procedures can the nurse delegate to the UAP? (Select all that apply) • Take postoperative vital signs for a client who has an epidual following knee arthroplasty • Collect a sputum specimen for a client with a fever of unknown origin • Ambulate a client who had a femoral-popliteal bypass graft yesterday 138. A male client with cirrhosis has ascites and reports feeling short of breath. The client is in semi Fowler position with his arms at his side. What action should the nurse implement? • 139. A client with a history of chronic pain requests a nonopioid analgesic. The client is alert but has difficulty describing the exact nature and location of the pain to the nurse. Which action should the nurse implement next? • 140. A client with a chronic health problem has difficulty ambulating short distance due to generalized weakness, but is able to bear weight on both legs. To assist with ambulation and provide the greatest stability, what assistive device is best for this client? • Crutches with 2 point gait. • Crutches with 3 point gait. • Crutches with 4 point gait. • A quad cane 141. The nurse uses the parkland formula (4ml x kg x total body surface area = 24 hours fluid replacement) to calculate the 24-hours IV fluid replacement for a client with 40% burns who weighs 76kg. How many ml should the client receive? (Enter numeric value only.) • 142. A client with leukemia undergoes a bone marrow biopsy. The client’s laboratory values indicate the client has thrombocytopenia. Based on this data, which nursing assessment is most important following the procedure? • 143. An 18-year-old female client is seen at the health department for treatment of condylomata acuminate (perineal warts) caused by the human papillomavirus (HPV). Which intervention should the nurse implement? • 144. A client admitted to the psychiatric unit diagnosed with major depression wants to sleep during the day, refuses to take a bath, and refuses to eat. Which nursing intervention should the nurse implement first? • 145. A client with history of bilateral adrenalectomy is admitted with a week, irregular pulse, and hypotension. Which assessment finding warrants immediate intervention by the nurse? • 146. The mother of a 7-month-old brings the infant to the clinic because the skin in the diaper area is excoriated and red, but there are no blisters or bleeding. The mother reports no evidence of watery stools. Which nursing intervention should the nurse implement? • 147. A resident of a long-term care facility, who has moderate dementia, is having difficulty eating in the dining room. The client becomes frustrated when dropping utensils on the floor and then refuses to eat. What action should the nurse implement? • 148. A client is receiving mesalamine 800 mg PO TID. Which assessment is most important for the nurse to perform to assess the effectiveness of the medication? • 149. While in the medical records department, the nurse observes several old medical records with names visible in waste container. What action should the nurse implement? • 150. A 16-year-old adolescent with meningococcal meningitis is receiving a continuous IV infusion of penicillin G, which is prescribed as 20 million units in a total volume of 2 liters of normal saline every 24 hr. The pharmacy delivers 10 million units/ liters of normal saline. How many ml/hr should the nurse program the infusion pump? (Enter numeric value only. If rounding is required, round to the nearest whole number.) • Answer 83 • 151. While visiting a female client who has heart failure (HF) and osteoarthritis, the home health nurse determines that the client is having more difficulty getting in and out of the bed than she did previously. Which action should the nurse implement first? • 152. A client has an intravenous fluid infusing in the right forearm. To determine the client’s distal pulse rate most accurately, which action should the nurse implement? • 153. A child is admitted to the pediatric unit diagnosed with sickle cell crisis. When the nurse walks into the room, the unlicensed assistive personnel (UAP) is encouraging the child to stay in bed in the supine position. Which action should the nurse implement? • 154. A preschool-aged boy is admitted to the pediatric unit following successful resuscitation from a near-drowning incident. While providing care to child, the nurse begins talking with his preadolescent brother who rescued the child from the swimming pool and initiated resuscitation. The nurse notices the older boy becomes withdrawn when asked about what happened. What action should the nurse take? • 155. After six days on a mechanical ventilator, a male client is extubated and place on 40% oxygen via face mask. He is awake and cooperative, but complaining of a severe sore throat. While sipping water to swallow a medication, the client begins coughing, as if strangled. What intervention is most important for the nurse to implement? • 156. The nurse is interacting with a female client who is diagnosed with postpartum depression. Which finding should the nurse document as an objective signs of depression? (Select all that apply) • 157. A client in the postanesthesia care unit (PACU) has an eight (8) on the Aldrete postanesthesia scoring system. What intervention should nurse implement? • 158. In caring for the body of a client who just died, which tasks can be delegate to the unlicensed assistive personnel (UAP)? (Select all that apply.) Place personal religious artifacts on the body. • 159. An adult male reports the last time he received penicillin he developed a severe maculopapular rash all over his chest. What information should the nurse provide the client about future antibiotic prescriptions? • 160. A client with a prescription for “do not resuscitate” (DNR) begins to manifest signs of impending death. After notifying the family of the client’s status, what priority action should the nurse implement? • 161. A client with cirrhosis of the liver is admitted with complications related to end stage liver disease. Which intervention should the nurse implement? (Select all that apply.) • Monitor abdominal girth. • Increase oral fluid intake to 1500 ml daily. • Report serum albumin and globulin levels. • Provide diet low in phosphorous. • Note signs of swelling and edema. • 162. During discharge teaching, the nurse discusses the parameters for weight monitoring with a client who was recently diagnosed with heart failure (HF). Which information is most important for the client to acknowledge? • 163. Which problem, noted in the client’s history, is important for the nurse to be aware of prior to administration of a newly prescribed selective serotonin reuptake inhibitor (SSRI)? • 164. When implementing a disaster intervention plan, which intervention should the nurse implement first? • Initiate the discharge of stable clients from hospital units • Identify a command center where activities are coordinated • Assess community safety needs impacted by the disaster • Instruct all essential off-duty personnel to report to the facility 165. The nurse is evaluating a client’s symptoms, and formulates the nursing diagnosis, “high risk for injury due to possible urinary tract infection.” Which symptoms indicate the need for this diagnosis? • 166. A client is admitted with metastatic carcinoma of the liver, ascites, and bilateral 4+ pitting edema of both lower extremities. When the client complains that the antiembolic stocking are too constricting, which intervention should the nurse implement? • 167. A client with muscular dystrophy is concerned about becoming totally dependent and is reluctant to call the nurse to assist with activities of daily living (ADLs). To achieve maximum mobility and independence, which intervention is most important for the nurse to include in the client’s plan of care? • 168. The nurse is teaching a postmenopausal client about osteoporosis prevention. The client reports that she smokes 2 packs of cigarettes a day and takes 750 mg calcium supplements daily. What information should the nurse include when teaching this client about osteoporosis prevention? • P 169. When evaluating a client’s rectal bleeding, which findings should the nurse document? • 170. The nurse is auscultating a client’s lung sounds. Which description should the nurse use to document this sound? • 171. An adult male is admitted to the emergency department after falling from a ladder. While waiting to have a computed tomography (CT) scan, he requests something for a severe headache. When the nurse offers him a prescribed does of acetaminophen, he asks for something stronger. Which intervention should the nurse implement? • 172. The nurse is managing the care of a client with Cushing’s syndrome. Which interventions should the nurse delegate to the unlicensed assistive personnel (UAP)? (Select all that apply) • e is admitted to a community palliative care unit. Which intervention is most important for the nurse to include in the client’s plan of care? • 174. An increased number of elderly persons are electing to undergo a new surgical procedure which cures glaucoma. What effect is the nurse likely to note as a result of this increases in glaucoma surgeries? • 175. The nurse is caring for a client who is entering the second stage of labor. Which action should the nurse implement first? • 176. To evaluate the effectiveness of male client’s new prescription for ezetimibe, which action should the clinic nurse implement? • 177. Diagnostic studies indicate that the elderly client has decreased bone density. In providing client teaching, which area of instruction is most important for the nurse to include? 178. Fall prevention measures. 179. A young adult client is admitted to the emergency room following a motor vehicle collision. The client’s head hit the dashboard. Admission assessment include: Blood pressure 85/45 mm Hg, temperature 98.6 F, pulse 124 beat/minute and respirations 22 breath/minute. Based on these data, the nurse formulates the first portion of nursing diagnosis as ” Risk of injury” What term best expresses the “related to” portion of nursing diagnosis? • Infection • Increase intracranial pressure • Shock • Head Injury. 180. An older male client with history of diabetes mellitus, chronic gout, and osteoarthritis comes to the clinic with a bag of medication bottles. Which intervention should the nurse implement first? • Identify pills in the bag. 181. A male client who was diagnosed with viral hepatitis A 4 weeks ago returns to the clinic complaining of weakness and fatigue. Which finding is most important for the nurse to report to the healthcare provider? • New onset of purple skin lesions. 182. In assessing a client twelve hours following transurethral resection of the prostate (TURP), the nurse observes that the urinary drainage tubing contains a large amount of clear pale pink urine and the continuous bladder irrigation is infusing slowly. What action should the nurse implement? • Ensure that no dependent loops are present in the tubing. 183. The healthcare provider prescribes the antibiotic Cefdinir (cephalosporin) 300mg PO every 12 h for a client with postoperative wound infections. Which feeds should the nurse encourage this client to eat? • Yogurt and/or buttermilk. • Avocados and cheese • Green leafy vegetables • Fresh fruits 184. The charge nurse is making assignment on a psychiatric unit for a practical nurse (PN) and newly license register nurse (RN). Which client should be assigned to the RN? • An adult female who has been depress for the past several month and denies suicidal ideations. A middle-age male who is in depressive phase on bipolar disease and is receiving Lithium. • A young male with schizophrenia who said voices is telling him to kill his psychiatric. • An elderly male who tell the staff and other client that he is superman and he can fly. • 185. A client at 30 week gestation is admitted due to preterm labor. A prescription of terbutaline sulfate 8.35 mg is gives subcutaneously. Based on which finding should the nurse withhold the next dose of this drug? • 186. In assessing an older female client with complication associated with chronic obstructive pulmonary disease (COPD), the nurse notices a change in the client’s appearance. Her face appears tense and she begs the nurse not to leave her alone. Her pulse rate is 100, and respirations are 26 per min. What is the primary nursing diagnosis? • 187. A client with a cervical spinal cord injury (SCI) has Crutchfield tongs and skeletal traction applied as a method of closed reduction. Which intervention is most important for the nurse to include in the client’s a plan of care? • 188. A client arrives on the surgical floor after major abdominal surgery. What intervention should the nurse perform first? • 189. A client is admitted to the emergency department with a respiratory rate of 34 breaths per minute and high pitched wheezing on inspiration and expiration, the medical diagnosis is severe exacerbation of asthma. Which assessment finding, obtained 10 min after the admission assessment, should the nurse report immediately to the emergency department healthcare provider? 190. No wheezing upon auscultation of the chest. 191. The nurse is planning a class for a group of clients with diabetes mellitus about blood glucose monitoring. In teaching the class as a whole, the nurse should emphasize the need to check glucose levels in which situation? • 192. A 350-bed acute care hospital declares an internal disaster because the emergency generators malfunctioned during a city-wide power failure. The UAPs working on a general medical unit ask the charge nurse what they should do first. What instruction should the charge nurse provide to these UAPs? • 193. The nurse is auscultating is auscultating a client’s heart sounds. Which description should the nurse use to document this sound? (Please listen to the audio file to select the option that applies.) • Murmur • s1 s2 • pericardial friction rub • s1 s2 s3 194. The healthcare provider changes a client’s medication prescription from IV to PO administration and double the dose. The nurse notes in the drug guide that the prescribed medication, when given orally, has a high first-pass effect and reduce bioavailability. What action should the nurse implement? • Administer the medication via the oral route as prescribed 195. A client refuses to ambulate, reporting abdominal discomfort and bloating caused by “too much gas buildup” the client’s abdomen is distended. Which prescribed PRN medication should the nurse administer? • Simethicone (Mylicon) 196. The public nurse health received funding to initiate primary prevention program in the community. Which program the best fits the nurse’s proposal? • Case management and screening for clients with HIV. • Regional relocation center for earthquake victims • Vitamin supplements for high-risk pregnant women. • Lead screening for children in low-income housing. • Rational: Primary prevention activities focus on health promotions and disease preventions, so vitamin for high-risk pregnant women provide adequate vitamin and mineral for fetal developmental. 197. When assessing and adult male who presents as the community health clinic with a history of hypertension, the nurse note that he has 2+ pitting edema in both ankles. He also has a history of gastroesophageal reflex disease (GERD) and depression. Which intervention is the most important for the nurse to implement? • Arrange to transport the client to the hospital • Instruct the client to keep a food journal, including portions size. • Review the client’s use of over the counter (OTC) medications. • Reinforce the importance of keeping the feet elevated. • Rationale: Sodium is used in several types of OTC medications. Including antacids, which the client may be using to treat his GERD. Further evaluation is need it to determine the need for hospitalization (A) A food journal (B) may help over, but dietary modifications are needed now since edema is present. (C) May relieve dependent edema, but not treat the underlying etiology. 198. An older client is admitted to the intensive care unit with severe abdominal pain, abdominal distention, and absent bowel sound. The client has a history of smoking 2 packs of cigarettes daily for 50 years and is currently restless and confused. Vital signs are: temperature 96`F, heart rate 122 beats/minute, respiratory rate 36 breaths/minute, mean arterial pressure(MAP) 64 mmHg and central venous pressure (CVP) 7 mmHg. Serum laboratory findings include: hemoglobin 6.5 grams/dl, platelets 6o, 000, and white blood cell count (WBC) 3,000/mm3. Based on these findings this client is at greatest risk for which pathophysiological condition? • Multiple organ dysfunction syndrome (MODS) • Disseminated intravascular coagulation (DIC) • Chronic obstructive disease. • Acquired immunodeficiency syndrome (AIDS) • Rational: MODS are a progressive dysfunction of two or more major organs that requires medical intervention to maintain homeostasis. This client has evidence of several organ systems that require intervention, such as blood pressure, hemoglobin, WBC, and respiratory rate. DIC may develop as a result of MODS. The other options are not correct. 199. A man expresses concern to the nurse about the care his mother is receiving while hospitalized. He believes that her care is not based on any ethical standards and ask what type of care he should expect from a public hospital. What action should the nurse take? • Provide the man and his mother with a copy of the Patient’s Bill of Rights 200. A client experiencing withdrawal from the benzodiazepines alprazolam (Xanax) is demonstrating severe agitation and tremors. What is the best initial nursing action? • Administer naloxone (Narcan) per PNR protocol • Initiate seizure precautions • Obtain a serum drug screen • Instruct the family about withdrawal symptoms. • Rationale: Withdrawal of CNS depressants, such as Xanax, results in rebound over-excitation of the CNS. Since the client exhibiting tremors, the nurse should anticipate seizure activity and protect the client. 201. The nurse is caring for a client who is taking a macrolide to treat a bacterial infection. Which finding should the nurse report to the healthcare provider before administering the next dose? • Jaundice • Nausea • Fever • Fatigue 202. A client with Alzheimer’s disease (AD) is receiving trazodone (Desyrel), a recently prescribed atypical antidepressant. The caregiver tells the home health nurse that the client’s mood and sleep patterns are improved, but there is no change in cognitive ability. How should the nurse respond to this information? • Explain that it may take several weeks for the medication to be effective • Confirm the desired effect of the medication has been achieved. • Notify the health care provider than a change may be needed. • Evaluate when and how the medication is being administered to the client. • Rationale: Trazodone o Desyrel, an atypical antidepressant, is prescribed for client with AD to improve mood and sleep. 203. A client with diabetic peripheral neuropathy has been taking pregabalin (Lyrica) for 4 days. Which finding indicates to the nurse that the medication is effective? • Reduced level of pain • Full volume of pedal pulses • Granulating tissue in foot ulcer • Improved visual acuity 204. A group of nurse-managers is asked to engage in a needs assessment for a piece of equipment that will be expensed to the organization’s budget. Which question is most important to consider when analyzing the cost-benefit for this piece of equipment? • How many departments can use this equipment? • Will the equipment require annual repair? • Is the cost of the equipment reasonable? • Can the equipment be updated each year? 205. While receiving a male postoperative client’s staples de nurse observe that the client’s eyes are closed and his face and hands are clenched. The client states, “I just hate having staples removed”. After acknowledgement the client’s anxiety, what action should the nurse implement? • Encourage the client to continue verbalize his anxiety • Attempt to distract the client with general conversation • Explain the procedure in detail while removing the staples • Reassure the client that this is a simple nursing procedure. • Rational: Distract is an effective strategy when a client experience anxiety during an uncomfortable procedure. (A & D) increase the client’s anxiety. 206. A male client is admitted for the removal of an internal fixation that was inserted for the fracture ankle. During the admission history, he tells the nurse he recently received vancomycin (vancomycin) for a methicillin-resistant Staphylococcus aureus (MRSA) wound infection. Which action should the nurse take? (Select all that apply.) • Collect multiple site screening culture for MRSA • Call healthcare provider for a prescription for linezolid (Zyrovix) • Place the client on contact transmission precautions • Obtain sputum specimen for culture and sensitivity • Continue to monitor for client sign of infection. • Rationale: Until multi-site screening cultures come back negative (A), the client should be maintained on contact isolation(C) to minimize the risk for nosocomial infection. Linezolid (Zyvox), a broad spectrum anti-infectant, is not indicated, unless the client has an active skin structure infection cause by MRSA or multidrug- resistant strains (MDRSP) of Staphylococcus aureus. A sputum culture is not indicated9D) based on the client’s history is a wound infection. 207. A vacuum-assistive closure (VAC) device is being use to provide wound care for a client who has stage III pressure ulcer on a below-the- knee (BKA) residual limb. Which intervention should the nurse implement to ensure maximum effectiveness of the device? • Ensure the transparent dressing has no tears that might create vacuum leaks 208. The nurse is developing the plan of care for a client with pneumonia and includes the nursing diagnosis of “Ineffective airway clearance related to thick pulmonary secretions.” Which intervention is most important for the nurse to include in the client’s plan of care? • Increase fluid intake to 3,000 ml/daily 209. The nurse plans to collect a 24- hour urine specimen for a creatinine clearance test. Which instruction should the nurse provide to the adult male client? • Clearance around the meatus, discard first portion of voiding, and collect the rest in a sterile bottle • Urinate at specific time, discard the urine, and collect all subsequent urine during the next 24 hours. • For the next 24 hours, notify the nurse when the bladder is full, and the nurse will collect catheterized specimens. • Urinate immediately into a urinal, and the lab will collect specimen every 6 hours, for the next 24 hours. • Rationale: Urinate at specific time, discard the urine, and collect all subsequent urine during the next 24 hours is the correct procedure for collecting 24-hour urine specimen. Discarding even one voided specimen invalidate the test. 210. The nurse is preparing to administer a histamine 2-receptor antagonist to a client with peptic ulcer disease. What is the primary purpose of this drug classification? • Decreases the amount of HCL secretion by the parietal cells in the stomach 211. The healthcare provider prescribes acarbose (Precose), an alpha-glucosidase inhibitor, for a client with Type 2 diabetes mellitus. Which information provides the best indicator of the drug’s effectiveness? • Hemoglobin A1C (HbA1C) reading less than 7% 212. The nurse assesses a client with new onset diarrhea. It is most important for the nurse to question the client about recent use of which type of medication? • Antibiotics • Anticoagulants • Antihypertensive • Anticholinergics 213. A neonate with a congenital heart defect (CHD) is demonstrating symptoms of heart failure (HF). Which interventions should the nurse include in the infant’s plan of care? • Give O2 at 6 L/nasal cannula for 3 repeated oximetry screens below 90% • Administer diuretics via secondary infusion in the morning only • Evaluate heart rate for effectiveness of cardio tonic medications • Use high energy formula 30 calories/ounce at Q3 hours feeding via soft nipples • Ensure Interrupted and frequent rest periods between procedures. • Rationale: Pulse oximetry screening supports prescribed level of O2. HR provides an evaluative criterion for cardiac medications, which reduce heart rate, increase strength contractions (inotropic effects) and consequently affect systemic circulation and tissue oxygenation. Breast milk or basic formula provide 20 calories/ounce, so frequent feedings with high energy formula. D minimize fatigue is necessary. 214. The nurse is caring for a 4-year-old male child who becomes unresponsive as his heart rate decreases to 40 beats/minute. His blood pressure is 88/70 mmHg, and his oxygen saturation is 70% while receiving 100% oxygen by non-rebreather face mask. In what sequence, from first to last, should the nurse implement these actions? (Place the first action on top and last action on the bottom.) 1. Start chest compressions with assisted manual ventilations 2. Administer epinephrine 0.01 mg/kg intraosseous (IO) 3. Apply pads and prepare for transthoracic pacing 4. Review the possible underlying causes for bradycardia 215. An elderly male client is admitted to the mental health unit with a sudden onset of global disorientation and is continuously conversing with his mother, who died 50 years ago. The nurse reviews the multiple prescriptions he is currently taking and assesses his urine specimen, which is cloudy, dark yellow, and has foul odor. These findings suggest that his client is experiencing which condition? • Delirium • Depression • Dementia • Psychotic episode 216. Following an esophagogastroduodenoscopy (EGD) a male client is drowsy and difficult to arouse, and his respiration are slow and shallow. Which action should the nurse implement? Select all that apply. • Prepare medication reversal agent • Check oxygen saturation level • Apply oxygen via nasal cannula • Initiate bag- valve mask ventilation. • Begin cardiopulmonary resuscitation • Rationale: Sedation, given during the procedure may need to be reverse if the client does not easily wake up. Oxygen saturation level should be asses, and oxygen applied to support respiratory effort and oxygenation. The client is still breathing so the bag- valve mask ventilation a mpressing the device to apply gentle suction in a closed surgical wound to facilitate the evacuation of subcutaneous fluids into the device. Compressing the device and securing the plug should restore function of the closed wound device. A small amount of drainage should be marked on the dressing, but replacing the dressing is not necessary and the nurse should not remove the device. Other options are not indicated. 354. A mother brings her 4-month-old son to the clinic with a quarter taped over his umbilicus, and tells the nurse the quarter is supposed to fix her child’s hernia. Which explanations should the nurse provide? • This hernia is a normal variation that resolves without treatment. • Restrictive clothing will be adequate to help the hernia go away. • An abdominal binder can be worn daily to reduce the protrusion. • The quarter should be secured with an elastic bandage wrap. • Rational: an umbilical hernia is a normal variation in infants that occurs due to an incomplete fusion of the abdominal musculature through the umbilical ring that usually resolves spontaneously as the child learns to walk. Other choices are ineffective and unnecessary. 355. A client who is admitted to the intensive care unit with syndrome of inappropriate antidiuretic hormone (SIADH) has developed osmotic demyelination. Which intervention should the nurse implement first? • Patch one eye. • Reorient often. • Range of motion. • Evaluate swallow • Rational: Osmotic demyelination, also known as central pontine myelinolysis, is nerve damage caused by the destruction of the myelin sheath covering nerve cells in the brainstem. The most common cause is a rapid, drastic change in sodium levels when a client is being treated for hyponatremia, a common occurrence in SIADH. Difficulty swallowing due to brainstem nerve damage should be care, but determining the client’s risk for aspiration is most important. 356. A client with possible acute kidney injury (AKI) is admitted to the hospital and mannitol is prescribed as a fluid challenge. Prior to carrying out this prescription, what intervention should the nurse implement? • Collect a clean catch urine specimen. • Instruct the client to empty the bladder. • Obtain vital signs and breath sounds. • No specific nursing action is required • Rational: the client’s baseline cardiovascular status should be determined before conducting the fluid challenge. If the client manifests changes in the vital signs and breath sounds associated with pulmonary edema, the administration of the fluid challenge should be terminate. Other options would not assure a safe administration of the medication. 357. A male client with COPD smokes two packs of cigarettes per day and is admitted to the hospital for a respiratory infection. He complains that he has trouble controlling respiratory distress at home when using his rescue inhaler. Which comment from the client indicates to the nurse that he is not using his inhaler properly? • “I have a hard time inhaling and holding my breath after I squeeze the inhaler, but I do my best” • “ I never use the inhaler unless I am feeling really short of breath” • I always shake the inhaler several times before I start” • “After I squeeze the inhaler and swallow, I always feel a slight wave of nausea, bit it goes away” 358. A nurse is planning to teach infant care and preventive measures for sudden infant death syndrome (SIDS) to a group of new parents. What information is most important for the nurse to include? • Ensure that the infant’s crib mattress is firm 359. A 6 -years-old who has asthma is demonstrating a prolonged expiratory phase and wheezing, and has 35% personal best peak expiratory flow rate (PEFR). Based on these finding, which action should the nurse implement first? • Administer a prescribed bronchodilator. • Report finding to the healthcare provider. • Encourage the child to cough and deep breath • Determine what trigger precipitated this attack. • Rationale: If the PEFR is below 50% in as asthmatic child, there is severe narrowing of the airway, and a bronchodilator should be administered immediately. Be should be implemented after A. C will not alleviate the symptoms and D is not a priority. 360. A client is receiving lactulose (Portalac) for signs of hepatic encephalopathy. To evaluate the client’s therapeutic response to this medication, which assessment should the nurse obtain? • Level of consciousness • Percussion of abdomen • Serum electrolytes • Blood glucose. • Rationale: Colonic bacteria digest lactulose to create a drug-induces acidic and hyperosmotic environment that draws water and blood ammonia into the colon and coverts ammonia to ammonium, which is trapped in the intestines and cannot be reabsorbed into the systemic circulation. This therapeutic action of lactulose is to reduce serum ammonia levels, which improves the client’s level of consciousness and metal status. 361. When administering an immunization in an adult client, the nurse palpates and administer the injection one inch below the acromion process into the center of the muscle mass. The nurse should document that the vaccine was administered at what site? • Rectus femenis • Ventrogluteous • Vastus lateralis • Deltoid • Rationale: The acromion process is a parameter identified for the deltoid site. 362. A primigravida a 40-weeks gestation with preeclampsia is admitted after having a seizure in the hot tub at a midwife’s birthing center. Based on documentation in the medical record, which action should the nurse implement? (Click on each chart tab for additional information. Please be sure to scroll to the bottom right corner of each tab to view all information contained in the client’s medical record.) • Continue to monitor the client’s blood pressure hourly 363. A female nurse who took drugs from the unit for personal use was temporarily released from duty. After completion of mandatory counseling, the nurse has asked administration to allow her to return to work. When the nurse administrator approaches the charge nurse with the impaired nurse request, which action is best for the charge nurse to take? • Since treatment is completed, assign the nurse to the route RN responsibilities • Ask to meet with impaired nurse’s therapist before allowing her back on the unit. • Allow the impaired nurse to return to work and monitor medication administration • Meet with staff to assess their feelings about the impaired nurse’s return to the unit. • Rationale: provides essential monitoring and helps ensure nurse compliance and promote client safety. 364. In making client care assignment, which client is best to assign to the practical nurse (PN) working on the unit with the nurse? • An immobile client receiving low molecular weight heparin q12 h. • A client who is receiving a continuous infusion of heparin and gets out of bed BID • A client who is being titrated off heparin infusion and started on PO warfarin (Coumadin) • An ambulatory client receiving warfarin (Coumadin) with INR of 5 second. • Rationale: A describe the most stable client. The other ones are at high risk for bleeding problems and require the assessment skills. 365. A client who is admitted to the intensive care unit with a right chest tube attached to a THORA-SEAL chest drainage unit becomes increasingly anxious and complain of difficulty breathing. The nurse determine the client is tachypneic with absent breath sounds in the client’s right lungs fields. Which additional finding indicates that the client has developed a tension pneumothorax? • Continuous bubbling in the water seal chamber • Decrease bright red blood drainage • Tachypnea and difficulty breathing • Tracheal deviation toward the left lung. • Rationale: Tracheal deviation toward the unaffected left lung with absent breath sounds over the affected right lung are classic late signs of a tension pneumothorax. 366. A low-risk primigravida at 28-weeks gestation arrives for her regular antepartal clinic visit. Which assessment finding should the nurse consider within normal limits for this client? • Pulse increase of 10 beats/minute • Proteinuria • Glucosuria • Fundal height 0f 22 centimeters 367. The nurse discovers that an elderly client with no history of cardiac or renal disease has an elevated serum magnesium level. To further investigate the cause of this electrolyte imbalance, what information is most important for the nurse to obtain from the client’s medical history? • Frequency of laxative use for chronic constipation 368. Which action should the nurse implement with auscultating anterior breath sounds? (Place the first action on top and last action on the bottom) • Correct order: (PADD) 1. Place stethoscope in suprasternal area to auscultate for bronchial sounds 2. Auscultate bronchovesicular sounds from side to side the first and second intercostal spaces 3. Displace female breast tissue and apply stethoscope directly on chest wall to hear vesicular sounds 4. Document normal breath sounds and location of adventitious breath sounds 369. A client with chronic alcoholism is admitted with a decreased serum magnesium level. Which snack option should the nurse recommend to this client? • Cheddar cheese and crackers. • Carrot and celery sticks. • Beef bologna sausage slices. • Dry roasted almonds. • Rational: alcoholism promotes inadequate food intake and gastrointestinal loss of magnesium include green leafy vegetables and nuts and seeds. Other snacks listed provide much lower amounts of magnesium per serving. 370. The nurse is preparing a teaching plan for an older female client diagnosed with osteoporosis. What expected outcome has the highest priority for this client? • Identifies 2 treatments for constipation due to immobility. • Names 3 home safety hazards to be resolve immediately. • State 4 risk factors for the development of osteoporosis. • Lists 5 calcium-rich foods to be added to her daily diet. • Rational: a major teaching goal for an elderly client with osteoporosis is maintenance of safety to prevent falls. Injury due to a fall, usually resulting in a hip fracture, can result in reduced mobility and associated complications. Other goals are also important when teaching clients who have osteoporosis, but they do not have the priority of preventing falls, which relates to safety. 371. The nurse is teaching a male adolescent recently diagnosed with type 1diabetes mellitus (DM) about self-injecting insulin. Which approach is best for the nurse to use to evaluate do you effectiveness of the teaching? • Ask the adolescent to describe his level of comfort with injecting himself with insulin. • Observe him as he demonstrates self-injection technique in another diabetic adolescent • Have the adolescent list the procedural steps for safe insulin administration. • Review his glycosylated hemoglobin level 3 months after the teaching session. • Rational: watching the adolescent perform the procedure with another adolescent provides peer support the most information regarding his skill with self-injection. Other options do not provide information about the effectiveness of nurse’s teaching. 372. A young adult woman visits the clinic and learns that she is positive for BRCA1 gene mutation and asks the nurse what to expect next. How should the nurse respond? • Explain that counseling will be provided to give her information about her cancer risk • Gather additional information about the client’s family history for all types of cancer. • Offer assurance that there are a variety of effective treatments for breast cancer. • Provide information about survival rates for women who have this genetic mutation. • Rational: BRACA1or BRACA2 genetic mutation indicates an increased risk for developing breast or ovarian cancer and genetic counseling should be provided to explain the increased risk (A)to the client along with options for increased screening or preventative measures. (B) Is completed by the genetic counselor before the client undergoes genetic testing. a positive BRACA1test is not an indicator of the presence of cancer and (C and D) are not appropriate responses prior to genetic counseling. 373. A mother runs into the emergency department with s toddler in her arms and tells the nurse that her child got into some cleaning products. The child smells of chemicals on hands, face, and on the front of the child's clothes. After ensuring the airway is patent, what action should the nurse implement first? • Call poison control emergency number. • Determine type of chemical exposure. • Obtain equipment for gastric lavage. • Assess child for altered sensorium. • Rational: once the type of chemical is determined, poison control should be called even if the chemical is unknown. If lavage is recommended by poison control, intubation and nasogastric tube may be needed as directed by poison control. Altered sensorium, such as lethargy, may occur if hydrocarbons are ingested 374. The nurse assigned unlicensed assistive personnel (UAP) to apply antiembolism stockings to a client. The nurse and UAP enters the room, the nurse observes the stockings that were applying by the UAP. The UAP states that the client requested application of the stockings as seen on the picture, for increased comfort. What action should the nurse take? • Ask the client if the stocking feel comfortable. • Supervise the UAP in the removal of the stockings. • Place a cover over the client’s toes to keep them warm. • Discussed effective use of the stockings with the client on UAP • Rational: antiembolism stockings are designed to fit securely and should be applied so that there are no bands of the fabric constricting venous return. The nurse should discuss the need for correct and effective use of the stockings with both the client and UAP to improve compliance. Other options do not correct the incorrect application of the stockings. 375. Nurses working on a surgical unit are concerned about the physicians treatment of clients during invasive procedures, such as dressing changes and insertion of IV lines. Clients are often crying during the procedures, and the physician is usually unconcerned or annoyed by the client’s response. To resolve this problem, what actions should the nurses take? (Arrange from the first action on the top of the list on the bottom) 1. Talk to the physician as a group in a non-confrontational manner. 2. Document concerns and report them to the charge nurse. 3. Submit a written report to the director of nursing. 4. Contact the hospital’s chief of medical services. 5. File a formal complaint with the state medical board. • Rational: nurses have both an ethical and legal responsibility to advocate for clients’ physical and emotional safety. Talking with the physician in a nonconfrontational manner is the first step in conflict resolution. If this is not effective, the organizational chain of ineffective, a formal complaint with the state medical board should be implemented. 376. While changing a client’s chest tube dressing, the nurse notes a crackling sensation when gentle pressure is applied to the skin at the insertion site. What is the best action for the nurse to take? • Apply a pressure dressing around the chest tube insertion site. • Assess the client for allergies to topical cleaning agents. • Measure the area of swelling and crackling. • Administer an oral antihistamine per PRN protocol. • Rational: a crackling sensation, or crepitus, indicates subcutaneous emphysema, or air leaking into the skin. This area should be measured and the finding documented. Other options are not indicated for crepitus. 377. To prevent infection by auto contamination during the acute phase of recovery from multiple burns, which intervention is most important for the nurse to implement? • Dress each wound separately. • Avoid sharing equipment between multiple clients. • Use gown, mask and gloves with dressing change. • Implement protective isolation. • Rational: each wound should be dressed separately using a new pair of sterile glove to avoid auto contamination (the transfer of microorganisms form one infected wound to a non-infected wound). The other choices do not prevent auto contamination. 378. The nurse is preparing an intravenous (IV) fluid infusion using an IV pump. Within 30 seconds of turning on the machine, the pump’s alarm beeps “occlusion”. What action should the nurse implement first? • Flush the vein with 3 ml of sterile normal saline. • Assess the IV catheter insertion site for infiltration. • Verify the threading of the tubing through the IV pump. • Determine if the clamp on the IV tubing is released • Rational: When the pump immediately beeps, it is often because the IV tubing clamp is occluding the flow, so the clamp should be checked first to ensure that it is open. If the alarm is not eliminated after the tubing clamp is released, flushing the IV site with saline is a common practice to clean the needle or to identify resistance due to another source. Local signs of infiltration may indicate the need to select another vein, but the pump’s beeping-this early in the procedure is likely due to a mechanical problem. If beeping continues after verifying that the clamp is released the placement or threading of the tubing through the pump should be verified. 379. A client with arthritis has been receiving treatment with naproxen and now reports ongoing stomach pain, increasing weakness, and fatigue. Which laboratory test should the nurse monitor? • Sed rate (ESR) • Hemoglobin • Calcium • Osmolality. • Rational: naproxen can cause gastric bleeding, so the nurse should monitor the client’s hemoglobin to assess for possible bleeding. Other options are not likely to be affected by the used of naproxen and are not related to the client’s current symptoms. 380. The nurse assesses a child in 90-90 traction. Where should did nurse assess for signs of compartment syndrome? • Rationale: compartment syndrome is the result of swelling and subsequent reduction in circulation to the area distal to the compartment. This can be a complication of traumatic injury and cast administration, so it is important to assess circulation distal to the casted prolonged capillary refill. 381. After receiving the Braden scale findings of residents at a long-term facility, the charge nurse should to tell the unlicensed assistive personnel (UAP) to prioritize the skin care for which client? • An older adult who is unable to communicate elimination needs. • An older man whose sheets are damped each time he is turned. • A woman with osteoporosis who is unable to bear weight. • A poorly nourished client who requires liquid supplement. • Rational: a Braden score of less than 18 indicates a risk for skin breakdown, and clients with such score require intensive nursing care. Constant moisture places the client at a high risk for skin breakdown, and interventions should be implemented to pull moisture away from the client’s skin. Other options may be risk factors but do not have as high a risk as constant exposure to moisture. 382. A client with acute renal failure (ARF) is admitted for uncontrolled type 1 diabetes Mellitus and hyperkalemia. The nurse administers an IV dose of regular insulin per sliding scale. Which intervention is the most important for the nurse to include in this client’s plan of care? • Monitor the client’s cardiac activity via telemetry. • Maintain venous access with an infusion of normal saline. • Assess glucose via fingerstick q4 to 6 hours. • Evaluate hourly urine output for return of normal renal function. • Rational: as insulin lowers the blood glucose of a client with diabetic ketoacidosis (DKA), potassium returns to the cell but may not impact hyperkalemia related to acute renal failure. The priority is to monitor the client for cardiac dysrhythmias related to abnormal serum potassium levels. IV access, assessment of glucose level, and monitoring urine output are important interventions, but do not have the priority of monitoring cardiac function. 383. A client with C-6 spinal cord injury rehabilitation. In the middle of the night the client reports a severe, pounding headache, and has observable piloerection or “goosebumps”. The nurse should asses for which trigger? • Loud hallway noise. • Fever • Full bladder • Frequent cough. • Rational: a pounding headache is a sign of autonomic hyperreflexia, an acute emergency that occurs because of an exaggerated sympathetic response in a client with a high level spinal cord injury. Any stimulus below the level of injury can trigger autonomic hyperreflexia, but the most common cause is an overly distended bladder. The other options are unlikely to produce the manifestation of autonomic hyperreflexia. 384. A nurse working on an endocrine unit should see which client first? • An adolescent male with diabetes who is arguing about his insulin dose. • An older client with Addison’s disease whose current blood sugar level is 62mg/dl (3.44 mmol/l). • An adult with a blood sugar of 384mg/dl (21.31mmol/l) and urine output of 350 ml in the last hour. • A client taking corticosteroids who has become disoriented in the last two hours. • Rational: meeting the client’s need for safety is a priority intervention. Mania and psychosis can occur during corticosteroids therapy, places the client at risk for injury, so the patient taking corticosteroids should be seen first. 385. A client is receiving and oral antibiotic suspension labeled 250 mg/2ml. The healthcare provider prescribes 200mg every 6 hours. How many ml should the nurse administer at each dose? (Enter numerical value only. If rounding is required, round to the nearest tenth) • Answer: 1.6 • Rational: using the formula D/H x Q • 200mg/250 mg x 2ml = 200/250 = 1.6 ml 386. Four hours after surgery, a client reports nausea and begins to vomit. The nurse notes that the client has a scopolamine transdermal patch applied behind the ear. What action should the nurse take? • Reposition the transdermal patch to the client’s trunk. • Remove the transdermal patch until the vomiting subsides. • Notify the healthcare provider of the vomiting. • Explain that this is a side effect of the medication in the patch. • Rational: transdermal scopolamine is used to prevent nausea and vomiting from anesthesia and surgery. The nurse should notify the healthcare provider if the medication is ineffective. The patch should be applied behind the ear and should remain in place to reduce the nausea and vomiting. Nausea and vomiting are no side effects of the medication. 387. The nurse identifies an electrolyte imbalance, an elevated pulse rate, and elevated BP for a client with chronic kidney disease. Which is the most important action for the nurse to take? • Monitor daily sodium intake. • Record usual eating patterns. • Measure ankle circumference. • Auscultate for irregular heart rate. • Rational: Chronic kidney failure (CKF) is a progressive, irreversible loss of kidney functions, decreasing glomerular filtration rate (GFR), and the kidney’s inability to excrete metabolic waste products and water, resulting in fluid overload, elevated pulse, elevated BP and electrolytes imbalances. The most important action for the nurse to implement is to auscultate for irregular heart rate (D) due to the decreased excretion of potassium by the kidneys. (A, B, and C) are not as important as monitoring for fatal cardiac dysrhythmias related to hyperkalemia. 388. A client with persistent low back pain has received a prescription for electronic stimulator (TENS) unit. After the nurse applies the electrodes and turns on the power, the client reports feeling a tingling sensation. How should the nurse respond? • Determine if the sensation feels uncomfortable. • Decrease the strength of the electrical signals. • Remove electrodes and observe for skin redness. • Check the amount of gel coating on the electrodes. • Rational: electronic stimulators, such as a transelectrical nerve stimulator (TENS) unit, have been found to be effective in reducing low back pain by “closing the gate” to pain stimuli. A tingling sensation should be felt when the power is turned on, and the nurse should assess whether the sensation is too strong, causing discomfort or muscle twitching. Decreasing the electrical signal may be indicated if the sensation is too strong. Other options are not necessary because the tingling sensation is expected. 389. A female client is extremely anxious after being informed that her mammogram was abnormal and needs to be repeated. Client is tearful and tells the nurse her mother died of breast cancer. What action should the nurse take? • Provide the client with information about treatment options for breast cancer. • Reassure the client that the final diagnosis has not been made. • Encourage the client to continue expressing her fears and concerns. • Suggest to the client that she seek a second opinion. • Rational: the nurse should show support for the client by encouraging her to continue expressing her concerns. A diagnosis has not yet been made, so it is too early to discuss treatment options. Other options dismiss the client’s feelings or are premature given that the diagnosis is not yet made. 390. The psychiatric nurse is talking to a newly admitted client when a male client diagnosed with antisocial behavior intrudes on the conversation and tells the nurse, “I have to talk to you right now! It is very important!” how should the nurse respond to this client? • Put his behavior on extinction and continue talking with the newly admitted. • Inform him that the nurse is busy admitting a new client and will talk to him later. • Encourage him to go to the nurse’s station and talk with another nurse. • Introduce him to the newly admitted client and ask him to him to join in the conversation. • Rational: the psychiatric nurse must set limits with antisocial behavior so that appropriate behavior is demonstrated. Interrupting a conversation is rude and inappropriate, so telling the client that they can talk later is the best course of action. Other options may cause the client to become angry and they do not address the client’s behavior. The nurse should not involve this client with newly admitted client’s admission procedure. 391. The charge nurse is planning for the shift and has a registered nurse (RN) and a practical nurse (PN) on the team. Which client should the charge nurse assign to the RN? • A 64 year old client who had a total hip replacement the previous day. • A 75 year old client with renal calculi who requires urine straining. • An adolescent with multiple contusions due to a fall that occurred 2 days ago. • A 30 year old depressed client who admits to suicide ideation. • RATIONALE: A client who is suicidal requires psychological assessment, therapeutic communication and knowledge beyond the educational level of a practical nurse (RN). Other clients could be cared for by the PN or the UAP, with supervision by the registered nurse. 392. A female client presents in the Emergency Department and tells the nurse that she was raped last night. Which question is most important for the nurse to ask? • Does she knows the person who raped her? • Has she taken a bath since the raped occurred? • Is the place where she lived a safe place? • Did she report the rape to the police Department? • RATIONALE: The priority action is collected the forensic evidence, so asking if the has taken a bath since the rape occurred is the most important information to obtain. Other options are used by law enforcement to determine the perpetrator and are not vital in providing client care at this time. 393. While caring for a client’s postoperative dressing, the nurse observes purulent drainage at the wound. Before reporting this finding to the healthcare provider, the nurse should review which of the client’s laboratory values? • Serum albumin • Creatinine level • Culture for sensitive organisms. • Serum blood glucose (BG) level • RATIONALE: A client who has a postoperative dressing with purulent drainage from the wound is experiencing an infection. The nurse should review the client’s laboratory culture for sensitive organisms (C) before reporting to the healthcare provider. (A, B and D) are not indicated at this time. 394. The nurse is demonstrating correct transfer procedures to the unlicensed assisted personnel (UAP) working on a rehabilitation unit. The UAPs ask the nurse how to safely move a physically disabled client from the wheelchair to a bed. What action should the nurse recommended? • Hold the client at arm’s length while transferring to better distribute the body weight. • Apply the gait belt around the client’s waits once standing position has been assumed. • Place a client’s locked wheelchair on the client’s strong side next to the bed. • Pull the client into position by reaching from the opposite side of the bed. • RATIONALE: Placing the wheelchair on the client’s strong side offers the greatest stability for the transfer. Holding the client arm’s length or pulling from the opposite site of the bed reflect poor body mechanism. Using a gait belt offers additional safety for the client, but should be done after the wheelchair has be put into the proper place and the wheels have been locked and before the client has assumed a standing position. 395. A client who is experiencing musculoskeletal pain receives a prescription for ketorolac 15mg IM q6 hours. The medication is depended in a 39mg/ml pre-filled syringe. Which action should the nurse implement when giving the medication? • Administer the entire pre-filled syringe deep in the dorsogluteal site. • Use a separate syringe to remove 15mg from the pre-filled syringe and give in the back of the arm. • Waste 0.5 ml from the pre-filled syringe and inject the medication in the ventrogluteal site. • Call the healthcare provider to request a prescription change to match the dispensed 30mg dose. • RATIONALE: The pre-filled contain 30mg /1ml, so 0.5ml should be wasted to obtain the correct dosage of 15mg for administration in the preferred IM ventrogluteal site. The nurse is responsible for calculating and preparing the prescribed dose using the available concentration, so other options are not indicated. 396. A client with a lower respiratory tract infection receives a prescription for ciprofloxacin 500mg PO q 12hours. When the client request an afternoon snack, which dietary choice should the nurse provide? • Vanilla-flavored yogurt • Low fat chocolate milk. • Calcium fortified juice • Cinnamon applesauce • RATIONALE: Dairy products and calcium fortified dairy products decrease the absorption of ciprofloxacin. Cinnamon applesauce contains no calcium, so this is the best snack selection. Since other options contains calcium, these snack should be avoided by a client who is taking ciprofloxacin. 397. The healthcare provider prescribes a low-fiber diet for a client with ulcerative colitis. Which food selection would indicate to the nurse the client understands they prescribed diet? • Roasted turkey canned vegetables • Baked potatoes with skin raw carrots • Pancakes whole-grain cereal's • Roast pork fresh strawberries • Rationale: Foods allowed on a low-fiber diet includes roasted or baked turkey and canned vegetables the foods in the other options are not low in fiber 398. An adult client with schizophrenia begin treatment three days ago with the Antipsychotic risperidone. The client also received prescription for trazodone as needed for sleep and clonazepam as needed for severe anxiety. When the client reports difficulty with swallowing, what action should the nurse take? • Obtain a prescription for an anticholinergic medication • Determine how many hours declined slept last night • Administer the PRN prescription for severe anxiety • Watch the thyroid cartilage move while the client swallows • Rationale: Antipsychotic medications have an extrapyramidal side effects one of which is difficult to swallowing the nurse should obtain a prescription for an anticholinergic medication which is used for the treatment of extrapyramidal symptoms. Other options are not warranted actions based on the symptoms presented. 399. One year after being discharged from the burn trauma unit, a client with a history of 40% full-thickness burns is admitted with bone pain and muscle weakness. Which intervention should the nurse include in the clients plan of care? • Encourage Progressive active range of motion • Teach need for dietary and supplementary vitamin D3 • Explain the need for skin exposure to sunlight without sunscreen • Instruct the client to use of muscle strengthening exercises • Rationale: Burn injury results in the acute loss of bone as well as the development of progressive vitamin D deficiency because burn scar tissue and adjacent normalappearing skin cannot convert normal quantities of the precursors for vitamin D3 that is synthesized from ultraviolet sun rays which is needed for strong bones. Clients with a history of full thickness burns should increase their dietary resources of vitamin D and supplemental D3 (B). range of motion (A) and muscle strengthening exercises (D) do not treat he underlying causes of the bone pain and weakness unprotected sunlight (C) should be avoided. 400. When teaching a group of school-age children how to reduce the risk of Lyme disease which instruction should the camp nurse include? • Wash hands frequently • Avoid drinking lake water • Wear long sleeves and pants • Do not share personal products • Rationale: Lyme disease is it tick bone disorder and is transmitted to a child via a tick bite. Keeping the skin covered reduces the risk of being bitten by a tick. Other options are not reduce the risk for tick bites. 401. A native-American male client diagnosed with pneumonia, states that in addition to his prescribed medical treatment of IV antibiotics he wishes to have a spiritual cleaning performed. Which outcome statement indicates that the best plan of care was followed? • Identifies his ethnocentric values and behaviors • States an understanding of the medical treatment • Participated actively in all treatments regimens • Expresses a desire for cultural assimilation • Rationale: indicates active participation by the client, which is required for treatment to be successful. The best plan of care should incorporate the valued and treatments of both cultures and in this case there is no apparent cultural clash between the two forms of treatment. The client has already identify he's cultural values (A). (B) Only considers one of the two treatment modalities desired by the client the client has already chosen how he wishes to assimilate his cultural values with the prescribed medical treatment (D). 402. A male client with cancer is admired to the oncology unit and tells the nurse that he is in the hospital for palliative care measures. The nurse notes that the client’s admission prescription include radiation therapy. What action should the nurse implement? • Ask the client about his expected goals for the hospitalization • Explain the palliative care measures can be provided at home • Notify do radiation department to withhold the treatment for now • Determine if the client wishes to cancel further radiation treatment • Rationale: Palliative care measures provide relief or control of symptoms, so it is important for the nurse to determine the client’s goals for symptom control while receiving treatment in the hospital. Although home care is available the client may not be legible for palliative care at home. Radiation therapy is an effective positive care measure used to manage symptoms and would be appropriate unless the radiation conflicts with the client goals. 403. A client with myasthenia Gravis (MG) is receiving immunosuppressive therapy. Review recent laboratory test results show that the client’s serum magnesium level has decreased below the normal range. In addition to contacting the healthcare provider, what nursing action is most important? • Check the visual difficulties • Note most recent hemoglobin level • Assessed for he and Hand joint pain • Observe rhythm on telemetry monitor • Rationale: If not treated a low little Serum magnesium level can affect myocardial depolarization leading to a lethal arrhythmia, and the nurse should assess for dysrhythmias before contacting the healthcare provider. Other choices are common in MG but do not contribute the Safety risk of low magnesium levels. 404. A young adult female presents at the emergency center with acute lower abdominal pain. Which assessment finding is most important for the nurse to report to the healthcare provider? • Pain scale rating at 9 on a 0-10 scale • Last menstrual period was 7 weeks ago • Reports white curdy vaginal discharge • History of irritable bowel syndrome IBS • Rationale: Acute lower abdominal pain in A young adult female can be indicative of an ectopic pregnancy, which can be life threatening. Since the clients last menstrual period was seven weeks ago a pregnancy test to be obtained to ruled out ectopic pregnancy, which can result in intra-abdominal hemorrhage caused by a ruptured Fallopian tube. Although the severity of pain requires treatment, the most significant finding is the clients last menstrual period. Other options are not the most important concerns. 405. A 154 pound client with diabetic ketoacidosis is receiving an IV of normal saline 100 ML with regular insulin 100 units. The healthcare provider prescribes a rate of 0.1 units/kg/hour. To deliver the correct dosage, the nurse should set the infusion pump to Infuse how many ml/hour? enter numeric value only • 7 • Rationale: Convert the client’s weight to kg, 2.2 pound: 1 kg:: 154 pounds: x kg = 154/2.2 = 70kg. Calculate the client infusion rate, 0.1 x 70 kg = 7 units/hour. Using the formula, D/H x Q = 7 units/hour / 100 units x 100 ml = 7ml / hour 406. The nurse is assessing a postpartum client who is 36 hours post-delivery. Which finding should the nurse report to the healthcare provider? • White blood count of 19,000 mm3 • Oral temperature of 100.6 F • Fundus deviated to the right side • Breasts are firm when palpated • Rationale: A temperature greater than 100.4 F (38 C) (B), which is indicative of endometriosis (infection of the lining of the uterus), should be reported to the health care provider. (A and D) are findings that are within normal limits in the postpartum period. Fundal deviation to one side (C) is an expected finding related to a full bladder, so the nurse should encourage the client to void. 407. A nurse who is working in the emergency department triage area is presented with four clients at the same time. The client presented with which symptoms requires the most immediate intervention by the nurse? • Low-grade fever, headache, and malaise for the past 72 hours • Unable to bear weight on the left foot, with the swelling and bruising • Chest discomfort one hour after consuming a large, spicy meal • One-inch bleeding laceration on the chain of the crying five-year-old • Rationale: Emergency triage involves quick assessment to prioritize the need for further evaluation and care. Those with trauma, chest pain, respiratory distress, or acute neurological changes are priority. In this example, while clients with other conditions require attention, the client with chest discomfort is at greatest risk and is a priority. 408. The nurse is planning to assess a client's oxygen saturation to determine if additional oxygen is needed via nasal cannula. The client has a bilateral below-the-knee amputation and pedal pulses that are weak and threaty. What action should the nurse take? • Document that an accurate oxygen saturation reading cannot be obtained • Elevate to client's hands for five minutes prior to obtaining a reading from the finger • Increase the oxygen based on the clients breathing patterns and lung sounds • Place the oximeter clip on the ear lobe to obtain the oxygen saturation reading • Rationale: Pulse oximeter clips can be attached to the earlobe to obtain an accurate measurement of oxygen saturation. Other options will not provide the needed assessment. 409. A young adult male who is being seen at the employee health care clinic for an annual assessment tell the nurse that his mother was diagnosed with schizophrenia when she was his age and that life with a schizophrenic mother was difficulty indeed. Which response is best for the nurse to provide? • Ask the client if he is worried about becoming schizophrenic at the age his mother was diagnosed. • Encourage the client to seek genetic counseling to determine his risk for mental illness. • Informed the client that his mother schizophrenic has affected his psychological development. • Tell the client that mental illness has a familial predisposition so he should see a psychiatrist. 410. A client on a long-term mental health unit repeatedly takes own pulse regardless of the circumstance. What action should the nurse implement? • Overlook the client’s behavior. • Distract client to interfere with the ritual. • Ask why the client checks the pulse. • Hold client’s hand to stop the behavior. 411. A client is discharged with automated peritoneal dialysis (PD) to be used nightly…which instructions should the nurse include? • Wash hands before cleaning exit site • Keep the head of the bed flat at night • Feel for a thrill and a distal pulse nightly • Do not get up if fluid is left in the abdomen 412. The charge nurse observes the practical nurse (PN) apply sterile gloves in preparation for performing a sterile dressing change. Which action by the PN requires correction by the charge nurse? • Opening the package • Picking up the second glove • Picking up the first glove • Positioning of the table 413. A male client reports to the clinic nurse that he has been feeling well and is often “dizzy” his blood pressure is elevated. Based on this findings, this client is at a greatest risk for which pathophysiological condition? • Stroke • Renal failure • Left ventricular hypertrophy • Pulmonary hypertension 414. The nurse ask the parent to stay during the examination of a male toddler’s genital area. Which intervention should the nurse implement? • Examine the genitalia as the last part of the total exam. • Use soothing statements to facilitate cooperation • Allow the child to keep underpants on to examine genitalia • Work slowly and methodically so not to stress the child 415. The nurse is changing a client’s IV tubing and closes the roller clamp on the new tubing setup when the bag of solution is….which action should the nurse take to ensure adequate filling of the drip chamber? • Lower the IV bag to a flat surface • Compress the drip chamber • Open the roller clamp • Squeeze the bag of IV solution 416. …An Insulin infusion for a client with diabetes mellitus who is experiencing hyperglycemic hyperosmolar…in addition to the client’s glucose, which laboratory value is most important for the nurse to monitor? • Serum potassium • Urine ketones • Urine albumin • Serum protein 417. A young adult who is hit with a baseball bat on the temporal area of the left skull is conscious when admitted to the ED and is transferred to the Neurological Unit to be monitored for signs of closed head injury. Which assessment finding is indicative of a developing epidural hematoma? • Altered consciousness within the first 24 hours after injury. • Cushing reflex and cerebral edema after 24 hours • Fever, nuchal rigidity and opisthotonos within hours • Headache and pupillary changes 48 hours after a head injury 418. In planning strategies to reduce a client's risk for complications following orthopedic surgery, the nurse recognizes which pathology as the underlying cause of osteomyelitis? • infectious process • metastatic process • autoimmune disorder • inflammatory disorder 419. A client with bipolar disorder began taking valproic acid (Depakote) 250 mg PO three times daily two months ago. Which finding provides the best indication that the medication regimen is effective? • The family reports a great reduction in client’s maniac behavior 420. Which intervention should the nurse implement during the administration of vesicant chemotherapeutic agent via an IV site in the client’s arm? • Assess IV site frequently for signs of extravasation 421. A client with a serum sodium level of 125 meq/mL should benefit most from the administration of which intravenous solution? • 0.9% sodium chloride solution (normal saline) • 0.45% sodium chloride solution (half normal saline) • 10% Dextrose in 0.45% sodium chloride • 5% dextrose in 0.2% sodium chloride 422. A client with Alzheimer’s disease falls in the bathroom. The nurse notifies the charge nurse and completes a fall follow-up assessment. What assessment finding warrants immediate intervention by the nurse? • Urinary incontinence • Left forearm hematoma • Disorientation to surroundings • Dislodge intravenous site • Rationale: The left forearm hematoma may be indicative an injury, such as broken bone, that requires immediate intervention. A may be likely be due to the inability to use the toilet due to the fall. Disorientation is a common symptom of Alzheimer’s disease. IV Dislodged is not an urgent concern. 423. The nurse is triaging clients in an urgent care clinic. The client with which symptoms should be referred to the health care provider immediately? • headache, photophobia, and nuchal rigidity • high fever, skin rash, and a productive cough • nausea, vomiting, and poor skin turgor • malaise, fever, and stiff, swollen joints • Rationale: Headache, photophobia, and nuchal rigidity are classic signs of meningeal infection, so this client should immediately be referred to the health care provider. AC D do not have priority of B 424. An adult male is brought to the emergency department by ambulance following a motorcycle accident. He was not wearing a helmet and presents with periorbital bruising and bloody drainage from both ears. Which assessment finding warrants immediate intervention by the nurse? • Rebound abdominal tenderness • nausea and projectile vomit • rib pain with deep inspiration • diminished bilateral breath sounds • Rationale: Projective vomiting is indicative of increasing intracranial pressure, which can lead to ischemic brain damage or death, so this finding warrants immediate intervention. Rebound abdominal tenderness may indicate internal bleeding. Diminished breath sound may be related to pain. Rib pain with inspiration may indicate rib fracture. 425. After placement of a left subclavian central venous catheter (CVC), the nurse receives report of the x-ray findings that indicate the CVC tip is in the client’s superior vena cava. Which action should the nurse implement? • Initiate intravenous fluid as prescribed • Notify the HCP of the need to reposition the catheter • Remove the catheter and apply direct pressure for 5 minute • Secure the catheter using aseptic technique • Rationale: Venous blood return to the heart and drains from the subclavian vein into the superior vena cava. The X-ray findings indicate proper placement of the CVC, so prescribed intravenous fluid can be started. A and B are not indicated at this time. The catheter should be secure immediate following insertion (C) 426. The nurse has received funding to design a health promotion project for AfricanAmerican women who are at risk for developing breast cancer. Which resource is most important in designing this program? • A listing of African-American women so live in the community • Participation of community leaders in planning the program • Morbidity data for breast cancer in women of all races • Technical assistance to produce a video on breast self-examination. • Rationale: When developing a culturally-competent health promotion project, the participation of stakeholders and community leaders is most important. A and B might be useful background information, but t=first the program should be developed. D may be useful fulfilling the plan developed by the health care team and the community leaders if funding for this assistance is included in the budget. 427. The home care nurse provide self-care instruction for a client chronic venous insufficiency cause by deep vein thrombosis. Which instructions should the nurse include in the client’s discharge teaching plan? Select all that apply • Avoid prolonged standing or sitting • Use recliner for long period of sitting • continue wearing elastic stocking • Maintain the bed flat while sleeping • Cross legs at knee but not at ankle 428. The nurse is interviewing a client with schizophrenia. Which client behavior requires immediate intervention? • Lip smacking and frequent eye blinking • Shuffling gait and stooped posture • Rocks back and forth in the chair • Muscle spasms of the back and neck • Rationale: An extra pyramidal symptom (EPS) characterized by abnormal muscle spasms of the neck (A) requires immediate intervention because it can cause difficulty swallowing and jeopardize the airway. Though (A, B and C) are also EPS caused by antipsychotic medication medications used to manage schizophrenia (D) has the highest priority to insure client safety is (A) 429. A male client was transferred yesterday from the emergency department to the telemetry unit because he had ST depression and resolved chest pain. When his EKG monitor alarms for ventricular tachycardia (VT), what action should the nurse take first? • Determine the client’s responsiveness and respirations • Bring the crash cart to the room to defibrillate the client. • Immediately initiate chest compressions. • Notify the emergency response team • Rationale: Activities, such as brushing teeth, can mimic the waveform of VI, so first he client should be assessed (A) to determine if the alarm is accurate. The crash cart can be brought to the room by someone else and defibrillation (B) delivered as indicated by the client’s rhythm. Based on as assessment of the client, CPR© as summoning the emergency response team (D) may be indicated. 430. A client with a large pleural effusion undergoes a thoracentesis. Following the procedure, which assessment finding warrants immediate intervention by the nurse? • The client has asymmetrical chest wall expansion • The clients complain of pain at the insertion site • The client chest’s x-ray indicates decreased pleural effusion • The client’s arterial blood gases are pH 7.35, PaO2 85, Pa CO2 35, HCO3 26 • Rationale: A potential complication of thoracentesis is a pneumothorax. The symptoms of a pneumothorax are uneven, unequal movement of the chest wall. A is an expected finding after the local anesthetic effects “wear off” B is a desired result of thoracentesis and C is within normal limits. 431. A client is receiving an IV solution labeled Heparin Sodium 20,000 Units in 5% dextrose injection 500 ml at 25 ml/hour. How many units of heparin is the client receiving each hour? • 1000 units/hour • Rationale:20000/500=40x25=1000 432. The nurse is preparing a client for discharge from the hospital following a liver transplant. Which instruction is most important for the nurse to include in this client’s discharge teaching plan? • Monitor for an elevated temperature • Measure the abdominal girth daily • Report the onset of sclera jaundice • Keep a record of daily urinary output • Rationale: The client should be instructed to monitor or elevated temperature because immunosuppressant agents, which are prescribed to reduce rejection after transplantation, place the client at risk for infection. The client should recognize sign of liver rejection, such as sclera jaundice and increasing abdominal girths, but fever may be the only sign of infection. A is not as important and monitoring for signs of infection. 433. The nurse is conducting health assessments. Which assessment finding increases a 56year-old woman’s risk for developing osteoporosis? • Body mass index of (BMI) of 31 • 20 pack-year history of cigarette smoking • Birth control pill usage until age 45 • Diabetes mellitus in family history • Rationale: Cigarette smoking (2 packs/day x 310 years = 20 packs-year) increases the risk of osteoporosis. BMI of 30 or greater falls in the category of obesity which increase weight bearing that is protective against osteoporosis. C contain estrogens, and are also protective against development of osteoporosis. D is not related to the development of osteoporosis. 434. A young couple who has been unsuccessful in conceiving a child for over a year is seen in the family planning clinic. During an initial visit, which intervention is most important for the nurse to implement? • Determine current sexual practice • Prepare a female client for an ultrasound • Request an sperm sample for ovulation • Evaluate hormone levels on both client • Rationale: First a history should be obtained including practices that might be related to the infertility, such as douching, daily ejaculation or the male partner’s exposure to heat, such as frequent sauna or work environment which can decrease sperm production (A B or C) may be indicated after a complete assessment is obtained. 435. The nurse administers an oral antiviral to a client with shingles. Which finding is most important for the nurse report to the health care provider? • Decreased white blood cell count • Pruritus and muscle aches • Elevated liver function tests • Vomiting and diarrhea • Rationale: Elevated liver function enzymes are a serious side effect of antivirals and should be reported. A decrease white blood count is a consistent finding with shingle B and (C and D) are side effects that affect that are of less priority than A. 436. A client in the intensive care unit is being mechanically ventilated, has an indwelling urinary catheter in place, and is exhibiting signs of restlessness. Which action should the nurse take first? • Review the heart rhythm on cardiac monitors • Check urinary catheter for obstruction • Auscultated bilateral breath sounds • Give PRN dose of lorazepam (Ativan) • Rationale: Restlessness often results from decreased oxygenation so breath sounds should be assessed first. Giving an anxiolytic such as lorazepam, might be indicated but first the client should be assessed for the cause of the restlessness. An obstruction in the urinary drainage system can cause a distended bladder that may result in restlessness, but patent airway is the priority intervention. The client should be assessed before evaluating the cardiac rhythm on the monitor. 437. The nurse makes a supervisory home visit to observe an unlicensed assistive personnel (UAP) who is providing personal care for a client with Alzheimer’s disease. The nurse observes that whenever the client gets upset, the UAP changes the subject. What action should the nurse take in response to this observation? • Tell the UAP to offer more choices during the personal care to prevent anxiety • Meet with the UAP later to role model more assertive communication techniques • Assume care of the client to ensure that effective communication is maintained. • Affirm that the UAP is using and effective strategy to reduce the client’s anxiety. • Rationale: Reduction is an effective technique is managing the anxiety of client with Alzheimer’s disease, so the nurse should affirm the UAP is using an effective strategy (A). Nurse assertive communication and offering more choices (B) may increase… an agitation (C) is not indicated since the UAP is using redirection, an effective strategy. 438. An older female who ambulate with a quad-cane prefer to use a wheel chair because she has a halting and unsteady gait at times. Which interventions should the nurse implement? (Select all that apply) • Move personal items within client’s reach • Lower bed to the lower possible position • Give directions to call for assistance • Assist client to the bathroom in 2 hours. • Encourage the use of the wheelchair • Raise all bed rails when the client is resting • Rationale: A client who needs assistive devices, such as quad-cane is at risk for falls. Precautions that should implement include ensuring that personal items are within reach the bed is in the lowest position and directions are given to call assistance to minimize the risk for falls. Frequently assisting the client to the bathroom help ensure this client does not go the bathroom by herself, thereby decreasing the possibility of falling. 439. In evaluating the effectiveness of a postoperative client’s intermittent pneumatic compression devices, which assessment is most important for the nurse to complete? • Evaluate the client’s ability to use an incentive spirometer • Monitor the amount of drainage from the client’s incision • Observe both lower extremities for redness and swelling • Palpate all peripheral pulse points for volume and strength • Rationale: Intermittent compression devices (ICDs) are used to reduce venous stasis and prevent venous thrombosis in mobile and postoperative clients and its effectiveness is best assessed by observing the client’s lower extremities for early signs of thrombophlebitis. 440. A school-age child who weighs 42 pounds receives a post-tonsillectomy prescription for promethazine (Phenergan) 0.5 mg/kg IM to prevent postoperative nausea. The medication is available in 25 mg/ml ampules. How many ml should the nurse administer? (Enter numeric value only. If rounding is required, round to the nearest tenth). • 0.4 • Rationale: Convert pounds to kg 42lbs = 19.09 kg • Next calculate to prescribed dose, 0.5 mg x 1909 kg = 9.545 • Then use the desired dose/ dose on hand x volume on hand (9.545/25x1ml =0.3818=0.4 ml) • Or use ratio proportion (9.545 mg: x ml = 25 mg: 1ml • 25x = 9.545 • X= 0.3818 = 0.4) 441. A nurse stops at the site of a motorcycle accident and finds a young adult male lying face down in the road in a puddle of water. It is raining, no one is available to send for help, and the cell phone is in the car about 50 feet away. What action should the nurse take first? • Examine the victim’s body surfaces for arterial bleeding • Stabilize the victim’s neck and roll over to evaluate his status • Return to the car to call emergency response 911 for help • Open the airway and initiate resuscitative measures 442. During a well-baby, 6-month visit, a mother tells the nurse that her infant has had fewer ear infections than her 10-year-old daughter. The nurse should explain that which vaccine is likely to have made the difference in the siblings’ incidence of otitis media? • Varicella Virus Vaccine Live • Hemophilic Influenza Type B (HiB) vaccine • Pneumococcal vaccine • Palivizumab vaccine for RSV 443. The healthcare provider prescribes Morphine Sulfate Oral Solution 38 mg PO q4 hours for a client who is opioid-tolerant. The available 30 mL bottle is labeled, 100 mg/5 mL (20mg/mL), and is packaged with a calibrated oral syringe to provide to provide accurate dose measurements. How many mL should the nurse administer? (Enter the numerical value only. If rounding is required, round to the nearest tenth.) • 1.9 • Rationale: D/H x Q 38/20x1=1.9 mL 444. The nurses observes that a postoperative client with a continuous bladder irrigation has a large blood clot in the urinary drainage tubing. What actions should the nurse perform first? • Observe the amount of urine in the client’s urinary drainage bag 445. Which medication should the nurse anticipate administering to a client who is diagnosed with myxedema coma? • Intravenous administration of thyroid hormones • Oral administration of hypnotic agents • Intravenous bolus of hydrocortisone • Subcutaneous administration of vitamin k • Rationale: The high mortality of myxedema coma requires immediate administration of IV thyroid hormones (A). (B) Is contraindicated, because eves small doses can cause profound somnolence lasting longer than expected. (C) Is administered to clients diagnosed with adrenal insufficiency (Addisonian crisis) and (D) to clients who have had an overdose of warfarin. 446. The nurse who works in labor and delivery is reassigned to the cardiac care unit for the day because of a low census in labor and delivery. Which assignments is best for the nurse to give this nurse? • Transfer a client to another unit • Monitor the central telemetry • Perform the admission • Assist cardiac nurses with their assignments • Rationale: When receiving staff from another specialty unit, the charge nurse should allow the nurse to assist where possible (D) without taking a client assignment so that the nurse is not asked to perform unfamiliar skills (A, B, C) are likely to involve skills the nurse is not accustomed to performing. 447. A client who had an emergency appendectomy is being mechanically ventilated, and soft wrist restrain are in place to prevent self extubation. Which outcome is most important for the nurse to include in the client’s plan of care? • Understand pain management scale • Maintain effective breathing patterns • Absence of ventilator associated pneumonia • No injuries refer to soft restrains occur • Rationale: Basic airway management (B) is the priority. Pain management (A), risk of infection (C), and prevention of injury (D) do not have the same priority as (C) 448. After a routine physical examination, the healthcare admits a woman with a history of Systemic Lupus Erythematous (SLE) to the hospital because she has 3+ pitting ankle edema and blood in her urine. Which assessment finding warrants immediate intervention by the nurse? • Blood pressure 170/98 • Joint and muscle aches • Urine output 300 ml/hr • Dark, rust-colored urine 449. The nurse is explaining the need to reduce salt intake to a client with primary hypertension. What explanation should the nurse provide? • High salt can damage the lining of the blood vessels • Too much salt can cause the kidneys to retain fluid • Excessive salt can cause blood vessels to constrict • Salt can cause information inside the blood vessels • Rationale: Excessive salt intake can contribute to primary hypertension by causing renal salt retention which influence water retention that expands blood volume and pressure (ACD) are not believed to contribute to primary hypertension. 450. In assessing a pressure ulcer on a client’s hip, which action should the nurse include? • Determine the degree of elasticity surrounding the lesion • Photograph the lesion with a ruler placed next to the lesion • Stage the depth of the ulcer using the Braden numeric scale • Use a gloved finger to palpate for tunneling around the lesion • Rationale: An ulcer extends into the dermis or subcutaneous tissue and is likely to increase in size and depth, so assessment should include photograph with measuring device to document the size of the lesion. 451. A nurse is planning discharge care for a male client with metastatic cancer. The client tells the nurse that he plans to return to work despite pain, fatigue, and impending death. Which goals is most important to include in this client’s plan of care? • Implements decisions about future hospices services within the next 3 months. • Marinating pain level below 4 when implementing outpatient pain clinic strategies. • Request home health care if independence become compromised for 5 days. • Arranges for short term counseling stressors impact work schedule for 2 weeks. • Rationale: An outpatient pain clinic provides the interdisciplinary services needed to manage chronic pain. Also the client has a terminal disease and is being discharge home, hospice and health care are not indicating at this time. Short term counseling is not an option. 452. The first paddle has been placed on the chest of a client who needs defibrillation. Where should the nurse place the second paddle? (Mark the location where the second paddle should be placed on the image). • 453. A client who had an open cholecystectomy two weeks ago comes to the emergency department with complaints of nausea, abdominal distention, and pain. Which assessment should the nurse implement? • Auscultate all quadrant of the abdomen. • Perform a digital rectal exam • Palpate the liver and spleen • Obtain a hemoccult of the client’s stool 454. The nurse is caring for several clients on a telemetry unit. Which client should the nurse assess first? The client who is demonstrating • A paced rhythm with 100% capture after pacemaker replacement • Normal sinus rhythm and complaining of chest pain • Atrial fibrillation with congestive heart failure and complaining of fatigue • Sinus tachycardia 3 days after a myocardial infarction 455. A 12-lead electrocardiogram (ECG) indicates a ST elevations in leads V1 to V4, for a client who reports having chest pain. The healthcare provider prescribe tissue plasminogen activator (t-PA). Prior to initiating the infusion, which interventions is most important for the nurse to implement? • Complete pre-infusion checklist 456. The nurse is evaluating the health teaching of a female client with condyloma acuminate. Which statement by the client indicates that teaching has been effective? • Early treatment is very effective • I will clean my hot tub better • These warts are caused by a fungus • I need to have regular pap smears 457. While the nurse is conducting a daily assessment of an older woman who resides in a long-term facility, the client begins to cry and tells the nurse that her family has stopped calling and visiting. What action should the nurse take first? • Ask the client when a family member last visited her. • Determine the client’s orientation to time and space • Review the client’s record regarding social interactions • Reassure the client of her family’s love for her 458. A female client with severe renal impairment is receiving enoxaparin (lovenox) 30 mg SUBQ BID. Which laboratory value due to enoxaparin should the nurse report to the healthcare provider? • creatinine clearance 25 mL/ minute • calcium 9 mg/dl • hemoglobin 12 grams/dl • partial thromboplastin time (PTT) 30 seconds 459. The nurse notes an increase in serosanguinous drainage from the abdominal surgical wound from an obese client. What action should the nurse implement? • Observe the wound for dehiscence 460. The nurse is assigned to care for clients on a medical unit. Based on the notes taken during the shift report, which client situation warrants the nurse’s immediate attention? • A 10-year-old who is receiving chemotherapy and the infusion pump is beeping • A young adult with Crohn’s disease who reports having diarrheal stools • An older adult with type 2 diabetes whose breakfast tray arrives 20 minutes late • A teenager who reports continued pain 30 minutes after receiving an oral analgesic • Rationale: an infiltration of a caustic agent can cause tissue damage and children are at greater risk for fluid volume imbalances 461. A nurse is conducting a physical assessment of a young adult. Which information provides the best indication of the individual nutritional status? • Condition of hair, nails, and skin • A 24-hour diet history • History of a recent weight loss • Status of current petite • Rationale: the assessment of hair, nails, and skin is most effective of long-term nutritional status, which is important in the healing process. 462. The nurse is preparing to administer an infusion of amino acid-dextrose total parenteral nutrition (TPN) through a central venous catheter (CVC) line. Which action should the nurse implement first? • Check the TPN solution for cloudiness • Attach the IV tubing to the central line • Set the infusion pump at the prescribed rate • Prime the IV tubing with TPN solution 463. A newly admitted client vomits into an emesis basin as seen in the picture. The nurse should consult with the healthcare provider before administering which of the client’s prescribes medications? • Clopidogrel (Plavix), an antiplatelet agent, given orally • Nitroglycerin (nitro-dur), an antianginal, to be given transdermally • Methylprednisolone (solu-medrol), a corticosteroid, to be given IV • Furosemide (lasix), a loop diuretic, to be given intravenously • Enoxaparin (lovenox), a low-molecular weight heparin to be given subcutaneous 464. A client diagnosed with bipolar disorder is going home on a week-end pass. Which suggestions should give the client’s family to help them prepare for the visit? • Encourage the family to plan daily activities to keep the client busy • Have friends and family visit the client at a welcome home party • Discuss the importance of continuing the usual at-home activities • Instruct family to monitor the client’s choice of television programs 465. On a busy day, one hour after the shift report is completed, the charge nurse learns that a female staff nurse who lives one hour away from the hospital forgot her prescription eye glasses at home. What action should the charge nurse take? • Encourage the nurse to purchase reading glasses in the hospital gift shop • Request another nurse to assist the staff nurse with her documentation • Ask the nurse to return home and get her prescription eyeglasses for work. • Tell the staff nurse to take a day off and change her weekly work schedule 466. A client with pneumonia has an IV of lactated ringer’s solution infusing at 30ml/hr current labor.…sodium level of 155 mEq/L, a serum potassium level of 4mEq/L…. what nursing intervention is most important? • Obtain a prescription to increase the IV rate 467. After teaching a male client with chronic kidney disease (CKD) about therapeutic diet…which menu of foods indicates that the teaching was effective? Select all that apply • A slice of whole grain toast • A bowl of cream of wheat 468. When five family members arrive at the hospital, they all begin asking the nurse questions regarding the prognosis of their critically ill mother. What intervention should the nurse implement first? • Include the family in client’s care • Request the chaplain’s presence • Ask the family to identify a specific spokesperson • Page the healthcare provider to speak with family. 469. An older male who is admitted for end stage of chronic obstructive pulmonary disease (COPD) tells the nurse …. The client provides the nurse with a living will and DNR. What action should the nurse implement? • Obtain a prescription for DNR 470. A client who is recently diagnosed with type 2 diabetes mellitus (DM) ask the nurse how this type of diabetes leads to high blood sugar. What Pathophysiology mechanism should the nurse explain about the occurrence of hyperglycemia in those who have type 2 DM? • The body cells develop resistance to the action of insulin. 471. During a left femoral artery aortogram, the healthcare provider inserts an arterial sheath and initiate. Through the sheath to dissolve an occluded artery. Which interventions should the nurse implement? • Instruct the client to keep the left leg straight • Observe the insertion site for a hematoma • Circle first noted drainage on the dressing 472. A client whose wrists are sutured from a recent suicide attempt is been transferred from a medical unit. Which nursing diagnosis is of the highest priority? • Risk for self-directed violence related to impulsive actions 473. The nurse reviews the signs of hypoglycemia with the parents of a child with Type I diabetes mellitus. The parents correctly understand signs of hypoglycemia if they include which symptoms? • Fruity breath odor • Polyphagia • Diaphoresis • Polydipsia 474. One day following a total knee replacement, a male client tells the nurse that he is unable to transfer because it is too painful. What action should the nurse implement? • Encourage use of analgesics before position change 475. The nurse is caring for a client with hypovolemic shock who is receiving two units of packed red blood cells (RBCs) through a large bore peripheral IV. What action promotes maintenance of the client’s cardiopulmonary stability during the blood transfusion? • Increase the oxygen flow via nasal cannula if dyspnea is present. • Place in a Trendelenburg position to increase cerebral blood flow • Monitor capillary glucose measurements hourly during transfusion. • Encourage increased intake of oral fluid to improve skin turgor. 476. A client with end-stage liver failure is declared brain dead. The family wants to discontinue feeding and donate any viable organs. Which action should the nurse take? • Contact the regional organ procurement agency 477. Which information is more important for the nurse to obtain when determining a client’s risk for (OSAS)? • Body mass index • Level of consciousness • Self-description of pain • Breath sounds 478. During the transfer of a client who had major abdominal surgery this morning, the post anesthesia care unit (PACU) nurse reports that the client, who is awake and responsive continues to report pain and nausea after receiving morphine 2 mg IV and ondansetron 4 mg IV 45 mints ago. Which elements of SBAR communication are missing from the report given by the PACU nurse? (Select all that apply) • Situation • Background • Assessment • Recommendation • Rationales. • Rationale: BCD are correct. The current situation is reported regarding the client’s nausea and pain (A). Based on SBAR communication, critical information about the client’s clinical history (B), and assessment (C) such as pain scale or vital signs related to client’s response to medication, are not included, nor are any recommendations for further follow-up (D). (E) Is not a component of SBAR communication 479. The nurse is triaging victims of a tornado at an emergency shelter. An adult woman who has been wandering and crying comes to the nurse. What action should the nurse take? • Check the client’s temperature, blood sugar, and urine output. • Transport the client for laboratory client for laboratory test and electrocardiogram (EKG) • Delegate care of the crying client to an unlicensed assistant • Send the client to the shelter’s nutrient center to obtain water and food. • Rationale: According to the simple triage and Rapid Treatment (START) protocol of triage, the nurse should determine which client fit the objective of providing the greatest good for the greatest number of people who are most likely to survive. Delegating the care of the crying person to an unlicensed assistant allow the nurse to care for the injured who require intervention based on their ability to breath, maintain circulation and follow simple commands. A and B are not indicated at this time. Although food and water may be indicative, the woman’s distress should not be dismissed by sending her to the shelter alone. 480. A client in septic shock has a double lumen central venous catheter with one liter of 0.9% Normal Saline Solution infusing at 1 ml/hour through one lumen and TPN infusing at 50 ml/hr. through one port. The nurse prepared newly prescribed IV antibiotic that should take 45 mints to infuse. What intervention should the nurse implement? • Use a secondary port of the Normal Saline solution to administer the antibiotic. • Add the antibiotic to the TPN solution, and continue the normal saline solution. • Stop the TPN infusion for the time needed to administer the prescribed antibiotic. • Add the antibiotic to the Normal Saline solution and continue both infusions. • Rationale: A client in septic shock needs antibiotic administered in a timely manner to ensure maintenance of therapeutic serum level. The nurse should administer the antibiotic using a secondary port of the Normal Saline solution. No other medications should be administered using TPN tubing or solution. TPN not should be place on hold because sudden cessation will cause rapid change in serum glucose levels. Excessively delays in the administration of the antibiotics. 481. A male client returns to the mental health clinic for assistance with his anxiety reaction that is manifested by a rapid heartbeat, sweating, shaking, and nausea while driving over the bay bridge. What action I the treatment plan should the nurse implement? • Tell the client to drive over the bridge until fear is manageable • Teach client to listen to music or audio books while driving • Encourage client to have spouse drive in stressful places. • Recommend that the client avoid driving over the bridge. • Rationale: Desensitization is component in the treatment plan for clients with panic attacks which is best approached with anxiety-reducing strategies, such as listening to audio book (B) during situation that precipitate symptoms (A) is a flooding technique that requires professional guidance. 482. Which intervention should the nurse include in the plan of care for a client with leukocytosis? • Avoid intramuscular injections • Monitor temperature regularly • Assess skin for petechiae or bruising • Implement protective isolation measures 483. The nurse is teaching a client about the antiulcer medications ranitidine which was… statement best describes the action of this drug? • It blocks the effects of histamine, causing decreased secretion of acid • Ranitidine will neutralize gastric acid and decrease gastric pH • This drug provides a protective coating over the gastric mucosa • It effectively blocks 97% of the gastric acid secreted in the stomach 484. A client with superficial burns to the face, neck, and hands resulting from a house fire…which assessment finding indicates to the nurse that the client should be monitored for carbon monoxide…? • Expiratory stridor and nasal flaring • Mucous membranes cherry red color • Carbonaceous particles in sputum • Pulse oximetry reading of 80 percent 485. A female client who was mechanically ventilated for 7 days is extubated. Two hours later…productive cough, and her respirations are rapids and shallow. Which intervention is most important? • Review record of recent analgesia • Provide frequent pulmonary toilet • Prepare the client for intubation • Obtain STAT arterial blood gases 486. The nurse delegates to an unlicensed assistive personnel (UAP) denture care for a client with…daily leaving. When making this assignment, which instruction is most important for the nurse to do? • Place a washcloth in the sink while cleaning the dentures 487. The nurse is assessing the emotional status of a client with Parkinson’s disease. Which client finding is most helpful in planning goals to meet the client’s emotional needs? • Cries frequently during the interview • Stares straight ahead without blinking • Face does not convey any emotion • Uses a monotone when speaking 488. When changing a diaper on a 2-day-old infant, the nurse observes that the baby’s legs are… this finding, what action should the nurse take next? • Notify the healthcare provider • Continue care since this is a normal finding • Document the finding in the record Perform range of motion to the joint 489. A school-aged child was recently diagnosed with celiac disease. Which instruction should the nurse give the classroom teacher? • The child should avoid eating homemade cookies and cupcakes during parties 490. The nurse is presenting information about fetal development to a group of parents with…when discussing cephalocaudal fetal development, which information should the nurse gives the parents? • A set order in fetal development is expected • Growth normally occurs within one organ at a time • Development progress from head to rump • Organ formation is directed by brain development 491. A client has a prescription for lorazepam 2mg for alcohol withdrawal symptoms. Which finding… the client? • Blood pressure 149/101 • Irregular pulse rate of 80 • Oral temperature is 98.9 F (37.1 C) • Pain rated 7 on scale 1-10 492. A client with end-stage liver failure is declared brain dead. The family wants to discontinue feeding and donate any viable organs. Which action should the nurse take? • Contact the regional organ procurement agency 493. A male client who was hit by a car while dodging through traffic is admitted to the emergency department with intracranial pressure (ICP). A computerized tomography (CT) scan reveals an intracranial bleed. After evacuation of hematoma, postoperative prescription include: intubation with controlled mechanical ventilation to PaCO2…what is the pathophysiological basis for this ventilator settings? • Hypocapnea reduces ICP 494. During a cardiopulmonary resuscitation of an intubated client, the nurse detects a palpable pulse throughout the two minutes cycle chest compression and absent breath sounds over the left lung. What action should the nurse implement? Prepare for the endotracheal tube to be repositioned 495. A male client is admitted with burns to his face and neck. Which position should the nurse place the client to prevent contract? • Hyperextended with neck supported by a rolled towel. 496. A male client is discharged from the intensive care unit following a myocardial infarction, and the healthcare provider low-sodium diet. Which lunch selection indicates to the nurse that this client understands the dietary restrictions? • Turkey salad sandwich. • Clam chowder • Macaroni and cheese • Bacon, lettuce, and tomato sandwich 497. The nurse prepares an intravenous solution and tubing for a client with a saline lock, as seen in the video. Which nurse takes next • • Open the roller clamp on the tubing. 498. The healthcare provider prescribes heparin protocol at18 units/kg/hr for a client with a possible pulmonary embolism. This client weighs 144 pounds. The available solution is labeled, heparin sodium 25,000 units in 5% dextrose 250 ml. the nurse should program the pump to deliver how many ml/hr? (Enter numeric value only. If rounding is require round to the nearest whole number.) • Answer 12 • Rationale: 144/2.2= 65kg • 18units/kg/hr • 65 kg x 18units/kg/hr= 1170 units/hr 25000 units heparin/250 ml of D5W = 100 units heparin per ml of solution Formula D/H x A = X 499. A client is admitted with a wound on the right hand and associated cellulitis. In assessing the client’s hand, which finding required most immediate follow-up by the nurse? • Cyanotic nailbeds • Localized tenderness • Diffuse erythema • Skin hot to touch 500. The nurse is arranging home care for an older client who has a new colostomy following a large bowel resection three day. The clients plan to live with a family member. Which action should the nurse implement? Select all that apply • Assess the client for self-care ability • Provide pain medication instructions • Teach care of ostomy to care provider 501. A female client with chronic urinary retention explains double voiding technique to the nurse by stating she voids partially, hold the remaining urine in her bladder for three minutes, then voids again to empty her bladder fully. How should the nurse respond? • Advise the client to empty her bladder fully when she first voids 502. When conducting diet teaching for a client who was diagnosed with hypoparathyroidism, which foods should the nurse encourage the client to eat? • Yogurt. • Processed cheese. • Nuts • Fresh turkey • Fresh chicken 503. The nurse is assessing a middle-aged adult who is diagnosed with osteoarthritis. Which factor in this client’s history is a contributor to the osteoarthritis? • Long distance runner since high school. Lactose intolerant since childhood Photosensitive to a drug currently taking • Recently treated for deep vein thrombosis 504. When assessing a male client, the nurse notes that he has unequal lung expansion. What conclusion regarding this finding is most likely to be accurate? The client has • A collapsed lung • A history of COPD • A chronic lung infection • Normally functioning lungs 505. The nurse manager is conducting an in-services education program on the fire evacuation of the newborn recovery. What intervention should the nurse manager disseminate to the staff? • Evacuate each infant with mother via wheelchair 506. An adult man reports that he recently experienced an episode of chest pressure and breathlessness when he was jogging in the neighborhood. He expresses concern because both of his deceased parents had heart disease and his father was a diabetic. He lives with his male partner, is a vegetarian, and takes atenolol which maintain his blood pressure at 138/74. Which risk factors should the nurse explore further with the client? Select all that apply • History of hypertension. • Family heath history. 507. A client with severe full-thickness burns is scheduled for an allografting procedure. Which information should the nurse provide the client? • Human source grafts require monitoring for signs of graft rejection 508. The nurse is administering a 750 ml cleansing enema to an adult client. After approximately150 ml of enema has informed, the client states, ‘stop I can’t hold anymore.” What action should the nurse take? • Clamp the tubing and instruct the client to breathe deeply before continuing. 509. The nurse requests a meals tray for a client follows Mormon beliefs and who is on clear liquid diet following abdominal surgery. Which meal item should the nurse request for this client? (Select all that apply) Apple juice • Chicken broth. • Hot chocolate • Orange juice • Black coffee 510. The nursing staff on a medical unit includes a registered nurse (RN), practical nurse (LPN), and unlicensed assistant personnel (UAP). Which task should the charge nurse assign to the RN? • Supervised a newly hired graduate nurse during an admission assessment 511. Following breakfast, the nurse is preparing to administer 0900 medications to clients on a medical floor. Which medication should be held until a later time? • The mucosal barrier, sucralfate (Carafate), for a client diagnosed with peptic ulcer disease. • Rationale: Carafate coats the mucosal lining prior to eating a meal 512. The father of 4-year-old has been battling metastatic lung cancer for the past 2 years. After discussing the remaining options with his healthcare provider, the client requests that all treatment stop and that no heroic measures be taken to save his life. When the client is transferred to the palliative care unit, which action is most important for the nurse working on the palliative care unit to take in facilitating continuity of care? • Obtain a detailed report from the nurse transferring the client. 513. The nurse is making a home visit to a male client who is in the moderate stage of Alzheimer’s diseases. The client’s wife is exhausted and tells the nurse that the family plans to take turns caring for the client in their home, each keeping him for two weeks at a time. How should the nurse respond? • Suggest enrolling the client in adult daycare instead of rotating among family. 514. The healthcare provider prescribes oxycodone/ aspirin 1 tab PO every 4h as needed for pain, for a client with polycystic kidney disease. Before administering this medication, which component of the prescription should the nurse question? • Aspirin content. • Dose • Route • Risk for addiction 515. A young adult male was admitted 36 hours ago for a head injury that occurred as the result of a motorcycle accident. In the last 4 hours, his urine output has increased to over 200 ml/H. Before reporting the finding to the healthcare provider, which intervention should the nurse implement? • Evaluate the urine osmolality and the serum osmolality values. 516. A female client is taking alendronate, a bisphosphate, for postmenopausal osteoporosis. The client tells the nurse that she is experiencing jaw pain. How should the nurse respond? • Report the client’s jaw pain to the healthcare provider. 517. A male client has received a prescription for orlistat for weight and nutrition management. In addition to the medication, the client states he plans to take a multivitamin. What teaching should the nurse provide? • Be sure to take the multivitamin and the medication at least two hours apart for best absorption and effectiveness. 518. Which intervention should the nurse implement for a client with a superficial (first degree) burn? • Place wet cloths on the burned areas for short periods of time. 519. What is the primary goal when planning nursing care for a client with degenerative joint disease (DJD)? • Achieve satisfactory pain control. 520. An adult woman who is seen in the clinic with possible neuropathic pain of the right leg rates her pain as a 7 on a 10 point scale. What action should the nurse take? • Encourage the client to describe the pain. 521. A client has both primary IV infusion and a secondary infusion of medication. An infusion pump is not available. The nurse needs to determine the current rate of infusion of the primary IV. Where should the nurse observe to determine the rate of infusion? • 522. The nurse is conducting the initial assessment of an ill client who is from another culture…. What response should the nurse provide? • “What practices do you believe will help you heal?” 523. The nurse is caring a client with NG tube. Which task can the nurse delegate to the UAP? • Disconnect the NG suction so the client can ambulate in the hallway 524. The nurse is collecting a sterile urine specimen using a straight catheter tray for culture…. (Arrange from first action to last). 1. Drape the client in a recumbent position for privacy 2. Open the urinary catheterization tray 3. Don sterile gloves using aseptic technique 4. Use forceps and swaps to clean the urinary meatus 525. The nurse is caring for a toddler with a severe birth anomaly that is dying. The parents… holding the child as death approaches. Which intervention is most important for the nurse? • Notify nursing supervisor and hospital chaplain of the child’s impending death. 526. The nurse is assessing a 4-year-old boy admitted to the hospital with the diagnosis of possible nephrotic syndrome. Which statement by the parents indicates a likely correlation to the child’s diagnosis? • “I couldn’t get my son’s socks and shoes on this morning” 527. Which interventions should the nurse include in a long-term plan of care for a client with COPD? • Reduce risk factors for infection • Limit fluid intake to reduce secretions • Use diaphragmatic breathing to achieve better exhalation • Administer high flow oxygen during sleep 528. A health care provider continuously dismisses the nursing care suggestions made by staff nurses. As a result…dealing with the healthcare provider. What action should the nursemanager implement? • Plan an interdisciplinary staff meeting to develop strategies to enhance client care 529. A 2-year-old girl is brought to the clinic for a routine assessment and all findings are within the normal limits. However, the mom expresses concern over her daughter’s protruding abdomen and tells the nurse that she is worry that her child is becoming overweight. How should the nurse respond to the mother’s comment? • Explain that a protruding abdomen is typical for toddlers 530. A female client reports she has not had a bowel movement for 3 days, but now is defecating frequent small amount of liquid stool. Which action should the nurse implement? • Digitally check the client for a fecal impaction 531. A client admitted with an acute coronary syndrome (ACS) receives eptifibatide, a glycoprotein (GP) IIB IIIA inhibitor, which important finding places the client at greatest risk? • Unresponsive to painful stimuli 532. A toddler presents to the clinic with a barking cough, strider, refractions with respiration, the child's skin is pink with capillary refill of 2 seconds. Which intervention should the nurse implement? • Administered Nebulized Epinephrine 533. The nurse caring for a client with dysphagia is attempting to insert an NG tube, but the client will not swallow and is not gagging. What action should the nurse implement to facilitate the NGT passage into the esophagus? • Flex the client’s head with chin to the chest and insert. 534. The nurse plans to use an electronic digital scale to weight a client who is able to stand. Which intervention should the nurse implement to ensure that measurement of the client’s weight is accurate? • Ensure that the scale is calibrated before a weight is obtained 535. The nurse observes a newly hired unlicensed assistive personnel (UAP) performing a fingestick to obtain a client’s blood glucose. Prior to sticking the client’s finger, the UAP explains the procedure and tell the client that it I painless. What action should the nurse take? • Allow the UAP to complete the procedure, then discuss the painless comment privately with the UAP. 536. An African-American man come into the hypertension screening booth at a community fair. The nurse finds that is blood pressure is 170/94 mmHg. The client tells the nurse that he has never been treated for high blood pressure. What response should the nurse make? • Your blood pressure is a little high. You need to have it rechecked within one week 537. While attempting to stablish risk reduction strategies in a community, the nurse notes that the regional studies have indicated….persons with irreversible mental deficiencies due to hypothyroidism. The nurse should seek funding to implement which screening measure? • T4 levels in newborns 538. After applying an alcohol-based hand rub to the palms of the hand and rubbing the hand together, what action should the nurse do next? • Place one hand on top of the other and interlace the fingers 539. A nurse is preparing to feed a 2-month-old male infant with heart failure who was born with congenital heart defect. Which intervention should the nurse implement? • Allow the infant to rest before feeding 540. While removing an IV infusion from the hand of a client who has AIDS, the nurse is struck with the needle. After washing the puncture site with soap & water, which action should the nurse take? • Notify the employee health nurse. 541. A nurse receive a shift report about a male client with Obsessive compulsive disorder (OCD). The nurse does morning rounds and reaches the client while he is repeatedly washing the top of the same table. What intervention should the nurse implement? • Allow time for the behavior and then redirect the clients to other activities 542. The nurse is caring for a client immediately after inserting a PICC line. Suddenly, the client becomes anxious and tachycardiac, and loud churning is heard over the pericardium upon auscultation. What action should the nurse take first? • Place client in Trendelenburg position on the left side. 543. A client admitted to the telemetry unit is having unrelieved chest pain after receiving 3 sublingual nitroglycerin tablets and morphine 8 mg IV. The electrocardiogram reveals sinus bradycardia with ST elevation. In what order should the nurse implement the nursing actions? (Arrange first to last) 1. Call the rapid response team to assist 2. Move the crash cart to the client room 3. Notify the client’s healthcare provider 4. Inform the family of the critical situation 544. The nurse is preparing dose # 7 of an IV piggyback infusion of tobramycin for a 73-yearol client with... Infected pseudomonas aeruginosa. Which assessment data warrants further intervention by the nurse? • Peak and through levels has not been drawn since the tobramycin was started 545. During a left femoral artery aortogram, the healthcare provider inserts an arterial sheath and initiate…through the sheath to dissolve an occluded artery. Which interventions should the nurse implement? • Instruct the client to keep the left leg straight • Observe the insertion site for a hematoma • Circle first noted drainage on the dressing 546. A client with HIV and pulmonary coccidioidomycosis is receiving amphotericin B. which assessment finding should the nurse report to the healthcare provider? • Urinary output of 25mL per hour 547. A client delivers a viable infant, but begins to have excessive uncontrolled vaginal…notifying the health care provider of the clients’ condition, what information is most…. • Maternal blood pressure • Maternal apical pulse rate • Time Pitocin infusion completed • Total amount of Pitocin infused 548. An infant born to a heroin-addicted mother is admitted to the neonatal care unit. What behaviors can…to exhibit? • Irritability and a high-pitched cry • Lethargy and poor suck • Facial abnormalities and microcephaly • Low birth weight and intrauterine growth retardation 549. A multigravida, full-term, laboring client complains of “back labor”. Vaginal examination reveals that the client’s 3 cm with 50% effacement and the fetal head is at -1 station. What should the nurse implement? • Turn the client to a lateral position • Apply counter-pressure to the sacral area • Notify the scrub nurse to prepare the OR • Ambulate the client between contractions 550. A client with gestational diabetes is undergoing a non-stress test (NST) at 34-week gestation… is 144 beats/minute. The client is instructed to mark the fetal monitor by pressing a button each time the baby moves. After 20 minutes, the nurse evaluates the fetal monitor strip. What… • Two FHR accelerations of 15 beats/minute x 15 seconds are recorded 551. A male infant born at 28-weeks gestation at an outlying hospital is being prepared for transport to a respiration are 92 breaths/minute and his heart rate is 156 beats/minute. Which drug is the transport administration to this infant? • Instill beractant 100 mg/kg in endotracheal tube. 552. A postpartal client complains that she has the urge to urinate every hour but is only able to void a small amount. What interventions provides the nurse with the most useful information? • Catheterize for residual urine after next voiding • Initiate a perineal pad count • Assess for a perineal hematoma • Determine the client’s usual voiding pattern 553. During a 26-week gestation prenatal exam, a client reports occasional dizziness…What intervention is best for the nurse to recommend to this client? • Lie on the left or right side when sleeping or resting 554. Artificial rupture of the membrane of a laboring reveals meconium-stained fluid, what is… the priority? • Have a meconium aspirator available at delivery 555. A 6-year-old child with acute infectious diarrhea is placed on a rehydration therapy…Which action should the nurse instruct the parents to take if the child begins to vomit? • Continue giving ORS frequently in small amounts 556. A client is receiving oxytocin (Pitocin) to augment early labor. Which assessment is most important time the infusion rate is increases? • Contraction pattern • Blood pressure • Infusion site • Pain level 557. An infant is placed in a radiant warmer immediately after birth. At one hour of age, the nurse finds the infant tachypneic, and hypotonic. What is the first action that the nurse should take? • Determine the infant’s blood sugar level 558. A toddler with a history of an acyanotic heart defect is admitted to the pediatric intensive…rate of 60 breaths/ minute, and a heart rate of 150 beats/minute. What action should the nurse take? • Obtain a pulse oximeter reading 559. In preparing a diabetes education program, which goal should the nurse identify as the primary emphasis for a class on diabetes self-management? • Enable clients to become active participants in controlling the disease process 560. To obtain an estimate of a client’s systolic B/P. What action should the nurse take first? • Palpate the client’s brachial pulse • Pump up the blood pressure cuff • Position the stethoscope diaphragm • Release the blood pressure cuff valve 561. A client is admitted to isolation with the diagnosis of active tuberculosis (TB). Which infection control measures should the nurse implement? • Negative pressure environment • Contact precautions • Droplet precautions • Protective environment 562. A client is receiving an IV of heparin sodium 25000 units in 5% dextrose injection 500 ml at 14 ml/hour…verify that the client is receiving the prescribed amount of heparin. How many units is the client receiving?700 563. Rationale: 25000/500x14=700 564. A client currently receiving an infusion labeled Heparin Sodium 25,000 Units in 5% Dextrose Injection 500 mL at 14 mL/hour. A prescription is received to change the rate of the infusion to 900 units of Heparin per hour. The nurse should set the infusion pump to deliver how many mL/hour? (Enter numeric value only). 18 • Rationale: 450000/25000=18 565. The nurse notes the client receiving heparin infusion labeled, Heparin Na 25,000 Units in 5% Dextrose injection 500 ml at 50ml/hr. What dose of Heparin is the client receiving per hour? • 2,500 566. A male client is returned to the surgical unit following a left nephrectomy and is medicated with morphine. His dressing has a small amount of bloody drainage, and a JacksonPratt bulb surgical drainage device is in place. Which interventions is most important for the nurse to include in this clients plan of care? • Monitor urine output hourly. • Assess for back muscle aches • Record drainage from drain • Obtain body weight daily 567. The family of a client who just died arrives on the nursing unit after receiving telephone notification of the death. Several family members state they would like to view the body. How should the nurse respond? • Offer to go with the family members to view the body. 568. The nurse is assessing a first day postpartum client. Which finding is most indicative of a postpartum infection? • Moderate amount of foul-smelling lochia. 569. An older woman who has difficulty hearing is being discharged from day surgery following a cataract extraction & lens implantation. Which intervention is most important for the nurse to implement to ensure the client's compliance with self-care? • Have the client vocalize the instructions provided. 570. A primigravida client is 36 weeks gestation is admitted to labor and delivery unit because her membranes ruptured 30minutes ago. Initial assessment indicates 2cm dilation, 50% effaced, -2 station, vertex presentation greenish colored amniotic fluid, and contractions occurring 3-5 minutes with a low FHR after the last contraction peaks: • Administer Oxygen via face mask • Apply an internal fetal heart monitor • Notify the healthcare provider • Use a vibroacoustic stimulator 571. A woman just received the Rubella vaccine after a delivery of a normal new born, has two children at home, ages 13 months and 3 years. Which instruction is most important to provide to the client? • Do not get pregnant for at least 3 months 572. Following a motor vehicle collision (MCV), a male adult in severe pain is brought to the emergency department via ambulance. His injured left leg is edematous, ecchymotic around the impact of injury on the thigh, and shorter than his right leg. Based on these findings, the client is at greatest risk for which complication? • Arterial ischemia • Tissue necrosis • Fat embolism • Nerve damage 573. A 2-year-old is bleeding from a laceration on the right lower extremity that occurred as the result of a motor vehicle collision. The nurse is selecting supplies to start an IV access. Which assessment finding is most significant in the nurse's selection of catheter size? • Thready brachial pulse. • Respirations of 24/minute • Right foot cool to touch • Swelling at the site of injury 574. The nurse prepares to insert an oral airway by first measuring for the correct sized airway. Which picture shows the correct approach to airway size measurement? • 575. A client with a recent colostomy expresses concern about the ability to control flatus. Which intervention is most important for the nurse to include in the client’s plan of care? • Adhere to a bland diet whenever planning to eat out • Decrease fluid intake at meal times • Avoid foods that caused gas before the colostomy • Eliminate foods high in cellulose 576. A male client arrives at the clinic with a severe sunburn and explains that he did not use sun screen because it was an overcast day. Large blisters are noted over his back and chest and his shirt is soaked with serosanguinous fluid. Which assessment finding warrants immediate intervention by the nurse? • Hypotension. • Fever and chills • Dizziness • Headache 577. A client with polycystic kidney disease (PKD) receiving antibiotics for an infected cyst is experiencing severe pain. What action should the nurse implement? • Hold the next dose of antibiotic until contacting the healthcare provider • Teach the client how to use a dry heating pad over the painful area • Encourage the client to practice pelvic floor exercises every hour • Assist the client to splint the site by applying an abdominal binder 578. Which statement is accurate regarding the pathological changes in the pulmonary system associated with acute (adult) respiratory distress syndrome (ARDS)? • Capillary hydrostatic pressure exceeds colloid osmotic pressure, producing interstitial edema • A high ventilation-to-perfusion ratio is characteristic of affected lung fields in ARDS • Functional residual capacity and lung compliance increase as the disease progresses • Interstitial edema that occurs due to capillary fluid shifts is usually more serious than alveolar edema 579. The nurse mixes 250 mg of debutamine in 250 ml of D5W and plans to administer the solution at rate client weighing 110 pounds. The nurse should set the infusion pump to administer how many ml per hour only. If rounding is required, round the nearest whole number.) • 45 580. During the intraoperative phase of care, the circulating nurse observes that the client is not adequately client's privacy. What is the best initial nursing action for the nurse to implement? • Instruct the scrub nurse to re-drape the client 581. An adult male who was admitted two days ago following a cerebrovascular accident (CVA) is confused and experiencing left-side weakness. He has tried to get out of bed several times, but is unable to ambulate without assistance. Which intervention is most important for the nurse to implement? • Ask a family member to sit with the client • Apply bilateral soft wrist restraints • Assign staff to check client q15 minutes • Install a bed exit safety monitoring device 582. A client in her first trimester of pregnancy complains of nausea. Which complementary therapy should the nurse recommend? • Drink chamomile tea at breakfast and in the evening. Eat food high in garlic with the evening meal • Join a yoga class that meets at least weekly • Increase cocoa in the diet and drink before bedtime 583. When gathering for a group therapy session at 1400 hours, a female client complains to the nurse that a smoking break has not been allowed all day. The nurse responds that 15 minute breaks were called over the unit intercom after breakfast and after lunch. The nurse is using what communication technique in responding to the client? • Doubt • Observation • Confrontation • Reflection 584. A female client with rheumatoid arthritis (RA) comes to the clinic complaining of joint pain and swelling. The client has been taking prednisone (Deltasone) and ibuprofen (Motrin Extra Strength) every day. To assist the client with self-management of her pain, which information should the nurse obtain? • Presence of bruising, weakness, or fatigue • Therapeutic exercise included in daily routine. • Average amount of protein eaten daily • Existence of gastrointestinal discomfort 585. The charge nurse of the Intensive Care Unit is making assignments for the permanent staff and one RN who was floated from a medical unit. The client with which condition is the best to assign to the float nurse? • Diabetic ketoacidosis and titrated IV insulin infusion • Emphysema extubated 3 hours ago receiving heated mist • Subdural hematoma with an intracranial monitoring device • Acute coronary syndrome treated with vasopressors 586. A client admitted to the emergency center had inspiratory and expiratory wheezing, nasal flaring, and thick, tenacious sputum secretions observed during the physical examination. Based on these assessment findings, what classification of pharmacologic agents should the nurse anticipate administering? • Beta blockers • Bronchodilators • Corticosteroids • Beta-adrenergics 587. The home health nurse is assessing a male client who has started peritoneal dialysis (PD) 5 days ago. Which assessment finding warrants immediate intervention by the nurse? • Finger stick blood glucose 120 mg/dL post exchange • Arteriovenous (AV) graft surgical site pulsations. • Anorexia and poor intake of adequate dietary protein • Cloudy dialysate output and rebound abdominal pain 588. A client’s telemetry monitor indicates ventricular fibrillation (VF). What should the nurse do first? • Administer epinephrine IV • Give an IV bolus of amiodarone • Provide immediate defibrillation • Prepare for synchronized cardioversion 589. In conducting a health assessment, the nurse determines that both parents of a child with asthma smoke cigarettes. What recommendation is best to the nurse to recommend to the parents? • avoid smoking in the house • stop smoking immediately • decrease the number of cigarettes smoke daily • obtain nicotine patches to assist in smoking sensation 590. A client who is schedule for an elective inguinal hernia repair today in day surgery is seem eating in the waiting area. What action should be taken by the nurse who is preparing to administer the preoperative medications? Review the surgical consent with the client • Explain that vomiting can occur during surgery • Remove the food from the client • Withhold the preoperative medication 591. The nurse is developing a plan of care for a middle-aged woman who is diagnosed with type 2 diabetes mellitus (DM). To lower her blood glucose and increase her serum high-density lipoprotein (HDL) levels, which instruction is most important for the nurse to provide? • Exercise at least three times weekly • Monitor blood glucose levels daily • Limit intake of foods high in saturated fat • Learn to read all food product labels 592. A client who has been in active labor for 12 hours suddenly tells the nurse that she has a strong urge to have a bowel movement. What action should the nurse take? • Allow the client to use a bedpan. • Assist the client to the bathroom • Perform a sterile vaginal exam • Explain the fetal head is descending. • Rationale: When a client in active labor suddenly expresses the urge to have a bowel movement, a sterile vaginal exam should be performed to determine if the fetus is descending. 593. The nurse assesses a 78-year-old male client who has left sides heart failure. Which symptoms would the nurse expect this client to exhibit? • Dyspnea, cough, and fatigue. • Hepatomegaly and distended neck veins • Pain over the pericardium and friction rub. • Narrowing pulse pressure and distant heart sounds. 594. A female client comes to the clinic complaining of fatigue and inability to sleep because she is the full-time caretaker for 22-year-old son who was paralyzed by a motor vehicle collision. She adds that her husband left her because he says he can’t take her behavior any more since all she does is care for their son. What intervention should the nurse implement? • Schedule a home visit in the afternoon to assess the son and client role as caregiver. • Acknowledge the client’s stress and suggest that she consider respite care. • Provide feedback to the client about her atonement for guilt about her son’s impairment. • Teach the client to problem-solve for herself and establish her own priorities. • Rationale: When this amount of disclosure is offered, the client is usually seeking information focuses on the client’s expression of worry, concern and stress and addresses the client’s need to initiate a request for assistance with respite care. 595. The nurse plans to administer a schedule dose of metoprolol (Toprol SR) at 0900 to a client with hypertension. At 0800, the nurse notes that client’s telemetry pattern shows a second degree heart block with a ventricular rate of 50. What action should the nurse take? • Administer the Tropol immediately and monitor the client until the heart rate increases. • Provide the dose of Tropol as scheduled and assign a UAP to monitor the client’s BP q30 minutes. • Give the Tropol as scheduled if the client’s systolic blood pressure reading is greater than 180. • Hold the scheduled dose of Tropol and notify the healthcare provider of the telemetry pattern. • Rationale: Beta blockers such as metoprolol (Tropol SR) are contraindicated in clients with second or third degree heart block because they decrease the heart rate. Therefore, the nurse should hold the medication. 596. A client who developed syndrome of inappropriate antidiuretic hormone (SIADH) associated with small carcinoma of the lung is preparing for discharge. When teaching the client about self-management with demeclocycline (Declomycin), the nurse should instruct the client to report which condition to the health care provider? • Insomnia • Muscle cramping • Increase appetite • Anxiety. Rationale: SIADH causes dilution hyponatremia because of the increased release of ADH, which is treated with water restriction and demeclocycline, a tetracycline derivate that blocks the action of ADH. Signs of hyponatremia (normal 136-145), which indicate the need for increasing the dosage of demeclocycline, should be reported to the healthcare provider. The signs include: plasma sodium level less than 120, anorexia, nausea, weight changes related to fluid disturbance, headache, weakness, fatigue, and muscle cramping. AC& D are not related to hyponatremia. 597. In determine the client position for insertion of an indwelling urinary catheter, it is most important for the nurse to recognize which client condition? • High urinary PH • Abdominal Ascites • Orthopnea • Fever. • Rationale: If the client is orthopneic, the nurse needs to adapt the insertion position that does not place the client in a supine position (the head of the bed should be elevated as much as possible). 598. The nurse is reviewing a client’s electrocardiogram and determines the PR interval (PRI) is prolonged. What does this finding indicate? • Initiation of the impulses from a location outside the SA node • Inability of the SA node to initiate an impulse at the normal rate • Increased conduction time from the SA node to the AV junction • Interference with the conduction through one or both ventricles. • Rationale: A prolonged PRI reflects an increased amount of time for an impulse to travel from the SA node through the AV node and is characteristic of a first degree heart block. 599. The nurse is teaching a male client with multiple sclerosis how to empty his bladder using the Crede Method. When performing a return demonstration, the client applies pressure to the umbilical areas of his abdomen. What instruction should the nurse provide? • Stroke the inner thigh below the perineum to initiate urinary flow • Contract, hold, and then relax the pubococcygeal muscle • Pour warm water over the external sphincter at the distal glans Apply downward manual pressure at the suprapubic regions. • Rationale: The Crede Method is used for those clients with atonic bladders, which is a concomitant of demyelinating disorders like multiple sclerosis. The client is applying pressure in the wrong region (umbilical Are) and should be instructed to apply pressure at the suprapubic are. 600. A 35 years old female client has just been admitted to the post anesthesia recovery unit following a partial thyroidectomy. Which statement reflects the nurse’s accurate understanding of the expected outcome for the client following this surgery? • Supplemental hormonal therapy will probably be unnecessary • The thyroid will regenerate to a normal size within a few years. • The client will be restricted from eating seafood • The remainder of the thyroid will be removed at a later date. 601. A client with gestational diabetes, at 39 weeks of gestation, is in the second stage of labor. After delivering of the fetal head, the nurse recognizes that shoulder dystocia is occurring. What intervention should the nurse implement first? • Prepare the client for an emergency cesarean birth • Encourage the client to move to a hands-and-knees position. • Assist the client to sharply flex her thighs up again the abdomen. • Lower the head of the bed an apply suprapubic pressure. • Rationale: Flexing the client’s thighs against the abdomen (Mc Robert’s maneuver) changes the angle o the pelvis and increase the pelvic diameter, making more room for the shoulders to emerge. ABD are implemented after C 602. The nurse should observe most closely for drug toxicity when a client receives a medication that has which characteristic? • Low bioavailability • Rapid onset of action • Short half life • Narrow therapeutic index. • Rationale: A drug with a narrow therapeutic index has a high risk for toxicity because there is a narrow range between the therapeutic dose and the toxic dose. 603. Following insertion of a LeVeen shunt in a client with cirrhosis of the liver, which assessment finding indicates to the nurse that the shunt is effective? • Decrease abdominal girth • Increased blood pressure • Clear breath sounds • Decrease serum albumin. 604. When finding a client sitting on the floor, the nurse calls for help from the unlicensed assistive personnel (UAP). Which task should the nurse ask the UAP to do? • Check for any abrasions or bruises. • Help the client to stand. • Get a blood pressure cuff. • Report the fall to the nurse-manager. 605. During the initial newborn assessment, the nurse finds that a newborn's heart rate is irregular. Which intervention should the nurse implement? • Notify the pediatrician immediately. • Teach the parents about congenital heart defects. • Document the finding in the infant's record. • Apply oxygen per nasal cannula at 3 L/min. 606. Which assessment finding indicates to the nurse a client’s readiness for pulmonary function tests? • Expresses an understanding of the procedure. 607. A young adult male is admitted to the emergency department with diabetic ketoacidosis (DKA). His pH is 7.25, HCO3 is 12 mEq/L or 12 mmol/L (SI), and blood glucose is 310 mg/dl or 17.2 mmol/L (SI). Which action should the nurse implement? • Infuse sodium chloride 0.9% (normal saline) 608. The nurse is assessing the thorax and lungs of a client who is having respiratory difficulty. Which finding is most indicative of respiratory distress? Contractions of the sternocleidomastoid muscle 609. After receiving lactulose, a client with hepatic encephalopathy has several loose stools. What action should the nurse implement? • Monitor mental status. 610. A client present at the clinic with blepharitis. What instructions should the nurse provide for home care? • Apply warm moist compresses then gently scrub eyelids with dilute baby shampoo 611. Dopamine protocol is prescribed for a male client who weigh 198 pounds to maintain the mean arterial pressure (MAP) greater than 65 mmHg. His current MAP is 50 mmHg, so the nurse increases the infusion to 7 mcg/kg/minute. The infusion is labeled dextrose 5% in water (D5W) 500 ml with dopamine 400 mg. The nurse should program the infusion pump to deliver how many ml/hour? • 47 612. The nurse is teaching a client with atrial fibrillation about a newly prescribed medication, dronedarone. Which information should the nurse include in client interactions? (Select all that apply) • Avoid eating grapefruit or drinking grapefruit juice. • Report changes in the use of daily supplements • Notify you heal care provider if your skin looks yellow 613. A male client recently released from a correctional facility arrives at the clinic with a cough, fever, and chills. His history reveals active tuberculosis (TB) 10 years ago. What action should the nurse implement? (Select all that apply) • Schedule the client for the chest radiograph • Obtain sputum for acid fast bacillus (AFB) testing • Place a mask on the client until he is moved to isolation. 614. A 16-year-old male is admitted to the pediatric intensive care unit after being involved in a house fire. He has full thickness burns to his lower torso and extremities. Before a dressing change to his legs, which intervention is most important for the nurse to implement? • Maintain strict aseptic technique. 615. While performing a skin inspection for a female adult client, the nurse observes a rash that is well circumscribed, has silvery scales and plaques, and is located on the elbows and knees. These assessment findings are likely to indicate which condition? • Tinea corporis • Herpes zoster • Psoriasis • Drug reaction 616. A client with acute pancreatitis is complaining of pain and nausea. Which interventions should the nurse implement (Select all that apply) • Monitor heart, lung, and kidney function. • Notify healthcare provider of serum amylase and lipase levels. • Review client’s abdominal ultrasound findings. • Position client on abdomen to provide organ stability • Encourage an increased intake of clear oral fluids 617. A nurse is caring for a client with Diabetes Insipidus. Which assessment finding warrants immediate intervention by the nurse? • Hypernatremia • Excessive thirst • Elevated heart rate • Poor skin turgor 618. In caring for a client receiving the amino glycoside antibiotic gentamicin, it is most important for the nurse to monitor which diagnostic test? • Serum creatinine 619. The nurse weighs a 6-month-old infant during a well-baby check-up and determines that the baby’s weight has tripled compared to the birth weight of 7 pounds 8 ounces. The mother asks if the baby is gaining enough weight. What response should the nurse offer? • What food does your baby usually eat in a normal day? • What was the baby’s weight at the last well-baby clinic visit? The baby is below the normal percentile for weight gain • Your baby is gaining weight right on schedule 620. A client who is at 36 weeks gestations is admitted with severe preclampsia. After a 6 gram loading dose of magnesium sulfate is administered, an intravenous infusion of magnesium sulfate at a rate of 2 grams/hour is initiated. Which assessment finding warrants immediate intervention by the nurse? 621. Urine output 20 ml/hour 622. What is the nurse’s priority goal when providing care for a 2-year-old child experience… • Stop the seizure activity • Decrease the temperature • Manage the airway • Protect the body from injury 623. The nurse is preparing to discharge an older adult female client who is at risk for hy…nurse include with this client’s discharge teaching? • Report any muscle twitching or seizures • Take vitamin D with calcium daily • Low fat yogurt is a good source of calcium • Keep a diet record to monitor calcium intake • Avoid seafood, particularly selfish 624. The husband of a client with advanced ovarian cancer wants his wife to have every treatment available. When the husband leaves, the client tells the nurse that she has had enough chemotherapy and wants to stop all treatments but knows her husband will sign the consent form for more treatment. The nurse’s response should include which information? • The husband cannot sign the consent for the client, her signature is required • The client’s specific wishes should be discussed with her healthcare provider • The healthcare team will formulate a plan of care to keep the client comfortable 625. The nurse is preparing a 50 ml dose of 50% dextrose IV for a client with insulin…medication? Push the undiluted Dextrose slowly through the currently infusion IV 626. The daughter of an older female client tells the clinic nurse that she is no longer able to care for her mother since her mother has lost the ability to perform activities of daily living (ADLs) due to aging. Which options should the nurse discuss with the daughter? • Home hospice agency • Long-term care facility • Rehabilitation facility • Independent senior apartment • Home health agency 627. A male client with cancer, who is receiving antineoplastic drugs, is admitted to the…what findings is most often manifest this condition? • Ecchymosis and hematemesis • Weight loss and alopecia • Weakness and activity intolerance • Sore throat and fever 628. A 7-year-old boy is brought to the clinic because of facial edema. He reports that he has been voiding small amounts of dark, cloudy, tea-colored urine. The parents state that their son had a sore throat 2 weeks earlier, but it has resolved. After assessing the child’s vital signs and weight, what intervention should the nurse implement next? • Measure the child’s abdominal girth • Perform an ostoscopic examination • Collect a urine specimen for routine urinalysis • Obtain a blood specimen for serum electrolytes 629. The nurse observes an adolescent client prepare to administer a prescribed corticosteroid medication using a metered dose inhaler as seen in the picture. What action should the nurse take? • Remind the client to hold his breath after inhaling the medication • Confirm that the client has correctly shaken the inhaler Affirm that the client has correctly positioned the inhaler • Ask the client if he has a spacer to use for this medication • 630. The nurse teaches an adolescent male client how to use a metered dose inhaler. Seen in the picture. What instruction should the nurse provide? • Move the device one to two inches away from the mouth • Secure the mouthpiece under the tongue • Press down on the device after breathing in fully • Breathe out slowly and deeply while compressing the device 631. A 3-year-old boy with a congenital heart defect is brought to the clinic by his mother…During the assessment, the mother asks the nurse why her child is at the 5th percent…response is best for the nurse to provide? • Does your child seem mentally slower than his peers also? • “His smaller size is probably due to the heart disease” • Haven’t you been feeding him according to recommended daily allowances for children? • You should not worry about the growth tables. They are only averages for children 632. A client with hypertension receives a prescription for enalapril, an angiotensin…instruction should the nurse include in the medication teaching plan? • Increase intake of potassium-rich foods • Report increased bruising of bleeding • Stop medication if a cough develops • Limit intake of leafy green vegetables 633. When administering ceftriaxone sodium (Rocephin) intravenously to a client before…most immediate intervention by the nurse? • Stridor • Nausea • Headache • Pruritis 634. The nurse is assessing a client with a small bowel obstruction who was hospitalized 24 hours ago. Which assessment finding should the nurse report immediately to the healthcare provider? • Rebound tenderness in the upper quadrants • Hypoactive bowel sounds in the lower quadrants • Tympany with percussion of the abdomen • Light colored gastric aspirate via the nasogastric tube 635. An adult female client is admitted to the psychiatric unit because of a complex handwashing ritual she performs daily that takes two hours or longer to complete. She worries about staying clean and refuses to sit on any of the chairs in the day area. This client’s handwashing is an example of which clinical behavior? • Addiction • Phobia • Compulsion • Obsession 636. A female client reports that she drank a liter of a solution to cleanse her intestines…immediately. How many ml of fluid intake should the nurse document? Whole number • 760 • Rationale: 1L=1000ml • Subtract the emesis, 1 cup (8 oz)=240ml • 1000-240=760 ml 637. Following routine diagnostic test, a client who is symptom-free is diagnosed with Paget’s disease. Client teaching should be directed toward what important goal for this client? • Maintain adequate cardiac output • Promote adequate tissue perfusion • Promote rest and sleep • Reduce the risk for injury 638. The mother of a one-month-old boy born at home brings the infant to his first well…was born two weeks after his due date, and that he is a “good, quiet baby” who almost…hypothyroidism, what question is most important for the nurse to ask the mother? • Is your son sleepy and difficult to feed? 639. In preparing assignments for the shift, which client is best for the charge nurse to assign to a practical nurse (PN)? • An older client post-stroke who is aphasic with right-sided hemiplegia 640. Following a gun shot wound to the abdomen, a young adult male had an emergency bowel…Multiple blood products while in the operating room. His current blood pressure is 78/52…He is being mechanically ventilated, and his oxygen saturation is 87%. His laboratory values…Grams / dl (70 mmol / L SI), platelets 20,000 / mm 3 (20 x 10 9 / L (SI units), and white blood cells. Based on these assessments findings, which intervention, should the nurse implements first? • Transfuse packed red blood cells 641. After checking the fingerstick glucose at 1630, what action should the nurse implement? • Administer 8 units of insulin aspart SubQ 642. Progressive kyphoscoliosis leading to respiratory distress is evident in a client with muscul…Which finding warrants immediate intervention by the nurse? • Evidence of hypoventilation 643. An adult male who lives alone is brought to the Emergency Department by his daughter who is unresponsive. Initial assessment indicated that the client has minimal respiratory effort, and his pupils are fixed and dilated. At the daughter’s request, the client is intubated and…Which nursing intervention has the highest priority? • Determine if the client has an executed living will 644. The nurse determines that a client’s pupils constricts as they change focus from a far object. What documentation should the nurse enter about this finding? • Pupils reactive to accommodation 645. Which nursing intervention has the highest priority for a multigravida who delivered… • Assess fundal tone and lochia flow 646. A client who had a gestational trophoblastic disease (GTD) evacuated 2 days ago is being…18 months-old child and lives in a rural area. Her husband takes the family car to work daily…transportation during the day. What intervention is most important for the nurse to implement? • Schedule a weekly home visit to draw hCG values. 647. A newly graduated female staff nurse approaches the nurse manager and request reassignment to another client because a male client is asking her for a date and making suggestive comments. Which response is best for the nurse manager to provide? • I’ll change your assignment, but let’s talk about you a nurse should respond to this kind of client. 648. After removing a left femoral arterial sheath, which assessment finding warrant immediately interventions by the nurse? (Select all that applied.) • Unrelieved back and flank pain. • Quarter-size red drainage at site • Cool and pale left leg and foot. • Tenderness over insertion site • Left groin egg-size hematoma. 649. Which instruction is most important for the nurse to provide a client who receives a new plan of care to treat osteoporosis? • Remain upright after taking the medication. 650. A newly hired home health care nurse is planning the initial visit to an adult client who has had multiple sclerosis (MS) for the past 20 years and is currently bed-bound and is lifted by a hoist. And unlicensed caregiver provides care 8 hours/ daily, 5 days/week. During the initial visit to this client, which intervention is most important to the nurse to implement? • Determine how the client is cared for when caregiver is not present. 651. A client with urticaria due to an environmental allergies is taking diphenhydramine... Which complaint should the nurse identify to the client as a side effect of the OTC medication? • Nausea and indigestion. • Hypersalivation • Eyelid and facial twitching • Increased appetite 652. In caring for a client with a PCA infusion of morphine sulfate through the right cephalic vein, The nurse assesses that the client in lethargic with a blood pressure of 90/60, pulse rate of 118 beats per minute, and respiratory rate of 8 breaths per minutes. What assessment should the nurse perform next? • Note the appearance and patency of the client’s peripheral IV site. • Palpate the volume of the client’s right radial pulse • Auscultate the client’s breath sounds bilaterally. • Observe the amount and dose of morphine in the PCA pump syringe. 653. A male client is having abdominal pain after a left femoral angioplasty and stent, and is asking for additional pain medication for right lower quadrant pain (9/10), two hours ago, he received hydrocodone / acetaminophen 7.5/7.50 mg his vital signs are elevated from reading of a previous hour: temperature 97.8 F, heart rate 102 beats / minute, respiration 20 breaths/minutes. His abdomen is swollen, the groin access site is tender, peripheral pulses are present, but left is greater than right. Preoperatively, clopidrogel was prescribed for a history of previous peripheral stents. Another nurse is holding manual pressure on the femoral arterial access site which may be leaking into the abdomen. What data is needed to make this report complete? • Surgeon needs to see client immediately to evaluate the situation 654. Which instruction is most important for the nurse to provide a client who is being discharge following treatment for Guillain-Barre syndrome? • Avoid exposure to respiratory infections • Use relaxation exercises when anxious • Plan short, frequent rest periods • Continue physical therapy at home 655. The nurse assesses a female client with obstructive sleep apnea syndrome (OSAS) who is 5 feet tall (152 cm) and weighs 155 pounds (70 kg), the client’s 24 hour diet history includes: no breakfast, cheeseburger and fries for lunch; lasagna, chocolate ice cream and a cola drink for dinner, and 2 glasses of wine in the evening before going to bed for a total caloric intake of 3500 calories. What instructions should the nurse provide? (Select all that apply) • Maintain current caloric intake • Avoid use of alcohol as a sleep aide at bedtime • Reduce intake of dairy products • Start a weight loss program • Set a goal of increasing BMI (Body Mass Index) 656. A male client with impaired renal function who takes ibuprofen daily for chronic arthritis…gastrointestinal (GI) bleeding. After administering IV fluids and a blood transfusion, his blood pressure is 100/70, and his renal output is 20 ml / hour. Which intervention should the nurse include in hours? • Evaluate daily serial renal laboratory studies for progressive elevations 657. The health care provider prescribes atenolol 50 mg daily for a client with angina pectoris…to the health care provider before administering this medication? • Irregular pulse • Tachycardia • Chest pain • Urinary frequency 658. When obtaining a rectal temperature with an electronic thermometer, which action is most important for the nurse to perform? • Hold the thermometer in place. 659. An adult female client is admitted to the psychiatric unit with a diagnosis of major depressive…medication therapy, the nurse notices the client has more energy, is giving her belongings…mood. Which intervention is best for the nurse to implement? • Ask the client if she has had any recent thoughts of harming herself 660. An adult female client with chronic kidney disease (CKD) asks the nurse if she can continue…Medications. Which medication provides the greatest threat to this client? Magnesium hydroxide (Maalox) 661. The nurse observes an unlicensed assistive personnel (UAP) using an alcohol-based clean…tray to the room. The UAP rub both hands thoroughly for 2 minutes while standing at the…should the nurse take? • Explain that the hand rub can be completed in less than 2 minutes 662. An adolescent’s mother calls the clinic because the teen is having recurrent vomiting and…Combative in the last 2 days. The mother states that the teen takes vitamins, calcium, mag…With aspirin. Which nursing intervention has highest priority? • Instruct the mother to take the teen to the emergency room 663. A male Korean-American client looks away when asked by the nurse to describe his problem. What is the best initial nursing action? • Allow several minutes for the client to respond • Ask social services to find a Korean interpreter • Repeat the question slowly and distinctly • Establish direct eye contact with the client 664. An older female client tells the nurse that her muscles have gradually been getting weak…what is the best initial response by the nurse? • Ask the client to describe the changes that have occurred 665. When organizing home visits for the day, which older client should the home health nurse plan to visit first? • A woman who takes naproxen (Naprosyn) and reports a recent onset of dark, tarry stools 666. A client is admitted for type 2 diabetes mellitus (DM) and chronic Kidney disease (CKD)…which breakfast selection by the client indicates effective learning? • Oatmeal with butter, artificial sweetener, and strawberries, and 6 ounces coffee 667. A client with a postoperative wound that eviscerated yesterday has an elevated temperature…most important for the nurse to implement? • Obtain a wound swab for culture and sensitivity 668. The nurse is reinforcing home care instructions with a client who is being discharged following…prostate (TURP). Which intervention is most important for the nurse to include in the client… • Report fresh blood in the urine 669. The nurse provides feeding tube instructions to the wife of a client with end stage cancer. The client’s wife performs a return demonstration correctly, but begins crying and tells the nurse, “I just don’t think I can do this every day.” The nurse should direct further teaching strategies toward which learning domain? • Cognitive • Affective • Comprehension • Psychomotor 670. A male client with rheumatoid arthritis is schedule for a procedure in the morning. The…unable to complete the procedure because of early morning stiffness. Which intervention…implement? • Assign a UAP to assist the client with a warm shower early in the morning 671. The nurse is caring for a client following a myelogram. Which assessment finding should the nurse report to the healthcare provider immediately? • Complain of headaches and stiff neck 672. A woman who takes pyridostigmine for myasthenia gravis (MG) arrives at the emergency department complaining of extreme muscle weakness. Her adult daughter tells the nurse that since yesterday her mother has been unable to smile, which assessment finding warrants immediate intervention by the nurse? • Uncontrollable drooling • Inability to raise voice • Tingling of extremities • Eyelid drooling 673. A client with multiple sclerosis (MS) is admitted to the medical unit. The client reports…which action should the nurse implement to reduce the client’s risk for falls? • Schedule frequent rest periods • Provide assistance to bedside commode Teach to patch one eye when ambulating 674. What is the nurse’s priority goal when providing care for a 2-year-old child experiencing seizure… • Stop the seizure activity • Decrease the temperature • Manage the airway • Protect the body from injury 675. A client is complaining of intermittent, left, lower abdominal pain that began two days ago…implement the following interventions? • Correct orders: (DPIA) 1. Determine when the client had last bowel movement 2. Position client supine with knees bent 3. Inspect abdominal contour 4. Auscultate all four abdominal quadrants 676. The nurse is caring for four clients…postoperative hemoglobin of 8.7 mg/dl; client C, newly admitted with potassium…an appendectomy who has a white blood cell count of 15,000mm3. What intervention… • Determine the availability of two units of packed cells in the blood bank for client B • Increase the oxygen flow rate to 4 liters/minute per face mask for client A • Remove any foods, such as banana or orange juice, for the breakfast tray for client C • Inform client D that surgery is likely to be delayed until the infection responds to antibiotics 677. A client with a new diagnosis of Raynaud’s disease lives alone. Which instruction should the nurse include in the client’s discharged teaching plan? • Keep room temperature 80 678. Sublingual nitroglycerin is administered to a male client with unstable angina who complains of crushing chest pain. Five minutes later the client becomes nauseated and his bloods pressure drops to 60/40. Which intervention should the nurse implement? • Infuse a rapid IV normal saline bolus 679. A male client tells the nurse that he is concerned that he may have a stomach ulcer, because he is experiencing heartburn and a dull growing pain that is relieved when he eats. What is the best response by the nurse? • Encourage the client to obtain a complete physical exam since these symptoms are consistent with an ulcer 680. A mother calls the nurse to report that at 0900 she administered a PO dose of digoxin to her 4-month-old infant, but at 0920 the baby vomited the medicine, what instruction should the nurse provide to this mother • Withhold this dose 681. When checking a third grader’s height and weight the school nurse notes that these measurements have not changed in the last year. The child is currently taking daily vitamins, albuterol, and methylphenidate for attention deficit hyperactivity disorder (ADHD). Which intervention should the nurse implement? • Refer child to the family healthcare provider 682. An adolescent receives a prescription for an injection of s-matriptan succinate 4 mg subcutaneously for a migraine headache. Using a vial labeled, 6 mg/ 0.5 ml, how many ml should the nurse administer? (Enter the numerical value only. If rounding is required, round to the nearest hundredth.) 0.33 mL • Rationale: 4mg x 0.5 ml=2/6=0.33 ml 683. An unlicensed assistive personnel (UAP) informs the nurse who is giving medications that a female client is crying. The client was just informed that she has a malignant tumor. What action should the nurse implement first? • Tell the client that the nurse will be back to talk to her after medications are given 684. The husband of an older woman, diagnosed with pernicious anemia, calls the clinic to report that his wife still has memory loss and some confusion since she received the first dose of nasal cyanocobalamin two days ago. He tells the nurse that he is worried that she might be getting Alzheimer’s disease. What action should the nurse take? • Explain that memory loss and confusion are common with vitamin B12 deficiency 685. While the school nurse is teaching a group of 14-year-olds, one of the participants remarks, “You are too young to be our teacher! You’re not much older than we are!” How should the nurse respond? “How old do you think I am?” • “We need to stay focused on the topic.” • “I think I am qualified to teach this group.” • “Do you think you can teach it any better?” 686. An unconscious client is admitted to the intensive care unit and is placed on a ventilator. The ventilator alarms continuously and the client's oxygen saturation level is 62%. What action should the nurse take first? • Begin manual ventilation immediately. 687. After diagnosis and initial treatment of a 3 year old with Cystic fibrosis, the nurse provides home care instructions to the mother, which statement by the child's mother indicates that she understands home care treatment to promote pulmonary functions? • Chest physiotherapy should be performed twice a day before a meal. 688. A middle-aged woman, diagnosed with Graves’ disease, asks the nurse about this condition. Which etiological pathology should the nurse include in the teaching plan about hyperthyroidism? (Select all that apply.) • Graves’ disease, an autoimmune condition, affects thyroid stimulating hormone receptors. • T3 and T4 hormone levels are increased • Large protruding eyeballs are a sign of hyperthyroid function • Weight gain is a common complaint in hyperthyroidism • Early treatment includes levothyroxine (Synthroid). 689. A male client who was admitted with an acute myocardial infarction receives a cardiac diet with sodium restriction and complains that his hamburger is flavorless. Which condiment should the nurse offer? • Fresh horseradish 690. While completing an admission assessment for a client with unstable angina, which closed questions should the nurse ask about the client's pain? • Does your pain occur when walking short distances? 691. A nurse who works in the nursery is attending the vaginal delivery of a term infant. What action should the nurse complete prior to leaving the delivery room? • place the id bands on the infant and mother 692. A female client with chronic urinary retention explains double voiding technique to the nurse by stating she voids partially, hold the remaining urine in her bladder for three minutes, then voids again to empty her bladder fully. How should the nurse respond? • Advise the client to empty her bladder fully when she first voids 693. A client is receiving an IV solution of nitroglycerin 100mg/500ml D5W at 10 mcg/ minute. The nurse should program the infusion pump to deliver how many ml/hour? ( Enter numeric value only) 3 ml/hour • Rationale : 0.01 x 500 x 60 / 100 = 3 694. When assessing a multigravida the first postpartum day, the nurse finds a moderate amount of lochia rubra, with the uterus firm, and three fingerbreadths above the umbilicus. What action should the nurse implement first? • Massage the uterus to decrease atony • Review the hemoglobin to determine hemorrhage • Increase intravenous infusion • Check for a distended bladder 695. A-12-years old boy has a body mass index (BMI) of 28, a systolic pressure and a glycosylated hemoglobin (HBA1C) of 7.8%. Which selection indicated that his mother understands the management of his diet? • One whole-wheat bagel with cream cheese, two strips of bacon, six ounces of orange juice. • Rationale: Diet - Foods high in carbohydrates and fiber, low fat. No honey, no ham, no high sugar, no frost food, avoid all whole wheat products. 696. Which class of drugs is the only source of a cure for septic shock? • Antiinfectives 697. A 59-year-old male client comes to the clinic and reports his concern over a lump that, “just popped up on my neck about a week ago.” In performing an examination of the lump, the nurse palpates a large, nontender, hardened left subclavian lymph node. There is not overlying tissue inflammation. What do these findings suggest? Malignancy • Bacterial infection • Viral infection • Lymphangitis 698. A gravida 2 para 1, at 38-weeks gestation, scheduled for a repeat cesarean section in one week, is brought to the labor and delivery unit complaining of contractions every 10 minutes. While assessing the client, the client’s mothers enter the labor suite and says in a loud voice, “I’ve had 8 children and I know she’s in labor. I want her to have her cesarean section right now!” what action should the nurse take? • Request the mother to leave the room • Tell the mother to stop speaking for the client • Request security to remove her from the room • Notify the charge nurse of the situation 699. While caring for a toddler receiving oxygen (02) via face mask, the nurse observes that the child’s lips and nares are dry and cracked. Which intervention should the nurse implement? • Ask the mother what she usually uses on the child’s lips and nose • Apply a petroleum jelly (Vaseline) to the child’s nose and lips • Use a topical lidocaine (Zylocaine viscous) analgesic for cracked lips • Use a water soluble lubricant on affected oral and nasal mucosa 700. The healthcare provider prescribes carboprost tromethamine (Hemabate) 250 mcg IM for a multigravida postpartum client who is experiencing heavy, bright red vaginal bleeding. Prior to administering this medication, which interventions should the RN implement? • Obtain a second IV access. • Decrease the room temperature. • Give the prescribed antiemetic. • Insert an indwelling catheter. 701. During the infusion of a second unit of packed red blood cells, the client’s temperature increases from 99 to 101.6 f. which intervention should the nurse implement? Stop the transfusion start a saline • Observe for a maculopapular rash • Report the fever to the blood bank • Give a PRN dose of acetaminophen 702. An elderly female client with osteoarthritis reports increasing pain and stiffness in her right knee and asks how to reduce these symptoms. In responding to the client, the nurse recognizes what pathology as the cause of her symptoms? • Destruction of joint cartilage. 703. When caring for a client with traumatic brain injury (TBI) who had a craniotomy for increased intracranial pressure (ICP), the nurse assesses the client using the Glasgow coma scale (GCS) every two hours. For the past 8 hours the client’s GCS score has been 14. What does this GCS finding indicate about the client? • Neurologically stable without indications of an increased ICP 704. A 46-year-old male client who had a myocardial infarction 24-hours ago comes to the nurse’s station fully dressed and wanting to go home. He tells the nurse that he is feeling much better at this time. Based on this behavior, which nursing problem should the nurse formulate? • Ineffective coping related to denial 705. In assessing a client 48 hours following a fracture, the nurse observes ecchymosis at the fracture site, and recognizes that hematoma formation at the bone fragment site has occurred. What action should the nurse implement? • Document the extend of the bruising in the medical record 706. A client is admitted for cellulitis surrounding an insect bite on the lower, right arm and intravenous (IV) antibiotic therapy is prescribed. Which action should the nurse implement before performing venipuncture? • Lower the left arm below the level of the heart 707. Which assessment finding of a postmenopausal woman necessitates a referral by the nurse to the healthcare provider for evaluation of thyroid functioning? • Cold sensitivity 708. A client with hyperthyroidism is admitted to the postoperative after subtotal thyroidectomy. Which of the client’s serum laboratory values requires intervention by the nurse? Total calcium 5.0 mg/dl 709. A female client on the mental health unit frequently asks the nurse when she can be discharged. Then, becoming more anxious, she begins to pace the hallway. What intervention should the nurse implement first? • Explore the client’s reasons for wanting to be discharged. 710. The nurse is assessing a primigravida a 39-weeks gestation during a weekly prenatal visit. Which finding is most important for the nurse to report to the healthcare provider? • Fetal heart rate of 200 beats/minute 711. A female client receives a prescription for alendronate sodium (Fosamax) to treat her newly diagnose osteoporosis. What instruction should the nurse include in the client’s teaching plan? • Take on an empty stomach with a full glass of water 712. The nurse is assessing a female client’s blood pressure because she reported feeling dizzy. The blood pressure cuff is inflated to 140 mm hg and as soon as the cuff is deflated a korotkoff sound is heard. Which intervention should the nurse implement next? • Wait 1 minute and palpate the systolic pressure before auscultating again. 713. After administering a proton pump inhibitor (PPI), which action should the nurse take to evaluate the effectiveness of the medication? • Ask the client about gastrointestinal pain 714. To reduce staff nurse role ambiguity, which strategy should the nurse-manager implement? • Review the staff nurse job description to ensure that it is clear, accurate, and current 715. A client with pneumonia has arterial blood gases levels at: PH 7.33; PaCO2 49 mm/hg; HCO3 25 mEq/L; PaO2 95. What intervention should the nurse implement based on these results? • Institute coughing and deep breathing protocols 716. The healthcare provider explains through an interpreter the risks and benefits of a scheduled surgical procedure to a non-English speaking female client. The client gives verbal consent and the healthcare provider leaves, instructing the nurse to witness the signature on the consent form. The client and the interpreter then speak together in the foreign language for an additional 2 minutes until the interpreter concludes, “She says it is OK.” What action should the nurse take next? Ask for a full explanation from the interpreter of the witnessed discussion 717. While assisting a male client who has muscular dystrophy (MD) to the bathroom, the nurse observes that he is awkward and clumsy. When he expresses his frustration and complains of hip discomfort, which intervention should the nurse implement? • Place a portable toilet next to the bed 718. A client with hyperthyroidism who has not been responsive to medications is admitted for evaluation. What action should the nurse implement? (Click on each chart tab for additional information. Please scroll to the bottom right corner of each tab to view all information contained in the client’s medical record.) • Notify the healthcare provider 719. While taking vital signs, a critically ill male client grabs the nurse’s hand and ask the nurse not to leave. What action is best for the nurse to take? • Pull up a chair and sit beside the client’s bed 720. The practical nurse (PN) is assigned to work with three registered nurses (RN) who are caring for neurologically compromised clients. The client with which change in status is best to assign to the PN? • Viral meningitis whose temperature changed from 101 F to 102 F. 721. An IV antibiotic is prescribed for a client with a postoperative infection. The medication is to be administered in 4 divided doses. What schedule is best for administering this prescription? • 1000, 1600, 2200, 0400 722. A male client notifies the nurse that he feels short of breath and has chest pressure radiating down his left arm. A STAT 12-lead electrocardiogram (ECG) is obtained and shows ST segment elevation in leads II, II, aVF and V4R. The nurse collects blood samples and gives a normal saline bolus. What action is most important for the nurse to implement? • Asses for contraindications for thrombolytic therapy 723. A client with Addison’s crisis is admitted for treatment with adrenal cortical supplementation. Based on the client’s admitting diagnosis, which findings require immediate action by the nurse? (Select all that apply) • Headache and tremors • Postural hypotension Pallor and diaphoresis • Irregular heart beat 724. A client with rapid respirations and audible rhonchi is admitted to the intensive care unit because of a pulmonary embolism (PE). Low-flow oxygen by nasal cannula and weight based heparin protocol is initiated. Which intervention is most important for the nurse to include in this client’s plan of care? • Evaluate daily blood clotting factors. 725. The nurse enters a client’s room to administer scheduled daily medications and observes the client leaning forward and using pursed lip breathing. Which action is most important for the nurse to implement first? • Evaluate the oxygen saturation 726. During a clinic visit, a client with a kidney transplant ask, “What will happen if chronic rejection develops?” which response is best for the nurse to provide? • Dialysis would need to be resumed if chronic rejection becomes a reality 727. The nurse enters a client’s room and observe the unlicensed assistive personnel (UAP) making an occupied bed as seen in the picture. What action should the nurse take first? • Place the side rails in an up position 728. A client is receiving continuous bladder irrigation via a triple-lumen suprapubic catheter that was placed during prostatectomy. Which report by the unlicensed assistive personnel (UAP) requires intervention by the nurse? • Leakage around catheter insertion site 729. A client with bleeding esophageal varices receives vasopressin (Pitressin) IV. What should the nurse monitor for during the IV infusion of this medication? • Chest pain and dysrhythmia 730. A male client with cancer who has lost 10 pounds during the last months tells the nurse that beef, chicken, and eggs, which used to be his favorite foods, now they taste “bitter”. He complains that he simply has no appetite. What action should the nurse implement? • Suggest the use of alternative sources of protein such as dairy products and nuts 731. A nurse plans to call the healthcare provider to report an 0600 serum potassium level of 2 mEq/L or mmol/L (SI), but the charge nurse tells the nurse that the healthcare provider does not like to receive early morning calls and will make rounds later in the morning. What action should the nurse make? • Contact the healthcare provider immediately to report the laboratory value regardless of the advice 732. Which actions should the nurse implement with auscultating anterior breath sounds? (Place the first action on top and last action on the bottom.) 1. Place stethoscope in suprasternal area to auscultate from bronchial sounds 2. Auscultate bronchovesicular sounds from side to side of the first and second intercostal spaces 3. Displace female breast tissue and apply stethoscope directly on chest wall to hear vesicular sounds 4. Document normal breath sounds and location of adventitious breath sounds 733. The nurse is preparing a community education program on osteoporosis. Which instruction is helpful in preventing bone loss and promoting bone formation? • Recommend weigh bearing physical activity 2. The husband of an older woman, diagnosed with pernicious anemia, calls the clinic to report that his wife still has memory loss and some confusion since she received the first dose of nasal cyanocobalamin two days ago. He tells the nurse that he is worried that she might be getting Alzheimer’s disease. What action should the nurse take? Explain that memory loss and confusion are common with vitamin B12 deficiency 3. A female client who is admitted to the mental health unit for opiate dependency is receiving clonidine 0.1 mg PO for withdrawal symptoms. The client begins to complain of feeling nervous and tells the nurse that her bones are itching. Which finding should the nurse identify as a contraindication for administering the medication? Blood pressure 90/76 mm Hg 4. During discharge teaching, an overweight client heart failure (HF) is asked to make a grocery list for the nurse to review. Which food choices included on the client’s list should the nurse encourage? (Select all that apply) A. Canned fruit in heavy syrup. B. Plain, air-popped popcorn. C. Cheddar cheese cubes. D. Natural whole almonds. E. Lightly salted potato chips 5. A client with Addison’s disease becomes weak, confused, and dehydrated following the onset of an acute viral infection. The client’s laboratory values include; sodium 129 mEq/l (129mmol/l SI), glucose 54 mg/dl (2.97mmol/l SI) and potassium 5.3 mmol/l SI). When reporting the findings to the HCP, the nurse anticipates a prescription for which intravenous medications? A. Regular insulin. B. Hydrocortisone C. Broad spectrum antibiotic D. Potassium chloride 6. An adolescent, whose mother recently died, comes to the school nurse complain headache. Which statement made by the students should warrant further explanation nurse? A. “I’ve had dreams about Mon since she died.” B. “I’ve been very sad and cry a lot at night.” C. “I miss Mon and would like to go see her’”. D. “ it’s hard to concentrate on my homework” 7. When washing soiled hands, the nurse first wets the hands and applies soap. The nurse should complete additional actions in which sequence? (Arrange from first action on top last action on bottom.) 1. Rub hands palm to palm. 2. Interlace the fingers, 3. Dry hands with paper towel. 4. Turn off the water faucet. 8. An Unna boot is applied to a client with a venous stasis ulcer. One week later, when the Unna boot is removed during a follow-up appointment, the nurse observes that the ulcer site contains bright red tissue. What action should the nurse take in response to this finding? Document the ongoing wound healing. 9. The nurse is caring for four clients who are on the rehabilitation unit, which client should the nurse assess first? A. A client with an above-the-knee amputation who is complaining of phantorn pain. B. A client who is receiving a continuous tube feeding and is now vomiting. C. A client with left hemiplegia who is scheduled for hemodialysis today. D. A client with pneumonia who is scheduled for pulmonary function studies. 10. A client’s telemetry monitor indicates ventricular fibrillation (VF). After delivering one counter shock, the nurse resumes chest compression, after another minute of compression , the client’s rhythm converts to supraventricular tachycardia (SVT) on the monitor, at this point , what is the priority intervention for the nurse? A. Prepare for transcutaneous pacing B. Administer IV epinephrine per ACLS protocol. C. Give IV dose of adenosine rapidly over 1-2 seconds. D. Deliver another defibrillator shock. 19. A client with a history of using illicit drugs intravenously is admitted with Kaposi’s sarcoma. Which intervention should the nurse include in this client’s admission plan of care? A. Identify local support HIV support groups. B. Assess for symptoms of AIDS dementia. C. Observe for adverse drug reaction. D. Monitor for secondary infections. 20. After an elderly female client receives treatment for drug toxicity, the HCP prescribes a 24- hour creatinine clearance test. Prior to starting the urine collection, the nurse notes that the client’s serum creatinine is 0.3mg/dl. What action should the nurse implement? A. Initiate the urine collection as prescribed. B. Notify the HCP of the results. C. Evaluate the client’s serum BUN level. D. Assess the client for signs of hypokalemia. 21. Immediately after extubation, a client who has been mechanically ventilated is placed on a 50% non-rebreather. The client is hoarse and complaining of a sore throat. Which assessment finding should the nurse report to the healthcare provider immediately? A. Blood tinged sputum B. Expiratory wheezing C. Upper airway stridor D. Oxygen saturations 90% 22. The nurse is collecting sterile sample for culture and sensitivity from a disposable three chamber water-seal drainage system connected to a pleural chest tube. The nurse should obtain the sample from which site on the drainage system? A. Stopper port located above the water-seal level B. Plastic tubing located at the chest insertion site C. Rubberized port at the bottom of collection chamber D. Tubbing located on the top of the suction chamber 23. While the nurse is preparing a scheduled intravenous (IV) medication, the client states that the IV site hurts and refuses to allow the nurse to administer a flush to assess the site. Which intervention should the nurse implement? A. Apply ice first, then a warm compress to the IV site B. Discontinue the painful IV after a new IV is inserted C. Review the medical record for the date of insertion D. Document that the medication was not administered 24. During a staff meeting, a nurse verbally attacks the nurse manager conducting the meeting, stating, “you always let your favorites have holidays off give then easier assignments. You are unfair and prejudiced” how should the nurse-manager respond? A. I would prefer to discuss this with you privately. B. Give me specific examples to support your statements. C. Does anyone else on the staff fell the same way D. Your remarks are not true and are very unkind 25. An adult is admitted to the emergency department following ingestion of a bottle of antidepressants secondary to chronic paint. A nasogastric tube and a left subclavian venous catheter are placed. The nurse auscultates audible breath sounds on the right side, faint sounds procedure should the nurse prepare for first? A. Insertion of a left- sided chest tube. B. Placement of an endotracheal tube. C. Retraction of the nasogastric tube D. Setup of patient- controlled analgesia 26. A client is admitted to the hospital after experiencing a brain attack, commonly referred to as a stroke or cerebral vascular accident (CVA). The nurse should request a referral for speech therapy if the client exhibits which finding? Abnormal responses for cranial nerves I and II Persistent coughing while drinking Unilateral facial drooping Inappropriate or exaggerated mood swings 27. A male client is admitted with a severe asthma attack. For the last 3 hours he has experienced increased shortness of breath. His arterial blood gas results are: pH 7.22 PaCO2 55 mmHg; HCO3 25 mEq/L or mmol/L (SI). Which intervention should the nurse implement? A. Space care to provide periods of rest B. Instruct client to purse lip breathe C. Administer PRN dose of albuterol D. Position client for maximum comfort 28. A young adult female with chronic kidney disease (CKD) due to recurring pyelonephritis is hospitalized with basilar crackles and peripheral edema. She is complaining of severe nausea and the cardiac monitor indicates sinus tachycardia with frequent premature ventricular contraction. Her blood pressure is 200 /110 mm Hg, and her temperature is 101 F which PRN medication should the nurse administers first? A. Enalapril B. Furosemide C. Acetaminophen D. Promethazine 29. When entering a client’s room to administer an 0900 IV antibiotic, the nurse finds that the client is engaged in sexual activity with a visitor. Which actions should the nurse implement? A. Ignore the behavior and hang the IV antibiotic B. tell the client to stop the inappropriate behavior C. Leave the room and close the door quietly D. Complete an unusual occurrence report 30. The nurse is caring for a client who is experiencing a tonic-clonic seizure. Which actions should the nurse implement? (Select all that apply) Ease the client to the floor Loosen restrictive clothing Note the duration of the seizure 40. A child with heart failure (HF) is taking digitalis. Which signs indicates to the nurse that the child may be experiencing digitalis toxicity? Tachycarcia Dyspnea Vomiting Muscle cramps 41. An antacid is prescribed for a client with gastroesophageal (GERD). The client asks the nurse, “How does this help my GERD?” What is the best response by the nurse? A. This medication will coat the lining of your esophagus B. Antacids will neutralize the acid in your stomach C. It will improve the emptying of food through your stomach D. antacids decrease the production of gastric secretions 42. The nurse suspect may be hemorrhaging internally. Which findings of an orthostatic test may indicate to the nurse of major bleed? A decrease in the systolic b/p of 10mm/hg with a corresponding increase of heart rate of 20. 43. A male adult is admitted because of an acetaminophen overdose. After transfer to the mental health unit, the client is told he has liver damage. Which information is most important for the nurse to include in the client’s discharge plan? A. Avoid exposure to large crowds B. Do not take any over-the-counter medications C. Call the crisis hot line if feeling lonely D. Eat a high carbohydrate, low fat, low protein diet 44. A client arrives in the emergency center with a blood alcohol level of 500 mg/dl. When transferred to the observation unit, the client becomes demanding, aggressive, and shouts at the staff. Which assessments finding is most important for the nurse to identify in the first 24 hours? A. Decreased appetite B. Nausea and elevated blood pressure C. Difficulty walking D. Agitation and threats to harms staff 45. A male client who had a small bowel resection acquired methicillin- resistant Staphylococcus aureus (MRSA) while hospitalized. He was treated and released, but is readmitted today because of diarrhea and dehydration. It is most important for the nurse to implement which intervention? Maintain contact transmission precautions 46. The nurse applies a blood pressure cuff around a client’s left thigh. To measure the client’s blood pressure, where should the diaphragm of the stethoscope be placed? (Mark the loication on one of the images.) “On left thigh with arrow pointing to inner thigh” 47. To reduce the risk of symptoms exacerbation for a client with multiple sclerosis (MS), which instructions should the nurse include in the client’s discharge plan? (Select all that apply). A. Practice relaxation exercises B. Limit fluids to avoid bladder distention C. Space activities to allow for rest periods D. Avoid persons with infections E. Take warm baths before starting exercise 48. A preoperative client states he is not allergic to any medications. What is the most important nursing action for the nurse to implement next? A. Record “no known drug allergies” on preoperative checklist B. Assess client’s allergies to non-drug substances C. Assess client’s knowledge of an allergy response D. Flag “no known drug allergies” on the front of the chart 49. During a visit to the planned parenthood clinic, a young woman tells the nurse that she is going to discontinue taking the oral contraceptives she has taken for three years because she wants to get pregnant. History indicates that her grandfather has adult onset diabetes and that she was treated for chlamydia six months ago, which factor in this client’s history poses the greatest risk for this woman’s pregnancy? A. Family history of adult onset diabetes. B. Treatment for chlamydia in the past year C. Client’s age and previous sexual behavior D. Three year history of taking oral contraceptives 50. When conducting diet teaching for a client who was diagnosed with a myocardial infarction, which snack foods should the nurse encourage the client to eat? (Select all that apply). A. Fresh turkey slices and berries B. Fresh vegetables with mayonnaise dip C. Soda crackers and peanut butter D. Chicken bouillon soup and toast E. raw unsalted almonds and apples 61. An unlicensed assistive personnel (UAP) reports that a client’s right hand and fingers spasms when taking the blood pressure using the same arm. After confirming the presence of spams what action should the nurse take? A. Ask the UAP to take the blood pressure in the other arm B. Tell the UAP to use a different sphygmomanometer. C. Review the client’s serum calcium level D. Administer PRN antianxiety medication. 63. The nurse is caring for a 17-year-old male who fell 20 feet 5 months ago while climbing the side of a cliff and has been in a sustained vegetative state since the accident. Which intervention should the nurse implement? A. Inquire about food allergies and food likes and dislikes B. Talk directly to the adolescent while providing care C. Initiate open communication with the teen’s parents D. Monitor vital signs and neuro status every 2 hours 64. Following an open reduction of the tibia, the nurse notes bleeding on the client’s cast. Which action should the nurse implement? A. No action is required since postoperative bleeding can be expected B. Lower the client’s head while assessing for symptoms of shock C. Call the health care provider and prepare to take the client back to the operating room D. Outline the area with ink and check it every 15 minutes to see if the area has increased 65. While a child is hospitalized with acute glomerulonephritis, the parents ask why blood pressure readings are taken so often. Which response by the nurse is most accurate? A. Blood pressure fluctuations means that the condition has become chronic B. Elevated blood pressure must be anticipated and identified quickly C. Hypotension leading to sudden shock can develop at any time D. Sodium intake with meals and snacks affects the blood pressure 66. The mother of a child recently diagnosed with asthma asks the nurse how to help protect her child from having asthmatic attacks. To avoid triggers for asthmatic attacks, which instructions should the nurse provide the mother? (Select all that apply) Close car windows and use air conditioner Avoid sudden changes in temperature Keep away from pets with long hair Stay indoors when grass is being cut 67. Which client should the charge nurse on the oncology unit assign to an RN, rather than a practical nurse (PN)? An elderly female client with cancer whose children who are trying to decide whether to change to palliative care measures or continue disease control 68. An elderly male client is admitted to the urology unit with acute renal failure due to a postrenal obstruction. Which questions best assists the nurse in obtaining relevant historical data? A. “Have you had a heart attack in the last 6 months” B. “Have you had any difficulty in starting your urinary stream” C. “Have you taken any antibiotics recently” D. “Have you received any blood products in the last year” 69. A school nurse is called to the soccer field because a child has a nose bleed (epistaxis). In what position should the nurse place the child? Sitting up and leaning forward 70. A child is diagnosed with acquired aplastic anemia. The nurse knows that this child has the best prognosis with which treatment regimen? A. Bone marrow transplantation B. Blood transfusion C. Chemotherapy D. Immunosuppressive therapy 71. For the past 24 hours, an antidiarrheal agent, diphenoxylate, has been administered to a bedridden, older client with infectious gastroenteritis. Which finding requires the nurse to take further action? Tented skin turgor 72. An unconscious client is admitted to the intensive care unit and is placed on a ventilator. The ventilator alarms continuously and the client's oxygen saturation level is 62%. What action should the nurse take first? Begin manual ventilation immediately. 73. An elderly client seems confused and reports the onset of nausea, dysuria, and urgency with incontinence. Which action should the nurse implement? Obtain a clean catch mid-stream specimen 73. A client’s subjective data includes dysuria, urgency, and urinary frequency. What action should the nurse implement next? Collect a clean-catch specimen 74. A client is admitted with an exacerbation of heart failure secondary to COPD. Which observations by the nurse require immediate intervention to reduce the likelihood of harm to this client? (Select all that apply). A. A bedside commode is positioned near the bed B. A saline lock is present in the right forearm C. A full pitcher of water is on the bedside table D. The client is lying in a supine position in bed E. A low sodium diet tray was brought to the room 75. A client with a traumatic brain injury becomes progressively less responsive to stimuli. The client has a “Do Not Resuscitate” prescription, and the nurse observes that the unlicensed assistive personnel (UAP) has stopped turning the client from side to side as previously schedules. What action should the nurse take? A. Advise the UAP to resume positioning the client on schedule B. Encourage the UAP to provide comfort care measures only C. Assume total care of the client to monitor neurologic function D. Assign a practical nurse to assist the UAP in turning the client 76. The nurse reviews the laboratory findings of a client with an open fracture of the tibia. The white blood cell (WBC) count and erythrocyte sedimentation rate (ESR) are elevated. Before reporting this information to the healthcare provider, what assessment should the nurse obtain? A. Degree of skin elasticity B. Appearance of wound C. Bilateral pedal pulse force D. Onset of any bleeding 77. The HCP prescribes methotrexate 7.5 mg PO weekly, in 3 divides doses for a child with rheumatoid arthritis whose body surface area (BSA) is 0.6 m2. The therapeutic dosage of methotrexate PO is 5 to 15 mg/m2/week. How many mg should the nurse administer in each of the three doses given weekly? (Enter the numeric value only. If round is required, round to the nearest tenth.) ___1.5_______ 78. An alert older client with diabetes mellitus type 1 is admitted with a serum glucose of 420 mg/dl (23.31 mmol/L (SI)). As the nurse administers 10 units of regular insulin intravenous (IV), the client immediately begins to vomit. What action should the nurse implement first? Turn the client to a lateral position 79. A client is admitted to the surgical unit with symptoms of a possible intestinal obstruction. When preparing to insert a nasogastric (NG) tube, which intervention should the nurse implement? A. Elevate the head of the bed 60 to 90 degrees B. Measure from corner of mouth to angle of jaw C. Administer a PRN analgesic D. Assess for a gag reflex 80. A woman with an anxiety disorder calls her obstetrician’s office and tells the nurse of increased anxiety since the normal vaginal delivery of her son three weeks ago. Since she is breastfeeding, she stopped taking her antianxiety medications, but thinks she may need to start taking them again because of her increased anxiety. What response is best for the nurse to provide this woman? Inform her that some antianxiety medications are safe to take while breastfeeding 81. At the end of a preoperative teaching session on pain management techniques, a client starts to cry and states, “I just know I can’t handle all the pain.” What is the priority nursing diagnosis for this client? Anxiety 82. In early septic shock states, what is the primary cause of hypotension? A. Cardiac failure B. A vagal response C. Peripheral vasoconstriction D. Peripheral vasodilation 83. The charge nurse observes a new nurse preparing to insert an intravenous (IV) catheter. The new nurse has gathered supplies, including intravenous catheters, an intravenous insertion kit, and a 4x4 sterile gauze dressing to cover and secure the insertion site. What action should the charge nurse take? A. Plan to observe the secured IV site after the insertion procedure B. Confirm that the nurse has gathered the necessary supplies C. Remind the nurse to tape the gauze dressing securely in place D. Instruct the nurse to use a transparent dressing over the site 84. An adult client comes to the clinic and reports his concern over a lump that “just popped up on my neck about a week ago.” In performing an examination of the lump, the nurse palpates a large, non-tender, hardened left subclavian lymph node. There is no overlying tissue inflammation. What do these finding suggest? A. Bacterial infection B. Lymphangitis C. Malignancy D. Viral infection 85. The nurse is preparing to administer an IV dose of ciprofloxacin to a client with urinary tract infection. Which client data requires the most immediate intervention by the nurse? A. Urine culture positive for MRSA B. Serum sodium of 145 mEq/L (145 mmol/L SI) C. Serum creatinine of 4.5 mg/dl (398 mcmol/L SI) D. White blood cell count of of 12,000 mm3 (12 x 109/L SI) 86. The unit clerk reports to the charge nurse that a healthcare provider has written several prescriptions that are illegible and it appears the healthcare provider used several unapproved abbreviations in the prescriptions. What actions should the charge nurse take? A. Complete and file an incident (variance) report B. Call the healthcare provider who wrote the prescription C. Contact the healthcare provider review board for instructions D. Report the situation to the house supervisor 93. A confused, older client with Alzheimer’s disease becomes incontinent of urine when attempting to find the bathroom. Which action should the nurse implement? A. Instruct the client to use the call button when a bedpan is needed B. Apply adult diapers after each attempt to void C. Check residual urine volume using an indwelling urinary catheter D. Assist the client’s to a bedside commode every two hours 94. The nurse discovers that an elderly client with no history of cardiac or renal disease has an elevated serum magnesium level. To further investigate the cause of this electrolyte imbalance, what information is most important for the nurse to obtain from the client’s medical history? Frequency of laxative use for chronic constipation 95. The nurse is caring for a group of clients with the help of a practical nurse (PN). Which nursing actions should the nurse assign to the PN? (Select all that apply.) A. Administer a dose of insulin per sliding scale for a client with type 2 diabetes mellitus (DM). B. Obtain postoperative vital signs for a client one day following unilateral knee arthroplasty C. Perform daily surgical dressing change for a client who had an abdominal hysterectomy D. Initiate patient controlled analgesia (PCA) pumps for two clients immediately postoperative E. Start the second blood transfusion for a client twelve hours following a below knee amputation 96. In caring for a client who is receiving linezolid IV for nosocomial pneumonia, which assessment finding is most important for the nurse to report to the healthcare provider? A. Watery diarrhea B. Yellow-tinged sputum C. Increased fatigue D. Nausea and headache 98. The nurse is preparing to mix two medications from two different multidose vials, A and B. In which order should these actions be implemented when drawing the solutions from the vials? (Arrange from first on top to last on the bottom) Verify the drug and dose with the label on the vial Inject the volume of air to be aspirated from each vial Aspirate the desired volume from vial A Aspirate the desired volume from vial B 104. An 11-year-old client is admitted to the mental health unit after trying to run away from home and threatening self-harm. The nurse establishes a goal to promote effective coping, and plans to ask the client to verbalize three ways to deal with stress. Which activity is best to establish rapport and accomplish this therapeutic goal? A. Bring the client to the team meeting to discuss the treatment plan B. Ask the client to write feeling in a journal and then review it together C. Explain the purpose of each medication the client is currently taking D. Play a board game with the client and begin taking about stressors 105. An adult male with schizophrenia who has been noncompliant in taking oral antipsychotic medications refuses a prescribed IM medication. Which action should the nurse take? A. Notify the healthcare provider of the client’s refusal B. Administer an oral PRN medication for agitation C. Ask for staff assistance with administering the injection D. explain that oral medications will no longer be required 106. An older male client with a history of diabetes mellitus, chronic gout, and osteoarthritis comes to the clinic with a bag of medication bottles. Which intervention should the nurse implement first? A. Record pain evaluation B. Assess blood glucose C. Identify pills in the bag D. Obtain a medical history 107. A male client with an antisocial personality disorder is admitted to an in-patient mental health unit for multiple substance dependency. When providing a history, the client justifies to the nurse his use of illicit drugs. Based on this pattern of behavior this client’s history is most likely to include which finding? A. Phobias and panic attacks when confronted by authority figures. B. Suicidal ideations and multiple attempts/ C. Multiple convictions for misdemeanors and class B felonies. D. Delusions of grandiosity and persecution 108. An adult male who fell from a roof and fractures his left femur is admitted for surgical stabilization after having a soft cast applied in the emergency department. Which assessment finding warrants immediate intervention by the nurse? A. Onset of mild confusion B. Pain score 8 out of 10 C. Pale, diaphoretic skin D. Weak palpable distal pulses 109. A client who has a suspected brain tumor is schedules for a computed (CT) scan. When preparing the client for the client for the CT scan, which intervention should the nurse implement? A. Determine if the client has had a knee or hip replacement B. Immobilize the client’s neck before moving onto stretcher C. Give an antiemetic to control nausea D. Obtain the client’s food allergy history 110. A client who is at 10-weeks gestation calls the clinic because she has been vomiting for the past 24 hours. The nurse determines that the client has no fever. Which instructions should the nurse give to this client? A. Remain on clear liquids until the vomiting subsides B. Come to the clinic to be seen by a healthcare provider C. Make an appointment at the clinic if a fever occurs D. Take nothing by mouth until there is no more nausea 111. The nurse is preparing to gavage feed a premature infant through an orogastric tube. During insertion of the tube, the infant’s heart rate drops to 60 beats / minute. Which action should the nurse take? A. Continue the insertion since this is a typical response B. pause and monitor for a continues drop of the heart rate C. Insert the feeding tube into the infant’s nasal passage D. Postpone the feeding until the infant’s vital signs and stable An infant is receiving gavage feedings via nasogastric tube. At the beginning of the feeding, the infant’s heart rate drops to 80 beats / minute. What action should the nurse take? Slow the feeding and monitor the infant’s response. 112. A male client is admitted with a bowel obstruction and intractable vomiting for the last several hours despite the use of antiemetics. Which intervention should the nurse implement first? (Please scroll and view each tab’s information in the client’s medical record before selecting the answer.) 113. While removing staples from a male client’s postoperative wound site, the nurse observes that the client’s eyes are closed and his face and hands are clenched. The client states, “I just hate having staples removed.” After acknowledging the client’s anxiety, what action should the nurse implement? Attempt to distract the client with general conversation 114. A client is being treated for syndrome of inappropriate antidiuretic hormone (SIADH). On examination, the client has a weight gain of 4.4 lbs (2 kg) in 24 hours and an elev ated blood pressure. Which intervention should the nurse implement first? A. Ensure client takes a diuretic q AM B. Obtain serum creatinine levels daily C. Measure ankle circumference D. Monitor daily sodium intake 115. The nurse and an unlicensed assistive personnel (UAP) are providing care for a client with a nasogastric tube (NGT) when the client begins to vomit. How should the nurse manage this situation? A. Determine the presence of hematemesis as the UAP irrigates the NGT B. Instruct the UAP to bring an antiemetic to the nurse at the bedside C. Assess the appearance of the emesis while the UAP checks bowel sounds D. Direct the UAP to measure the emesis while the nurse irrigates the NGT 116. A preschooler with constipation needs to increase fiber intake. Which snack suggestion should the nurse provide? A. soft pretzels B. fruit-flavored yogurt C. oatmeal cookies D. low fat cheese sticks 117. The mother of a 7-month-old brings the infant to the clinic because the skin in the diaper area is excoriated and red, but there are no blisters or bleeding. The mother reports no evidence of watery stools. Which nursing intervention should the nurse implement? Instruct the mother to change the child’s diaper more often. Encourage the mother to apply lotion with each diaper charge Tell the mother to cleanse with soap and water at each diaper change Ask the mother to decrease the infant’s intake of fruits for 24 hours. 118. After multiple attempts to stop drinking, an adult male is admitted to the medical intensive care unit (MICU) with delirium tremens. He is tachycardic, diaphoretic, restless, and disoriented. Which finding indicates a life- threatening condition? A.CIWA-Ar for alcohol withdrawal score of 30 A. Acute onset of unrelenting chest pain C. Widening QRS complexes and flat waves D. Intense tremor and involuntary muscle activity 125. The home health nurse is preparing to make daily visits to a group of clients. Which client should the nurse visit first? A. A client with congestive heart failure who reports a 3 pound weight gain in the last two days B. An immobile client with a stage 3 pressure ulcer on the coccyx who is having low back pain C. A client diagnosed with chronic obstructive pulmonary disease (COPD) who is short of breath D. A terminally ill older adult who has refused to eat or drink anything for the last 48 hours 126. A female client is admitted for diabetic crisis resulting from inadequate dietary practices. After stabilization, the nurse talks to the client about her prescribed diet. What client characteristic is most import for successful adherence to the diabetic diet? A. Knows that insulin must be given 30 min before eating B. Frequently eats fruits and vegetables at meals and between meals/ C. Has someone available who can prepare and oversee the diet D. Demonstrates willingness to adhere to the diet consistently 127. A client currently receiving an infusion labeled Heparin Sodium 25,000 Units in 5% Dextrose Injection 500 mL at 14 mL/hour. A prescription is received to change the rate of the infusion to 900 units of Heparin per hour. The nurse should set the infusion pump to deliver how many mL/hour? (Enter numeric value only). 700 Rationale: D/H x Q = 25000 / 500 x 14 = 700 128. Oxygen at 5l/min per nasal cannula is being administered to a 10 year old child with pneumonia. When planning care for this child, what principle of oxygen administration should the nurse consider? A. Taking a sedative at bedtime slows respiratory rate, which decreases oxygen? B. Avoid administration of oxygen at high levels for extendedperiods. C. Increase oxygen rate during sleep to compensate for slower respiratory rate. D. Oxygen is less toxic when it is humidified with a hydration source. 129. The nurse is caring for a client with acute kidney injury (AKI) secondary to gentamicin therapy the client’s serum blood potassium is elevated, which finding requires immediate action by the nurse? A. Tall peak T waves on the cardiac monitor B. Peripheral pitting edema at 2 + indentation C. Serum creatinine above 0.5 mg/dl or 44.2 micro-mmol/dl D. Anuria for the last 12 hours. 130. A client presents to the labor and delivery unit, screaming “THE BABY IS COMING” which action should the nurse implement first. Observe the perineum VIDEO 131. During orientation, a newly hired nurse demonstrates suctioning of a tracheostomy in a skills class, as seen in the video. After the demonstration, the supervising nurse expresses concern that the demonstrated procedure increased the client’s risk for which problem? A. Infection B. Ineffective airway clearance C. Altered comfort D. Impaired gas exchange 132. One day after abdominal surgery, an obese client complains of pain and heaviness in the right calf. What action should the nurse implement? Observe for unilateral swelling 133. A male client with diabetes mellitus type 2, who is taking pioglitazone PO daily, reports to the nurse the recent onset of nausea, accompanied by dark-colored urine, and a yellowish cast to his skin. What instructions should the nurse provide? A. “You have become dehydrated from the nausea. You will need to rest and increase fluid intake” B. “you need to seek immediate medical assistance to evaluate the cause of these symptoms” C. A urine specimen will be needed to determine what kind of infection you have developed” D. use insulin per sliding scale until the nausea resolves, and then resume your oral medication” 134. A male client with ulcerative colitis received a prescription for a corticosteroid last month, but because of the side effect he stopped taking the medication 6 year ago. Which finding warrants immediate intervention by the nurse? A. Hypotension and fever B. Anxiety and restlessness. C. Fluid retention D. Increased blood glucose. 138. A client in the intensive care unit is being mechanically ventilated, has an indwelling urinary catheter in place, an exhibiting signs of restlessness. Which action should the nurse take fist? Administer PRN dose of lorazepam Auscultate bilateral breath sounds Check urinary catheter for obstruction Review the heart rhythms on cardiac monitor. 139. A young adult female client with recurrent pelvic pain for 3 year returns to the clinic for relief of severe dysmenorrhea. The nurse reviews her medical record which indicates that the client has endometriosis. Based on this finding, what information should the nurse provide this client? A) Oral contraceptives increase the symptoms of endometriosis. B) The symptoms of endometriosis can increase with menopause. C) An option to diagnose disease extent and provide therapeutic treatment is laparoscopy. D) Infertile is successfully treated with removal of intra-abdominal endometrial lesions. 140. A 75-year-old female client is admitted to the orthopedic unit following an open reduction and internal fixation of a hip fracture. On the second postoperative day, the client becomes confused and repeatedly asks the nurse she is. What information for the nurse to obtain? A. Use of sleeping medications. B. History of alcohol use, C. Use of antianxiety medications, D. History of this behavior. 141. To reduce the risk of being named in malpractice lawsuit, which action is most important for the nurse to take? A. Establish a trusting nurse-client relationship. B. Complete an incident report following a client injury. C. Maintain current professional malpractice insurance, D. Adhere consistently to standards of care. 142. A client with multiple sclerosis is receiving beta-1b interferon every other day. To assess for possible bone marrow suppression caused by the medication, which serum laboratory test findings should the nurse monitor? (Select all that apply) A. Platelet count B. Red blood cell count (RBC) C. White blood cell count (WBC). D. Albumin and protein E. Sodium and potassium 143. Which assessment is more important for the nurse to include in the daily plan of care for a client with a burned extremity? Distal pulse intensity VIDEO 144. The nurse is auscultating a client’s lung sounds. Which description should the nurse use to document this sound? (Please listen to the audio file to select the option that applies.) https://www.youtube.com/watch?v=VGDdqtIhUdA High pitched or fine crackles. High pitched wheeze Rhonchi Stridor 151. The nurse needs to add a medication to a liter of 5% Dextrose in Water (D5W) that is already infusing into a client. At what location should the nurse inject the medication? 153. A client with a liver abscess develops septic shock. A sepsis resuscitation bundle protocol is initiated and the client receives a bolus of IV fluids. Which parameter should the nurse monitor to assess effectiveness of the fluid bolus? A. Mean arterial pressure (MAP) B. White blood cell count C. Blood culture D. Oxygen saturation 154. A 17-year –old male is brought to the emergency department by his parents because he has been coughing and running a fever with flu-like symptoms for the past 24 hours. Which intervention should the nurse implement first? A. Obtain a chest X-ray per protocol. B. Place a mask on the client’s face. C. Assess the client’s temperature. D. Determine the client’s blood pressure 155. An older client is admitted for repair of a broken hip. To reduce the risk for infection in the postoperative period, which nursing care interventions should the nurse include in the client’s plan of care? (Select all that apply) A. Teach client to use incentive spirometer q2 hours while awake. B. Remove urinary catheter as soon as possible and encourage voiding. C. Maintain sequential compression devices while in bed. D. Administer low molecular weight heparin as prescribed E. Assess pain level and medicate PRN as prescribed. 156. A client is scheduled to receive an IW dose of ondansetron (Zofran) eight hours after receiving chemotherapy. The client has saline lock and is sleeping quietly without any restlessness. The nurse caring for the client is not certified in chemotherapy administration. What action should the nurse take? Ask a chemotherapy-certified nurse to administer the Zofran Administer the ondasentron (Zofran) after flushing the saline lock with saline Hold the scheduled dose of Zofran until the client awakens Awaken the client to assess the need for administration of the Zofran. 158. The nurse note a visible prolapse of the umbilical cord after a client experiences spontaneous rupture of the membranes during labor. What intervention should the nurse implement immediately? Elevate the presenting part off the cord. 159. While visiting a female client who has heart failure (HF) and osteoarthritis, the home health nurse determines that the client is having more difficulty getting in and out of the bed than she did previously. Which action should the nurse implement first? Inquire about an electric bed for the client’s home use Submit a referral for an evaluation by a physical therapist. Explain the usual progression of osteoarthritis and HF Request social services to review the client’s resources. 160. A client is admitted to a mental health unit after attempting suicide by taking a handful of medications. In developing a plan of care for this client, which goal has the highest priority? B. Signs a no-self-harm contract. B. Sleep at least 6 hours nightly. C. Attends group therapy every day D. Verbalizes a positive self-image. The nurse is ready to insert an indwelling urinary catheter as seen in the picture. At this point in the procedure, what actions should the nurse take before inserting the catheter? (Select all that apply) A. Ask the client to bear down as if voiding to relax the sphincter B. Complete perianal care with soap and water C. Gently palpate the client’s bladder for distention D. Hold the catheter 3 – 4 inches (7.5 – 10 cm) from its tip E. Secure the urinary drainage bag to the bed frame [Show More]

Last updated: 1 year ago

Preview 1 out of 124 pages

Reviews( 0 )

$10.00

Add to cart

Instant download

Can't find what you want? Try our AI powered Search

OR

GET ASSIGNMENT HELP
91
0

Document information


Connected school, study & course


About the document


Uploaded On

Nov 16, 2020

Number of pages

124

Written in

Seller


seller-icon
quiz_bit

Member since 3 years

552 Documents Sold


Additional information

This document has been written for:

Uploaded

Nov 16, 2020

Downloads

 0

Views

 91

Document Keyword Tags

Recommended For You


$10.00
What is Browsegrades

In Browsegrades, a student can earn by offering help to other student. Students can help other students with materials by upploading their notes and earn money.

We are here to help

We're available through e-mail, Twitter, Facebook, and live chat.
 FAQ
 Questions? Leave a message!

Follow us on
 Twitter

Copyright © Browsegrades · High quality services·